XXII CURSO DE DERMATOPATOLOGÍA

Page 1

1


22º Curso de Dermatopatología

«El amor se bebe por la piel» La vieja sirena. José Luis Sampedro (1917-2013). 2


Profesores del curso Dra. Alegría Landa, Victoria Servicio de Dermatología. Hospital Universitario “Fundación Jiménez Díaz”, Madrid

Dr. Pinedo, Fernando Servicio de Anatomía Patológica Hospital de Alcorcón, Madrid

Dra. Beato Merino, María José Servicio de Anatomía Patológica Hospital Universitario “La Paz”, Madrid

Dra. Postigo, Concepción Servicio de Dermatología Hospital Universitario “12 de Octubre”, Madrid

Dra. Carrillo Gijón, Rosario Servicio de Anatomía Patológica Hospital Universitario “Ramón y Cajal”, Madrid

Dr. Requena Caballero, Luis Servicio de Dermatología. Hospital Universitario “Fundación Jiménez Díaz”, Madrid

Dra. Colmenero Blanco, Isabel Servicio de Anatomía Patológica Hospital del Niño Jesús, Madrid

Dr. Ríos Martín Juan José Servicio de Anatomía Patológica. Hospital Clínico Universitario “Virgen Macarena”, Sevilla

Dr. Cuevas Santos, Jesús Servicio de Anatomía Patológica Hospital Universitario de Guadalajara

Dra. Rodríguez Pinilla, María Socorro Servicio de Anatomía Patológica. Hospital Universitario “Fundación Jiménez Díaz”, Madrid

Dra. Eraña, Itziar Servicio de Anatomía Patológica. Hospital Universitario “Fundación Jiménez Díaz”, Madrid

Dr. Rodríguez Peralto, José Luís Servicio de Anatomía Patológica Hospital Universitario “12 de Octubre”, Madrid

Dra. Feito Rodríguez, Marta Servicio de Dermatología Hospital Universitario “La Paz”, Madrid

Dr. Roustán Guillón. Luis Gastón, Servicio de Dermatología. Hospital Universitario Puerta de Hierro. Majadahonda, Madrid

Dra. Fernández Figueras, María Teresa Servicio de Anatomía Patológica Hospital Universitario General de Cataluña-Grupo Quirón Salud. Barcelona

Dr. Santonja, Carlos Servicio de Anatomía Patológica Hospital Universitario “Son Palma de Mallorca

Dr. Fernández Flores, Ángel Servicio de Anatomía Patológica Hospital del Bierzo, Ponferrada, León

Espases”,

Dr. Santos Briz, Ángel Servicio de Anatomía Patológica Hospital Clínico Universitario de Salamanca

Dra. Fernández Guarino, Montserrat Servicio de Dermatología Hospital Universitario “Ramón y Cajal”, Madrid

Dr. Saus, Carlos Servicio de Anatomía Patológica Hospital Universitario “La Princesa”, Madrid

Dra. Garrido Ruiz, María Servicio de Anatomía Patológica Hospital Universitario 12 de Octubre, Madrid

Dra. Suárez Masa, Dolores Servicio de Anatomía Patológica. Hospital Universitario Puerta de Hierro. Majadahonda, Madrid

Dra. González Muñoz, Patricia Servicio de Dermatología Hospital Universitario de Guadalajara

Dra. Velasco Benito, Verónica Servicio de Anatomía Patológica Hospital Universitario de Cruces, Barakaldo

Dr. Monteagudo Castro, Carlos Servicio de Anatomía Patológica Hospital Clínico Universitario, Valencia Dra. Nájera, Laura Servicio de Anatomía Patológica. Hospital Universitario Puerta de Hierro. Majadahonda, Madrid 3


22º Curso de Dermatopatología

Índice Profesores del curso

3

Índice

4

Introducción

7

Conferencias

9

CLAVES EN DERMATOPATOLOGÍA Dr. José Luis Rodríguez Peralto

11

ENFOQUE CLÍNICO E HISTOPATOLÓGICO DE LAS LESIONES BASALOIDES Dra. María Teresa Fernández Figueras

17

DIFERENTES CARAS DEL LUPUS ERITEMATOSO CUTÁNEO Dr. Luis Requena

21

MICOSIS CUTÁNEAS SUPERFICIALES Dr. Ángel Santos Briz-Terrón

25

Casos clínico patológicos Caso Caso Caso Caso Caso Caso Caso Caso Caso

clínico clínico clínico clínico clínico clínico clínico clínico clínico

patológico patológico patológico patológico patológico patológico patológico patológico patológico

29 Número Número Número Número Número Número Número Número Número

1 2 3 4 5 6 7 8 9

31 33 35 37 41 45 47 49 53

4


Casos del Seminario

57

Caso del Seminario Número 1

59

Caso del Seminario Número 2

61

Caso del Seminario Número 3

65

Caso del Seminario Número 4

69

Caso del Seminario Número 5

73

Caso del Seminario Número 6

77

Caso del Seminario Número 7

79

Caso del Seminario Número 8

81

Caso del Seminario Número 9

83

Caso del Seminario Número 10

85

Caso del Seminario Número 11

87

Caso del Seminario Número 12

89

Caso del Seminario Número 13

91

Caso del Seminario Número 14

93

Caso del Seminario Número 15

95

Caso del Seminario Número 16

99

Caso del Seminario Número 17

101

Caso del Seminario Número 18

103

Caso del Seminario Número 19

107

Caso del Seminario Número 20

111

Caso del Seminario Número 21

113

Caso del Seminario Número 22

117

Caso del Seminario Número 23

121

Caso del Seminario Número 24

125

Caso del Seminario Número 25

127

Caso del Seminario Número 26

131

Caso del Seminario Número 27

135

Caso del Seminario Número 28

139

Caso del Seminario Número 29

141

Contraportada y patrocinadores

144

5


22Âş Curso de DermatopatologĂ­a

6


Introducción Los cursos de Dermatopatología que se celebran anualmente en Madrid, al amparo de la División Española de la AIP y de la SEAP, se han hecho clásicos a lo largo de los 21 últimos años como actividad de formación continuada muy apreciada tanto por dermatólogos como por patólogos. En este 22º Curso, del año 2019, se mantiene el objetivo fundamental de demostrar que la Dermatopatología de calidad debe estar sustentada por diagnósticos objetivos, y dirigida primordialmente al beneficio del paciente, resultante del trabajo médico conjunto del dermatólogo clínico con suficiente formación histopatológica, y del patólogo con suficientes conocimientos dermatológicos. Tradicionalmente, estos cursos se han planteado con un doble enfoque clínico e histopatológico. En las conferencias, la exposición histopatológica del patólogo va precedida de una introducción clínica por parte de un dermatólogo. El Curso incluye ocho presentaciones clínico-patológicas de casos reales, especialmente seleccionados para demostrar la correlación clínico-patológica. El segundo objetivo es hacer énfasis en la objetividad y especificidad de los diagnósticos histopatológicos. Para ello, el patólogo deberá esforzarse en intentar llegar a un diagnóstico específico, sin contar de antemano con los datos clínicos. La sesión ya clásica de «Claves Diagnósticas», repetidamente solicitada en cursos precedentes, y el Seminario de preparaciones, que se distribuyen con escasos datos clínicos, tienen esta finalidad. El Seminario de este 22º Curso consta de 30 casos, enviados a todos los inscritos vía online. Es tradicional que los participantes evalúen la preparación, contenido y presentación de las conferencias, de las sesiones clínico patológicas, y de cada caso del Seminario, así como de los ponentes y del desarrollo del Curso, incluyendo una encuesta sobre posibles temas a tratar en próximas ediciones. Este 22º Curso se realiza de nuevo en la ubicación original, donde comenzó, en el Hospital 12 de Octubre, concretamente en el Salón de Actos de su Residencia General.

7


22Âş Curso de DermatopatologĂ­a

8


Conferencias

9


22Âş Curso de DermatopatologĂ­a

10


Claves en Dermatopatología Dr osé Luis Rodríguez P eralto tamento de Anatomía P atológica. Dr.. JJosé Peralto eralto.. Depar Departamento Patológica. Hospital Universitario 12 de Octubre (Madrid).

Fig 1.- Esclerodermia. Signo de la linea

nóstico. Por ello, se han creado varios signos clave para identificar esta enfermedad como: El signo de la biopsia cuadrada (Fig.3) definida como una biopsia en la que las cuatro esquinas son en ángulo recto, aproximadamente 90º, formando un cuadrado o un rectángulo; el signo del molde (Fig 3), en el que los bordes de la biopsia son perfectamente cortados y paralelos entre sí, adquiriendo la forma de un molde de cocina y el signo de las glándulas ecrinas altas en las que las glándulas ecrinas están dispuestas en los 2/3 superiores de la dermis. Estos signos son útiles sólo cuando la biopsia es un punch. En 2005, se acuñó un nuevo signo muy útil, el “de la línea” (Fig 1,2 y 3) caracterizado por la presencia de una interfase linear y bien definida entre el colágeno esclerótico y el tejido subcutáneo adyacente. Este signo es el más sensible para identificar una morfea, independientemente del tipo de biopsia, aunque es el menos específico, ya que también se puede observar en las necrobiosis lipoídicas y en los procesos esclerodermiformes de la enfermedad injerto contra huésped. No obstante, para poderlo observar, es necesario

La mayoría de las enfermedades cutáneas inflamatorias o tumorales se pueden diagnosticar fácilmente siguiendo los patrones clásicos histopatológicos de distribución de la inflamación, el tipo de célula inflamatoria y una buena correlación clínico-patológica. Sin embargo, otra herramienta útil que puede facilitar la clasificación de los diferentes procesos, es la presencia de algunos detalles histopatológicos específicos, observados en muchos de los casos, las claves, que pueden ayudar a realizar el diagnóstico histopatológico. Describimos a continuación algunas de estas claves EL SIGNO DE LA LINEA El sigo de la línea es típico de la morfea o esclerodermia localizada, una enfermedad cutánea que clínicamente se caracteriza por placas induradas de superficie lisa que afectan la piel y a veces el tejido celular subcutáneo. El diagnóstico, aunque se sospecha clínicamente, se confirma siempre por biopsia. En muchas ocasiones es difícil de hacer, ya que las lesiones se localizan en piel del tronco y en esta zona, la dermis es gruesa, lo que dificulta su diag11


22º Curso de Dermatopatología

Fig. 2.- Esclerodermia. Signo de la linea

Fig. 3.- Esclerodermia. Signo de la línea, biopsia cuadrada y molde

que la biopsia incluya tejido celular subcutáneo. co de lupus. No obstante en muchas ocasiones, existen dificultades importantes para establecer el diagnóstico diferencial entre ambas entidades. En estos casos, la presencia de grupos de células dendríticas plasmocitoides CD123 + facilita enormemente el diagnóstico, ya que son típicas del lupus eritematoso y en el contexto de una alopecia cicatricial, permite efectuar el diagnóstico de alopecia cicatricial secundaria a lupus eritematoso.

CD3 Y CD8 INTRAFOLICULAR EN UNA ALOPECIA CON MINIATURIZACIÓN DE LOS FOLÍCULOS La presencia de CD3 y CD8 dentro del folículo piloso mimiaturizado es una clave diagnóstica de Alopecia areata. El diagnóstico diferencial entre la alopecia areata y la alopecia androgenética no es fácil, especialmente cuando la alopecia areata se produce en áreas androgenéticas. Como es bien conocido, en ambas alopecias se produce una inversión de la proporción de folículos en anagen en relación con catagen o telogen, sin cicatrices fibrosas, aunque si con estelas fibrosas. En estos casos difíciles clínicamente, nos podemos ayudar de la inmunohistoquímica. Cuando la densidad de CD3 y CD8 es superior al 50% del infiltrado inflamatorio intrafolicular, podemos diagnosticar el caso como de alopecia areata. Además, la presencia intrafolicular de CD3 en más del 50% es el test más sensible y específico de alopecia areata (Fig. 4).

PROLIFERACIÓN MELANOCÍTICA PROFUNDA DE FORMA TRIANGULAR CON PIGMENTACIÓN EN DAMERO Es una clave de nevus penetrante profundo (Fig. 5 y 6). Las lesiones melanocíticas benignas adquiridas suelen ocupar y expandir la dermis papilar, a veces con conexión con la epidermis suprayacente. Su extensión a la dermis profunda es habitual en los nevus congénitos, donde se identifican dos clonos diferentes, uno en la dermis papilar y otro en la reticular. Por otra parte, los nevus azules son proliferaciones melanocíticas fusocelulares en dermis reticular que nunca alcanzan la dermis papilar ni la epidermis. Sin embargo, a veces nos encontramos con proliferaciones melanocíticas polimorfas y pleomorfas, conectadas con la epidermis que adquieren una morfología triangular, de base superficial, que se extienden a la dermis reticular y que están constituidas por varios tipos de células melánicas: redondeadas con halos claros, fusiformes, pleomorfas, y epitelioides. Este pleomorfismo celular y su extensión profunda, sugieren en ocasiones un diagnóstico de malignidad. Sin embargo, la presencia de pigmentación en damero o tablero de ajedrez, en el contexto de una proliferación melanocítica pleomórfica triangular con células epitelioides claras, facilita

ACÚMULOS DE CÉLULAS DENDRÍTICAS PLASMOCITOIDES CD123 + EN ALOPECIA CICATRICIAL La presencia de grupos de células dendríticas plasmocitoides CD123 + en el seno de una alopecia cicatricial permite distinguir un lupus de un liquen plano. El diagnóstico de una alopecia cicatricial es a veces complicado, aunque lo facilita la presencia de cicatrices fibrosas que sustituyen folículos pilosos terminales en anagen y la existencia de conductos sudoríparos dilatados tipo siringoma con inflamación acompañante. El reto más frecuente en la alopecia cicatricial es diferenciar un origen en lupus o en liquen plano. La fibrosis mixoide infundibular acompañada de infiltrado linfocitario alto, apunta a liquen plano, mientras que cuando el infiltrado es profundo en el bulbo, es más diagnósti12


Fig. 4.- Alopecia areata. CD3

Figs. 5 y 6.- Nevus penetrante profundo

enormemente el diagnóstico de nevus penetrante profundo.

te diagnosticarlos como amiloidoma AL (Fig 8), tras descartar una amiloidosis sistémica previa, mieloma u otras enfermedades de las cadenas ligeras. Algunos de estos casos se asocian a linfomas B de la zona marginal, primarios cutáneos, por lo que al menos un pequeño grupo de amiloidomas AL cutáneos puedan representar una manifestación inusual de un linfoma B cutáneo de la zona marginal.

DEPÓSITO EOSINOFILO AMORFO NODULAR DERMICO Es una clave de Amiloidosis nodular o amiloidoma, una entidad infrecuente, caracterizada histopatológicamente por la presencia de un nódulo amorfo eosinófilo que ocupa la dermis reticular y frecuentemente el tejido celular subcutáneo (Fig 7). Esta masa de material eosinófilo es fluorescente con la luz ultravioleta tras tinción con Tioflavina y adquiere color verde manzana tras tinción con Rojo Congo y someterse a la luz polarizada. Un dato relevante en algunos de estos casos es la presencia de grupos de células plasmáticas con restricción de cadenas ligeras, Kappa o Lambda. La proteína que se deposita en estos pacientes es AL, lo que permi-

BIBLIOGRAFÍA 1. Draznin M, Fung M. The line sign. J Cutan Pathol. 2006;33:80–81. 2. Sarah Y, Draznin M, and Fung MA. The “Line Sign” Is a Rapid and Efficient Diagnostic “Test” for Morphea: Clinicopathological Study of 73 Cases. Am J Dermatopathol 2018;40:873–878 13


22Âş Curso de DermatopatologĂ­a

Fig. 7 y 8.- Amiloidoma AL

14


3. Fung MA. Inflammatory diseases of the dermis and epidermis. In: Busam KJ, ed. Dermatopathology. 2nd ed. Philadelphia, PA: Elsevier Saudners; 2015:71–72. 4. Winfield H, Jaworsky C. Connective tissue diseases. In: Elder DE, ed. Lever’s Histopathology of the Skin. 11th ed. Alphen aan den Rijn: Wolters Kluwer; 2015:348–349. 5. Ferringer T. Alterations in collagen and elastin. In: Elston DM, Ferringer T, ed. Dermatopathology. 2nd ed. Philadelphia, PA: Elsevier Saunders; 2014:214–215 6. Kamyab K. Rezvani M, Seirafi H, Mortazavi S, Teymourpour A, Abtahi S, Nasimi M. Distinguishing immunohistochemical features of alopecia areata from androgenic alopecia. 7. Kolivras A1, Thompson C2. Distinguishing diffuse alopecia areata (AA) from pattern hair loss (PHL) using CD3 (+) T cells. J Am Acad Dermatol. 2016; 74(5):937-44. 8. Clusters of CD123 plasmacytoid dendritic cells help distinguish lupus alopecia from lichen planopilaris. Kolivras A, Thompson C. J Am Acad Dermatol 2016; 74 (6): 1267-1269 9. McNiff JM, Kaplan DH. Plasmacytoid dendritic cells are present in cutaneous dermatomyo-

sitis lesions in a pattern distinct from lupus erythematosus. J Cutan Pathol. 2008;35:452-456. 10. Brown TT, Choi E-YK, Thomas DG, Hristov AC, Chan MP. Comparative analysis of rosacea and cutaneous lupus erythematosus: histopathologic features, T-cell subsets, and plasmacytoid dendritic cells. J Am Acad Dermatol. 2014;71:100107. 11. Liau JY, Chuang SS, Chu C-Y, Ku WH, Tsai JH, Shih TF. The presence of clusters of plasmacytoid dendritic cells is a helpful feature for differentiating lupus panniculitis from subcutaneous panniculitis-like T-cell lymphoma. Histopathology. 2013;62: 1057-1066. 12. Walsh NM1, Lano IM, Green P, Gallant C, Pasternak S, Ly TY, Requena L, Kutzner H, Chott A, Cerroni L. AL Amyloidoma of the Skin/Subcutis: Cutaneous Amyloidosis, Plasma Cell Dyscrasia or a Manifestation of Primary Cutaneous Marginal Zone Lymphoma? Am J Surg Pathol. 2017 Aug;41(8):1069-1076. 13. Gargallo V, Angulo L, Hernández E, Ropdríguez-Peralto JL, Zarco C. Massive Subcutaneous Masses on the Back Related to ²2-MicroglobulinAmyloidosis. JAMA Dermatol. 2015 May;151(5):564-5

15


22Âş Curso de DermatopatologĂ­a

16


Enfoque clínico e histopatológico de las lesiones basaloides Dra. María Ter esa F ernández Figueras. Ser vicio de Anatomía P atológica eresa Fernández Servicio Patológica Hospital Universitari General de Catalunya - Grupo Quirón Salud. Barcelona.

Figura 1

Las proliferaciones de células basaloides son uno de los procesos más frecuentes en dermatopatología. Se definen como lesiones constituidas por células que recuerdan a las de la capa basal de la epidermis y van desde lesiones reactivas, inducidas a veces por las células estromales, a tumores de alto grado. Pese a su frecuencia, muchos aspectos diagnósticos están mal definidos y éste hecho puede repercutir tanto en el tratamiento individualizado de los pacientes, como en las conclusiones obtenidas en estudios clínicos o ensayos terapéuticos. El carcinoma basocelular es considerado el pro-

totipo de los tumores de células basaloides, pese a que citológicamente no se parece las células basales de la epidermis sino a las de los gérmenes foliculares con los que también comparte la expresión de Ber-EP4 y la disposición en empalizada periférica (mucho menos evidente la basal de la epidermis). Uno de los hallazgos más característicos del carcinoma basocelular es la presencia de hendiduras en torno a los nidos tumorales. Usualmente se dice que estas hendiduras aparecen entre el epitelio y el estroma, aunque la expresión de Ber-EP4 en ambos lados de la hendidura sugiere que se producen 17


22º Curso de Dermatopatología

en el seno de la propia membrana celular, ya que Ber-EP4 (EpCAM) es una glicoproteína de membrana. Esta retracción es un artefacto que típicamente aparece en las biopsias procesadas en parafina, pero no en los cortes congelados. Tampoco se observa habitualmente hendiduras en las biopsias que han sido inicialmente congeladas y posteriormente procesadas en parafina. En contra de la creencia de que el carcinoma basocelular es un tumor de aspecto relativamente monomorfo y por tanto constituye el modelo ideal para el desarrollo de algoritmos diagnósticos mediante inteligencia artificial, este tumor tiene una gran variabilidad tanto en patrones de crecimiento como rasgos citológicos y tipos de diferenciación. La clasificación del carcinoma basocelular según el patrón de crecimiento es probablemente la más relevante en cuanto a su potencial biológico y en la que se basan la mayor parte de decisiones terapéuticas. La clasificación más internacionalmente aceptada y que aparece en la última edición de la WHO divide estos tumores en lesiones de crecimiento nodular, infiltrante, esclerodermiforme, superficial y micronodular. No obstante, esta clasificación no es aplicada de manera uniforme en el estudio histopatológico entre diferentes centros, ni siquiera en un mismo país dado que no existe un consenso en la definición e identificación de estos patrones. El carcinoma basocelular de patrón micronodular puede asociarse a otras variantes o aparecer de forma individual. En su crecimiento envía delgados tentáculos que invaden en profundidad causando en algunos nidos desmoplasia y en otros una completa ausencia de respuesta estromal, lo que lo hace aún más difícil de identificar (Fig 1). Es probablemente el tipo de carcinoma basocelular más agresivo, pero en algunos informes es confundido con los nódulos de pequeño tamaño que aparecen frecuentemente en la periferia de los carcinomas basocelulares de crecimiento nodular (Fig 2). Por ese motivo en algunos estudios se la ha considerado de bajo riesgo. Por otro lado, algunos dermatopatólogos incluyen esta variante micronodular en el grupo de los carcinomas basocelulares infiltrativos (pese tener a menudo bordes perfectamente redondeados) y por tanto no usan este término. Tampoco existe una clara diferenciación entre las definiciones de patrón esclerosante e infiltrativo, ni estudios que demuestren la validez y la necesidad de esta distinción. Finalmente, no existe unanimidad en cómo reportar lesiones mixtas, por ejemplo carcinomas basocelulares nodulares sobre

De arriba a abajo, figuras 2, 3, 4 y 5 18


Figura 6

da), mioepitelial (Fig 5 inferior derecha) o escamosa. Esta última debe distinguirse del carcinoma basoescamoso que contiene áreas de carcinoma escamoso infiltrante en transición con focos de carcinoma basocelular y cuyo pronóstico está ligado al del carcinoma escamoso infiltrante. El carcinoma escamoso no es el único tumor que puede aparecer combinado con un carcinoma basocelular, ya que puede entremezclarse con otros procesos como queratosis seborreicas o melanomas. La relación entre un melanoma y un carcinoma basocelular puede ser (1) colisión de tumores, (2) colonización de células del melanoma a los nidos de carcinoma basocelular, en cuyo caso pese a haber atravesado la membrana basal, las células del melanoma no han infiltrado directamente dermis, por lo que el comportamiento suele ser similar al de un melanoma in situ y (3) lesiones combinadas de carcinoma basocelular y melanoma, en las que además de colonización del carcinoma basocelular por células de melanoma existe infiltración directa del melanoma a dermis y estas áreas de infiltración son las que indicaran el pronóstico. Una posibilidad alternativa es la existencia de tumores de células muy primitivas que compartan características

superficiales, ni si es necesario indicar que tipo de tumor es el que afecta al margen de resección cuando esté está afectado. Además, pese a ser poco frecuentes, las variedades con tropismo por folículos pilosos o glándulas sudoríparas que aparecen de forma «aparentemente multifocal» entorno a esos elementos no tienen reflejo en esta clasificación (Fig 3). Las variantes citológicas del carcinoma basocelular son extraordinariamente polimorfas y van desde lesiones con células claras que simulan un hidradenoma nodular (Fig 4 superior izquierda) a otras con células en anillo de sello (Fig 4 superior derecha), células monstruosas (Fig 4 inferior derecha), con células granulares (Fig 4 inferior izquierda), células multinucleadas similares a las de Warthin-Finkeldey, acantólisis, elastofagocitosis, rosetas y pseudorosetas y pigmentación. En cuanto a los tipos de diferenciación, pese a ser el carcinoma basocelular un tumor básicamente del folículo piloso, puede mostrar diferenciación foliculosebáceo apocrina avanzada (Fig 5 superior derecha), ductal (Fig 5 superior izquierda), neuroendocrina con trabéculas que simulan un carcinoma de células de Merkel (Fig 5 inferior izquier19


22º Curso de Dermatopatología

de célula basaloide y melanocitaria con un patrón de crecimiento similar al del carcinoma basocelular, aunque no podría considerarse que se trate realmente de este tumor. Muchos tumores anexiales foliculares y de glándulas sebáceas están también constituidos por células de aspecto basaloide. Así mismo algunos carcinomas escamosos cutáneos usuales contienen células de aspecto basaloide que deben distinguirse la de los carcinomas escamosos basaloides asociados a HPV. Los carcinomas escamosos basaloides asociados HPV aparecen con mayor frecuencia en el epitelio de transición anal y en las vías aéreas superiores, pero se han descrito casos cutáneos cuyo pronóstico es más agresivo que el de los carcinomas escamosos usuales. La presencia de uno de estos tumores debe alertar sobre el riesgo de tener las lesiones de alto grado en otras localizaciones. Las queratosis seborreicas son lesiones extremadamente frecuentes con una amplia gama de presentaciones microscópicas cuya etiología es desconocida. Uno de los hallazgos microscópicos más característicos es la presencia de agregados de pequeñas células escamosas parecidas a las de la capa basal de la epidermis, que se entremezclan con células similares al del estrato espinoso rodeando pseudoquistes córneos. Eventualmente pueden dar lugar a carcinomas escamosos de tipo bowenoide, pueden contener nidos de células de Merkel benignas o agregados de células basaloides similares a las del carcinoma basocelular. Por último, podemos encontrar nidos de células basaloides similares a las del carcinoma basocelular en áreas de inducción epitelial. A veces tan prominentes que son parecen tricoepiteliomas (Fig. 6), compartiendo con ellos la frecuente colonización por células de Merkel (CK20+), lo que ayuda a descartar el diagnóstico de carcinoma basocelular.

sociations. J Am Acad Dermatol. 2019 Feb;80(2):303-317. 4. Cameron MC, Lee E, Hibler BP, Giordano CN, Barker CA, Mori S, Cordova M, Nehal KS, Rossi AM. Basal cell carcinoma: Contemporary approaches to diagnosis, treatment, and prevention. J Am Acad Dermatol. 2019 Feb;80(2):321339. 5. Cribier B. Les difficultés du diagnostic : du carcinome basocellulaire aux tumeurs trichoblastiques: From basal cell carcinoma to trichoblastic tumors: a diagnostic challenge. Ann Dermatol Venereol. 2018 Nov;145 Suppl 5:VS3-VS11. 6. Manzotti C, Ríos Martín JJ, Gutiérrez Domingo Á, Gutiérrez Cierco JL, González Cámpora R. Seborrheic keratosis with basal clear cells: Report of a case and review of the literature. J Cutan Pathol. 2017 Jun;44(6):600-602. 7. McFalls J, Okon L, Cannon S, Lee JB. Intraepidermal proliferation of Merkel cells within a seborrheic keratosis: Merkel cell carcinoma in situ or Merkel cell hyperplasia? J Cutan Pathol. 2017 May;44(5):480-485. 8. Georgescu M, Blödorn-Schlicht N, BöerAuer A. Induction of Desmoplastic Trichoepithelioma in a Dermatofibroma. Am J Dermatopathol. 2018 Dec;40(12):924-926.

REFERENCIAS 1. Ríos-Martín JJ, Moreno-Ramírez D, González-Cámpora R. What is the cause of retraction spaces associated with basal cell carcinoma? J Cutan Pathol. 2012 Jul;39(7):729-30. 2. WHO Classification of Tumours, 4th Edition 2019, Volume 11. Edited by Elder DE, Massi D, Scolyer RA, Willemze R 3. Cameron MC, Lee E, Hibler BP, Barker CA, Mori S, Cordova M, Nehal KS, Rossi AM. Basal cell carcinoma: Epidemiology; pathophysiology; clinical and histological subtypes; and disease as20


Diferentes caras del lupus eritematoso cutáneo Dr Dr.. Luis Requena. Ser vicio de Dermatolo gía. Fundación Jiménez Díaz, Univ ersidad Autónoma. Madrid. Servicio Dermatología. Universidad

Figura 1

LUPUS ERITEMATOSO (LE): CONCEPTO Enfermedad inflamatoria crónica, de etiología desconocida, incluida entre las enfermedades inmunitarias del tejido conectivo. TÉRMINOS CONFUSOS • LE discoide: Se ha utilizado tanto para designar lesiones con forma de disco o moneda, como para designar formas de LE sólo con lesiones cutáneas • LE crónico: Se ha utilizado para designar LE sólo con lesiones cutáneas en contraposición con el LE sistémico • LE diseminado: Se ha utilizado como sinónimo de LE sistémico y para designar formas de LE sólo con lesiones cutáneas extensas. 21


22º Curso de Dermatopatología

LUPUS ERITEMATOSO (LE) FORMAS CLÍNICAS CUTÁNEAS • Lupus eritematoso cutáneo benigno: - Localizado o discoide - Generalizado - Hipertrófico - Paniculitis lúpica • Lupus eritematoso cutáneo subagudo: - Forma anular-policíclica - Forma eritematoescamosa o psoriasiforme • Lupus eritematoso sistémico: - Eritema facial en alas de mariposa - Lesiones eritemato-edematosas de localizaciones múltiples - Erosiones y ulceras en mucosa bucal - Lesiones ampollosas - Paniculitis - Sin lesiones cutáneas

• Es más frecuente en mujeres y las lesiones se localizan preferentemente en cara, parte alta de escote y espalda, superficie de extensión de antebrazos y dorso de manos. • Dos formas clínicas 1) Forma eritematoescamosa o psoriasiforme 2) Forma anular o policíclica • Fotosensibiliad y alopecia difusa no cicatricial • Escasa afectación sistémica: Artralgias, fiebre, ligera afectación renal • Anticuerpos anti-Ro-SSa en la mayoría de los pacientes. Anti-DNAn sólo en el 15-20% de los pacientes LUPUS ERITEMATOSO SISTÉMICO (LES) LESIONES CUTÁNEAS • Más frecuentes en mujeres jóvenes y con lesiones preferentemente localizadas en la cara y áreas fotoexpuestas del escote. • Placas eritematosas en la cara, con disposición en vespertilio o en alas de mariposa, con telangiectasias, pero con escaso o nulo componente queratósico. • Lesiones en manos y en pies: Placas eritematosas, con telangiectasias en palmas y plantas y cara palmar de dedos de manos y pies. A veces simulan sabañones (chilblain-lupus). A veces úlceras en pulpejos. • Alopecia difusa no cicatricial. • Síndrome de Raynaud. • Úlceras orales, livedo reticularis. • Lesiones cutáneas más raras: Vasculitis, LE ampolloso, LE-liquen plano.

LUPUS ERITEMATOSO CUTÁNEO BENIGNO (LECB) • LOCALIZADO O DISCOIDE: Placa bien delimitada, preferentemente localizada en la cara o en el cuero cabelludo con: a) Borde eritematoso b) Área queratósica, con tapones córneos localizados en los orificios foliculares (al desprender la escama: “signo del rastrillo” o espigones córneos que dejan ver los orificios foliculares dilatados) c) Área central de atrofia: En el cuero cabelludo: Alopecia cicatricial • GENERALIZADO: Lesiones similares a las descritas, pero salpicadas por toda la superficie corporal. • HIPERTRÓFICO: Predomina el componente queratósico, lupus cretáceo de Devergie, lupus verrugoso: En cara y dorso de manos. • LUPUS TUMIDUS: Predomina el componente eritematoso con escaso o nulo componente queratósico. En cara y tronco. • PANICULITIS LÚPICA: En hombros, brazos y cara. Deja lipoatrofia.

LUPUS ERITEMATOSO SISTÉMICO (LES) LESIONES SISTÉMICAS • Fiebre, pérdida de peso, astenia. • Artritis o artralgia en el 90% de los casos: Las articulaciones más afectadas son las interfalángicas, muñecas y rodillas. • Afectación renal: Glomerulopatía mesangial. Lo más frecuente es la nefritis lúpica focal media de buen pronóstico. Más rara la nefritis proliferativa difusa de peor pronóstico. • Dolor pleural y derrame. Más rara afectación pulmonar. • Neuropatía periférica o psicosis. • Anomalías hematológicas: Anemia, leucopenia y trombopenia, anticoagulante lúpico, falsaserologia de lues +, elevación de la VSG, disminución de C3 y C4, anti-DNAn.

LUPUS ERITEMATOSO CUTÁNEO SUBAGUDO (LECS) • Lesiones cutáneas que evolucionan por brotes (a diferencia de las lesiones fijas del LECB o las agudas del LES). • El LECS ocupa un lugar intermedio entre LECB y el LES. 22


HISTOPATOLOGÍA DEL ERITEMATOSO • Epidermis con atrofia, hiperqueratosis, tapones córneos en los infundíbulos foliculares y acrosiringios. • Degeneración vacuolar de la hilera basal de la epidermis con cuerpos de Civatte. • Engrosamiento de la membrana basal. • Infiltrado linfocitario perivascular superficial y profundo y depósitos de mucina intersticial. • Formas especiales: * LE ampolloso: Ampolla subepidérmica, con neutrófilos. * LE tumidus: No afectación de la unión dermo-epidérmica. Infiltrado linfocitario perivascular y depósitos de mucina. * Paniculitis lúpica: Paniculitis lobulillar con linfocitos y células plasmáticas y nódulos linfoides. * LES: Atrofia epidérmica, escaso infiltrado, prominente degeneración vacuolar y a veces neutrófilos en la unión dermo-epidérmica

• Medicamentos con asociación definitiva: - Hidralazina - Procainamida - Isoniazida - Metildopa - Clorpromazina - Quinidina • Medicamentos con asociación probable: - Anticonvulsionantes - Antitiroideos - Penicilamina - Betabloqueantes - Litio • Medicamentos con asociación esporádica: - Ácido paraaminosalicílico - Estrógenos - Sales de oro - Penicilina - Griseofulvina - Reserpina - Tetraciclinas LE NEONATAL • Recién nacidos de madres con anticuerpos antiRo-SSa que en el momento del nacimiento ya presentan lesiones eritemato-escamosas e hiperqueratosis folicular afectando a la cara (sobre todo alrededor de los ojos) e histopatología de LE. • Bloqueo cardíaco congénito. • Fotosensibilidad, hepatoesplenomegalia, adenopatías, leucopenia y trombopenia. • Anticuerpos anti-Ro-SSa + en el 100% de los casos. • Las lesiones cutáneas y los anticuerpos antiRo-SSa desaparecen a los 6-12 meses. • Buen pronóstico cuando sólo existen lesiones cutáneas. Peor pronóstico cuando existe bloqueo cardíaco completo (20-30% de mortalidad). Si existe bloqueo cardíaco completo se requiere implantar un marcapasos.

INMUNOFLUORESCENCIA EN EL LUPUS ERITEMATOSO • Inmunofluorescencia directa: Banda lúpica positiva: Depósito lineal de Igs a lo largo de la unión dermo-epidérmica. A veces se observa un depósito granular o de cuerpos ovoides fluorescentes. En pacientes con LES se observa tanto en piel lesional como en piel sana, mientras que en los pacientes con LECB es más frecuentemente positiva en piel lesional que en piel sana. Más frecuentemente positiva en la piel de la cara que en la piel de otras áreas y en fases activas de la enfermedad. • Inmunofluorescencia indirecta: Anticuerpos antinucleares (ANA) en sustratos de lineas celulares de tumores humanos (Hep-2 o KB). Cuatro patrones: - Patrón homogéneo - Patrón de refuerzo periférico - Patrón moteado - Patrón nucleolar LE INDUCIDO POR FÁRMACOS • Aparición de clínica de LE, con ANA positivos al tomar un medicamento y desaparición de la clínica y las anomalías de laboratorio al suspender el medicamento. • Anticuerpos anti-histonas positivos. 23


22Âş Curso de DermatopatologĂ­a

24


Micosis cutáneas superficiales Dr el Santos-Briz Ter rón. Dr.. Áng Ángel errón. Ser vicio de Anatomía P atológica. Complejo Asistencial Univ ersitario de Salamanca Servicio Patológica. Universitario

Figuras 1 y 2

hincapié en las claves dermatopatológicas que ayudan al diagnóstico. Se tratarán de forma individualizada las siguientes infecciones:

Las micosis cutáneas superficiales son aquellas infecciones fúngicas que afectan al estrato córneo de la piel y anejos (pelo y uñas). 1, 2,3 Existen múltiples factores que contribuyen a la aparición de micosis superficiales, no siendo los hongos por sí solos causa suficiente. Entre ellos destacan diferentes estados de inmunosupresión (infección por VIH o SIDA, corticoterapia, quimioterapia...), diabetes mellitus, antibioterapia, obesidad, embarazo y lactancia. Existen además factores ambientales contribuyentes como el calor, humedad (condicionan la maceración), utilización de ropa oclusiva o una deficiente higiene corporal.2,3 En esta charla se comentan las características histológicas de las micosis superficiales, haciendo

Dermatofitos3 Hongos filamentosos (patógenos primarios sin forma colonizadora) con actividad queratinasa que determina su apetencia por pelo, piel y uñas. Presentan, por tanto, predilección por tejidos queratinizados de hombres y animales. Se clasifican según el origen de la infección y su hábitat natural en geofílicos (suelo), zoofílicos (animales) y antropofílicos (hombre)1(ver TABLA 1). Se conoce que las especies zoofílicas producen patología más intensa y de evolución aguda, mien25


22º Curso de Dermatopatología

TABLA 1. Clasificación de los hongos dermatofitos según su hábitat natural. Se indica la forma de transmisión más frecuente, patrón epidemiológico de aparición más frecuente y género y especie más representativos.5

oportunistas. Se considera que el 99% de los cultivos positivos en laboratorios de micología humana lo son para las 10 especies más frecuentes y ubicuas: E. floccosum, M. canis, M. g ypseum, T.rubrum, T.mentagrophytes, T. tonsurans, M. audouinii, T.verrucosum, T.violaceum, T.schoenleinii. De entre éstas, son las 6 primeras las más frecuentes en nuestro medio (ver Tabla 2).

tras que la especies antropofílicas producen una respuesta inflamatoria moderada y con tendencia a la cronicidad. Reciben el nombre genérico de TIÑAS y afectan más frecuentemente piel y uñas (45% cada una) y menos frecuentemente al pelo (10%) 1. Existen tres géneros3: - TRICHOPHYTON spp.: Es el género dermatofítico más frecuente. Existen múltiples especies, clásicamente se ha definido al T. rubrum como la especie más frecuente causante de la gran mayoría de tiñas pedis, ungium, cruris y corporis. No obstante, hay datos que señalan a la especie T. mentagrophytes (variante intergiditale y mentagrophytes) como la más frecuente en España1. Otras especies se encuentran delimitadas geográficamente como el T. concentricum (Oceanía) o T. soudanense (África). Es posible que, en los próximos años por la inmigración4, se vea incrementada la frecuencia de otros como el T. violaceum y T. tonsurans (de origen africano). 5 - MICROSPORUM spp.: destaca M. canis, causante de la mayor parte de tiñas capitis. Especies como M. audouinii(tiña capitis originaria de África) y M. gypseum (tiña capitisy corporis en pacientes en estrecho contacto con tierra/suelo) son opciones a valorar, aunque mucho menos frecuentes.6,7 - EPIDERMOPHYTON spp.: la especie más relevante es E. floccosum, siendo la causa más frecuente de tiña cruris en adultos jóvenes.

Dermatofitosis o Tiñas2, 5, 8 Se clasifican topográficamente. Según su distribución céfalo-caudal son: - Tiña barbae: relacionada con el contacto con animales, puede dejar alopecia cicatricial. - Tiña capitis: es la tiña más frecuente en la infancia. Se subdivide en3, 5, 9: (Tabla 3) - Tiña corporis: junto con la tiña ungium, es la dermatofitosis más frecuente en nuestro país. Las lesiones crecen típicamente de manera centrífuga y tienen un aspecto anular. Existe una forma profunda granulomatosa folicular denominada “granuloma de Majocchi”, que aparece en piernas de mujeres relacionada con la depilación (Fig.2). - Tiña de la mano: es generalmente unilateral y conviene descartar tiña pedis contralateral (autoinoculación por rascado). - Tiña cruris: afecta a los pliegues corporales, siendo especialmente frecuente en los inguinales y raíz proximal de muslos. Aparece más en pacientes varones, jóvenes, deportistas, con exceso de sudoración y utilización de ropa ajustada. - Tiña pedis: está descrita como la dermatofitosis más frecuente en la población general, pero muchas series no la recogen como tal, probablemente debido a que está infradiagnosticada y responde muy bien a tratamiento tópico (rápida resolución, sin necesidad de pruebas microbiológicas de confirmación). Existen varias formas: cróni-

Levaduras Son organismos que normalmente se comportan como saprófitos pero, en su forma filamentosa, son patógenos. Destacan el género MALASSEZIA spp. (antiguamente denominada Pityrosporumspp.), según algunos autores la causante de la micosis superficial más frecuente8, y el género CÁNDIDA spp. considerada dentro del grupo de las micosis 26


TABLA 2. Hongo más frecuentemente causante de patología según la clínica1, 3

TABLA 3 ca intertriginosa (característicamente en el 4º espacio interdigital), crónica hiperqueratósica (forma en mocasín, afectando a planta del pie y bordes laterales) y aguda dishidrosiforme (típicamente unilateral). - Tiña ungium: la forma subungueal distal es la más frecuente presentando hiperqueratosis subun-

gueal con o sin onicolisis. La subungueal proximal es poco frecuente y algunas series la asocian a infección por VIH por lo que deben solicitarse serologías. La leuconiquiadermatofítica u onimicosis blanca superficial se produce por afectación de la tabla externa dando un aspecto en tiza. La distrófica total se considera la evolución de cualquiera de 27


22º Curso de Dermatopatología

BIBLIOGRAFÍA (1) ColominaRodr1íuez J. et al. Análisis etiológico de las micosis cutáneas superficiales por dermatofitos en la Comunidad Valenciana (20082013). Piel(barc).2015;30(6): 393-402. (2) Elewski B.E., Lauren C. Hughey L.C. et al. Fungaldiseases. Infections, infestations and bites. Section 12. Dermatology (Jean L. Bolognia). Elsevier. 2018. (3) Molina de Diego A. Aspectos clínicos, diagnósticos y terapéuticos de las dermatofitosis. EnfermInfeccMicrobiolClin. 2011;29(Supl 3):33-39. (4) Buitrago MJ., Cuenca-Estrella M. Epidemiología actual y diagnóstico de laboratorio de las micosis endémicas en España. EnfermInfeccMicrobiolClin. 2012;30(7):407–413 (5) Ferrándiz C. et al. Micosis cutáneas en la edad pediátrica. MedClin (Barc). 2006;126(Supl 1):37-42. (6) Escutia B., Febrer et. al. Tineacapitis por Microsporumaudouinii. RevIberoamMicol 2001; 18: 88-90. (7) García-Matos P., et al,. Dermatofitosis por Microsporumgypseum: Descripcion de ocho casosyrevision de la literatura. RevIberoamMicol 2004; 21: 147-149. (8) Delgado Florencio V., Crespo Erchiga V. Micosis cutáneas. Medicina Clínica. 2005; Vol 125. Num 12: 4805–4815. (9) P. Kelly B. Superficial FungalInfections. Pediatrics in Review Vol.33 No.4 April 2012. (10) Romero Navarrete M. et al. Tiña negra. Revisión de la literatura internacional y énfasis de casos publicados en México. Dermatología CMQ2012; 10(3): 205-211.

las anteriores. Los dermatofitos son la causa más frecuente de onicomicosis (Fig.3), aunque debe recordarse que tanto los mohos como los hongos del género Candidaspp pueden producir onicomicosis. - Tiña incognito: hace referencia a una dermatofitosis tratada con inmunosupresores tópicos (corticoides, tacrolimus…) lo que modifica su aspecto, con una menor expresión clínica por desaparición del componente inflamatorio. Pitiriasis versicolor5, 8 Es una infección asintomática que afecta fundamentalmente a adolescentes y adultos jóvenes. Está producida por diversas especias del género Malasseziaspp., siendo M. globosa la especie más frecuentemente causante de la patología en España. Son hongos lipofílicos que en su forma colonizadora (flora normal de la piel) se encuentran en forma de levadura mientras que, bajo condiciones determinadas (calor, humedad, inmunosupresión…), adoptan su forma patógena filamentosa. La clínica consiste en máculas anaranjadas-pardas (generalmente hiperpigmentadas en pieles claras e hipopigmentadas en pieles oscuras), con escamas finas desprendibles con el rascado (signo de la uñada). Cuando curan pueden dejar una hipopigmentación residual de meses de duración. Ésta se debe a que el hongo posee un enzima dopa-tirosinasa capaz de inhibir temporalmente la síntesis de melanina. Tiña Negra10 Es una micosis superficial de distribución mundial aunque característica de áreas tropicales. Causada por Hortaeawerneckii que se contagia por contacto con un medio acuoso salino.Se manifiesta como máculas marrón hiperpigmentadasoscuras o negras, bien delimitadas, no inflamatorias y asintomáticas, que afectan predominantemente a las palmas de las manos, aunque pueden aparecer en otras localizaciones. Responde bien a tratamiento tópico.

28


Casos clĂ­nicopatolĂłgicos

29


22Âş Curso de DermatopatologĂ­a

30


Caso clínico-patológico 1 Dras. M. Fernández Guarino y R. Carrillo. Hospital Ramón y Cajal (Madrid).

Figura 1

Historia Clínica Varón de 79 años con antecedentes personales de infarto de miocardio, hipertensión arterial y diabetes mellitus de dos años de evolución, en tratamiento con ácido acetilsalicílico, acenocumarol, Valsartán (ARA II) y Eucreas (metformina y vildagliptina). Consulta por lesiones muy pruriginosas de 4 meses de evolución de inicio lento y progresivo, aunque con empeoramiento franco en los dos últimos meses, afectando a su calidad de vida. Analítica (hemograma y bioquímica) sin alteraciones. A la exploración física se objetivan ampollas tensas pequeñas y costras melicéricas, sobre placas eritemato-edematosas, distribuidas de manera difusa. (Fig. 1)

la ampolla presenta un discreto infiltrado linfocitario perivascular con aislados eosinófilos. Con las técnicas de inmunofluorescencia directa se advierten depósitos lineales basales de IgG y C3. Diagnóstico PENFIGOIDE AMPOLLOSO SECUNDARIO A GLIPTINA. Comentarios El penfigoide ampolloso es una enfermedad autoimmune adquirida caracterizada por ampollas subepidérmicas en la que la causa subyacente es una respuesta humoral y cellular contra los antígenos BP180 y BP230. Los inhibidores de la Dipeptidil-peptidadsa-4 utilizados como fármacos antidiabéticos orales recientemente se han reconocido como posibles agentes productores de un síndrome de tipo penfigoide ampolloso. Presentamos un caso en el que la histología además del despegamiento dermo-epidérmico característico del penfigoide, presentaba algunos queratinocitos necróticos, lo que favorecía desde el pun-

Descripción Microscópica La biopsia demuestra una ampolla subepidérmica. En el techo de la ampolla se observa una epidermis ligeramente acantótica sin trastornos de la queratinización. La capa basal demuestra un moderado número de queratinocitos necróticos salpicados entre queratinocitos conservados. El suelo de 31


22º Curso de Dermatopatología

Figura 2

BIBLIOGRAFÍA 1. Béné J, Moulis G, Bennani I, Auffret M, Coupe P, Babai S, Hillaire-Buys D, Micallef J, Gautier S. Bullous pemphigoid and dipeptidyl peptidase IV inhibitors: a case-noncase study in the French Pharmacovigilance Database. Br J Dermatol. 2016 Aug; 175(2):296-301. 2. Aouidad I, Fite C, Marinho E, Deschamps L, Crickx B, Descamps V. A case report of bullous pemphigoid induced by dipeptidyl peptidase-4 inhibitors. JAMA Dermatol. 2013 Feb;149(2):243-5.

to de vista histológico un origen medicamentoso.

Figura 3 32


Caso clínico-patológico 2 Dras. M. Fernández Guarino y R. Carrillo. Hospital Ramón y Cajal (Madrid).

Figura 1

Historia Clínica Mujer de 58 años con antecedentes de hipotiroidismo en tratamiento con Eutirox. Acude a consulta de Dermatología por una lesión cutánea en cara externa de pie izquierdo de 7 años de evolución, que fue creciendo progresivamente hasta mantenerse estable hace 2 años. La paciente niega herida o traumatismo previo en la zona. La analítica reciente no presenta alteraciones, salvo una TSH ligeramente elevada. A la exploración física se observa una placa sobre-elevada de contorno y superficie irregulares, pardusca, asintomática, de 4 cm de diámetro, de consistencia elástica. La lesión fue biopsiada en 2013 y en 2018 con diagnóstico histopatológico de cicatriz hipertrófica

Figura 2

Descripción Microscópica Piel acral con ligera hiperqueratosis ortoque33


22º Curso de Dermatopatología

Figuras 3 y 4

BIBLIOGRAFÍA 1. Nakazato S1, Nomura T1, Yamane N1, Shinkuma S1, Natsuga K1, Fujita Y1, Arita K1, Shimizu H1. Nodular morphoea: a first case associated with linear morphoea. Eur J Dermatol. 2016;26(1):95-6. 2. Ohata C1, Yasunaga M, Tsuruta D, Ishii N, Hamada T, Dainichi T, Karashima T, Nakama T, Hashimoto T. Nodular morphea (NM): report of a case of concurrent NM and morphea profunda associated with limited type systemic sclerosis, and overview and definition for NM. Eur J Dermatol. 2013;23(1):87-93 3. Kauer F, Simon JC, Sticherling M. Nodular morphea. Dermatology. 2009;218(1):63-6. 4. Rencic A, Brinster N, Nousari CH. Keloid morphea and nodular scleroderma: two distinct clinical variants of scleroderma? J Cutan Med Surg. 2003;7(1):20-4.

ratótica. La dermis papilar y reticular está ocupada por proliferación de fibroblastos y colágeno dispuestos paralelos a la epidermis y vasos perpendiculares a la misma. La lesión destruye todos los anejos cutáneos que no se reconocen en esta área. No se observan haces de colágeno de aspecto queloideo ni infiltrado inflamatorio. La imagen histológica es indistinguible de una reacción cicatricial. Diagnóstico MORFEA NODULAR Comentarios La esclerodermia se puede presentar de forma estrictamente limitada a la piel como en la morfea o en la forma de esclerosis sistémica. En la morfea nodular o queloidea los pacientes desarrollan lesiones que clínica e histológicamente pueden simular una cicatriz. En el caso que presentamos la clínica era inespecífica pero el aspecto histológico de ambas biopsias era indistinguible de una cicatriz hipertrófica. Presentamos esta forma inusual de la morfea y discutimos las dificultades diagnósticas y la importancia de la correlación clínico-patológica para llegar a un diagnóstico correcto definitivo.

34


Caso clínico-patológico 3 Drs. G. Roustan y L. Nájera Hospital Puerta de Hierro (Majadahonda - Madrid). Historia Clínica Varón de 65 años con historia de fiebre mediterránea familiar que presenta fiebre, abscesos cutáneos y subcutáneos múltiples y recurrentes, linfadenopatía cervical, pápulas y placas eritemato-edematosas generalizadas, policondritis recidivante y anemia y trombopenia. Evolución Las serologías para VHB, VHC, VIH, citomegalovirus, sífilis, toxoplasma, parvovirus, coxiella, bartonella, brucella y Borrelia burgdorferi fueron negativas, así como los repetidos cultivos microbiológicos de sangre, médula ósea, orina, esputo y ganglio linfático, y la PCR en médula ósea para Tropheryma whipplei y Mycobacterium tuberculosis. Los estudios inmunológicos de ANA, ANCA, crioglobulinemia y proteinograma fueron negativos o normales. El PET-TC mostró linfadenopatías cervicales sugestivas de proceso linfoproliferativo, abscesos subcorticales en ambos riñones y fracturas en quinta y sexta costillas. Descripción Microscópica El estudio histopatológico del ganglio cervical evidenció una linfadenitis abscesificada necrotizante; todas las biopsias cutáneas se diagnosticaron de dermatosis neutrofílica/síndrome de Sweet, y las tres biopsias de médula ósea realizadas mostraron cambios reactivos, hiperplasia de la serie granulocítica y rasgos atípicos en la serie megacariocítica en la última.

Figura 1

Diagnóstico DERMATOSIS NEUTROFÍLICA, POLICONDRITIS RECIDIVANTE Y ENFERMEDAD DE KIKUCHI. Comentarios El síndrome de Sweet es una dermatosis neutrofílica, que hasta en un 20% de los casos se asocia a neoplasias malignas hematológicas. Cuando se desarrolla un síndrome mielodisplásico, puede ser clínica e histopatológicamente atípico. La asociación de síndrome de Sweet y poli-

Figura 2 35


22º Curso de Dermatopatología

Figura 3

condritis recidivante es muy poco frecuente y se ha descrito en la literatura como factor predictivo de mielodisplasia, con presencia de alteraciones cutáneas entre de 2 a 8 años antes de que se desarrolle el proceso hematológico. Por lo tanto, el proceso cutáneo y policondrítico puede ayudar a diagnosticar precozmente la mielodisplasia en estos pacientes y a su manejo terapéutico.

BIBLIOGRAFÍA 1. Vignon-Pennamen MD, Juillard C, Rybojad M, Wallach D, Daniel MT, Morel P, Verola O, Janin A. Chronic recurrent lymphocytic Sweet syndrome as a predictive marker of myelodysplasia: a report of 9 cases. Arch Dermatol. 2006;142:11706. 2. Diamantino Fda E, Raimundo PM, Fidalgo AI. Sweet’s Syndrome and relapsing polychondritis signal myelodysplastic syndrome. An Bras Dermatol. 2011;86 (4 Suppl 1):S173-7. 36


Caso clínico-patológico 4 Drs. P eg ori, P P.. González Muñoz, V. Tarín Gr Greg egori, P.. Cobo Rodríguez, I. Eraña Tomás. Hospital Universitario de Guadalajara y Hospital Fundación Jiménez Díaz (Madrid)

Figura 1

Historia Clínica Mujer de 51 años con antecedentes personales de fibromialgia, asma y depresión en tratamiento con múltiples fármacos que acude a Urgencias por un cuadro de una semana de evolución consistente en fiebre de hasta 39ºC, odinofagia, tos productiva y dolor abdominal tipo cólico con diarrea sin productos patológicos a los que se había asociado hacía unos días lesiones cutáneas pruriginosas. A la exploración se evidenciaba edema facial y un rash morbiliforme, máculo-pápuloso confluente que afectaban a > 80% de la superficie corporal adquiriendo una tonalidad violácea en extremidades inferiores. Además, destacaban la presencia de adenopatías laterocervicales de pequeño tamaño y rodaderas siendo el resto de la exploración física

normal. En la analítica sanguínea se observó una leucocitosis con eosinofilia, alteración de perfil hepático y elevación de PCR y VSG. En la anamnesis destacaba la introducción 1 mes antes por parte de psiquiatría de lamotrigina para control de su cuadro depresivo. Se procedió al ingreso de la paciente para completar el estudio y con la sospecha de síndrome de Dress se inició tratamiento con corticoides orales a dosis de 1 mg/kg día. Los resultados de las pruebas complementarias solicitadas descartaron un origen infeccioso del cuadro. Las pruebas de autoinmunidad y la radiografía simple de tórax también fueron normales. Descripción Microscópica Se realizan dos biopsias cutáneas por punch ob37


22º Curso de Dermatopatología

servándose en una de ellas marcado edema en dermis papilar que conlleva la separación de la epidermis, por debajo del cual se observa un denso infiltrado polimorfo de predominio linfoide dispuesto en banda y con tendencia rodear estructuras vasculares. Tras el estudio inmunohistoquímico se puede observar que el infiltrado está constituido en su mayor parte por linfocitos T los cuales muestran atipia citológica significativa, con predominio de los T CD4 sobre los CD8. La expresión de CD30 es débil y focal. Dados estos hallazgos histológicos y ante la sospecha de un síndrome linfoproliferativo T, se realiza un inmunofenotipo de sangre periférica que no mostró alteraciones y un TAC body que no evidenciaba adenopatías. Además, la evolución clínica de la paciente tras la instauración del tratamiento ha sido excelente con resolución completa de las lesiones y de las alteraciones analíticas que presentaba. Ante estos hallazgos se llega a la conclusión de que estamos ante un infiltrado inflamatorio atípico de naturaleza reactiva (reacción linfomatoide) en relación a fármacos.

Figura 2

Figura 3

Diagnóstico SÍNDROME DE DRESS Comentarios Existen múltiples situaciones descritas en la literatura que pueden simular tanto clínica como histológicamente un linfoma T cutáneo. Esto puede ocurrir por ejemplo ante vacunas, tatuajes, infecciones y reacciones a drogas, entre otras causas. De ahí que sea imprescindible una adecuada correlación clínico-patológica para realizar un correcto diagnóstico.

Figura 4 38


BIBLIOGRAFÍA 1. Jacobsen E; Tarabadkar ES; Shinohara MM. Generalized fixed drug eruption mimicking CD8+ cutaneous T-cell lymphoma in HIV. J. Cutan. Pathol 2019; 46:134-137 2. Tashiro Y; Azukizawa H, Asada H, et al. Druginduced hypersensitivity syndrome/drug reaction with eosinophilia and systemic symptoms due to lamotrigine differs from that due to other drugs. J Dermatol. 2019 Jan 21. doi: 10.1111/13468138.14776. 3. Magro CM, Crowson AN, Kovatich J, et al. Drug-Induced Reversible Lymphoid Dyscrasia: A Clonal Lymphomatoid Dermatitis of Memory and Activated T Cells. Human Pathol. 2003. Vol34. No2. doi:10.1053/hupa.2003.4. 4. Yi-Chin Chen, and Yu-Hung Wu. Linear Folliculotropic CD30-Positive Lymphomatoid Drug Reaction .. Am J Dermatopathol 2017;39. No5:e62–e65. 5. Magro CM, Olson LC, Nguyen GH, et al. CD30 Positive Lymphomatoid Angiocentric Drug

Reactions: Characterization of a Series of 20 Cases. Am J Dermatopathol 2017;39:508–517 . 6. Greil W, Zhang X, Stassen H, et al. Cutaneous adverse drug reactions to psychotropic drugs and their risk factors – a case-control study. European Neuropsychopharmacology 2018; 000, 1– 11. 7. Sawada Y, Toshiki R, Kawakami C, et al. Valsartan-induced drug eruption followed by CD30+ pseudolymphomatous eruption. Acta Derm Venereol. 2010; 90:521-522. 8. Marucci G, Sgarbanti E, Maestri A, et al. Gemcitabine-associated CD8+ CD30+ pseudolymphoma. Br J Dermatol 2001; 145: 650-652. 9. Saeed SA, Bazza M, Zaman M, et al. Cefuroxime induced lymphomatoid hypersensitivity reaction. Postgrad Med J. 2009; 19:292-294 10. Nathan DL, Belsito DV. Carbamazepineinduced pseudolymphoma with CD30 positive cells. J Am Acad Dermatol 1998; 38 (5 pt 2): 806809.

39


22Âş Curso de DermatopatologĂ­a

40


Caso clínico-patológico 5 Drs. M. Feito y M. Beato. Hospital La Paz (Madrid)

Figura 1

Historia Clínica Un niño varón nació con una lesión pigmentada discrómica en la región lumbar, de varios centímetros de diámetro (Fig. 1). A la exploración física, se objetivaba un nevo melanocítico congénito con patrón spilus, consistente en varias máculas y manchas de color marrón claro, marrón oscuro y eritematosas, que oscilaban entre 2-4 mm (las de menor tamaño) y 1-3 cm de diámetro (las de mayor tamaño). Una de ellas, situada en el sector superior, era ligeramente palpable. A los diez días de vida, se hizo una biopsia de una de las manchas del nevo y se extirpó la lesión palpable. Tres años después, se objetivó una tumoración inguinal izquierda de 2 cm de diámetro, y dos años después de esto, otra de 1 cm de diámetro, de coloración azulada, en la axila izquierda.

Descripción Microscópica Al microscopio, el nevo era un nevus melanocítico congénito convencional, con melanocitos situados en la epidermis y por toda la dermis, en esta última uniformes y pequeños, aislados entre los haces de colágeno (Fig. 2). La lesión palpable era distinta: En ella, se observaban melanocitos grandes y de citoplasma claro; estos estaban situados por todos los estratos de la epidermis y, en la dermis formaban nidos confluentes y sábanas, no maduraban, y mostraban figuras de mitosis (Figs. 3, 4 y 5). Evolución Tres años después, se objetivó una tumoración inguinal izquierda de 2 cm de diámetro, desplazable y no adherida a planos profundos, con ecografía sugestiva de malignidad. La lesión fue extirpa41


22º Curso de Dermatopatología

da, y la biopsia mostró grandes nidos confluentes de melanocitos atípicos. Las pruebas de imagen no demostraron otras lesiones a distancia. Tras otros dos años asintomático, el paciente desarrolló otra tumoración de 1 cm de diámetro, de coloración azulada, bien delimitada pero en este caso adherida a planos profundos, en la axila izquierda (Fig. 6). El estudio histopatológico también demostró nódulos compuestos por sábanas de melanoticos atípicos.

Figura 2

Diagnóstico: MELANOMA CONGÉNITO SOBRE NEVUS MELANOCÍTICO DE PATRÓN SPILUS CON METÁSTASIS GANGLIONARES POSTERIORES EN LA INGLE Y AXILA IZQUIERDAS. Discusión Los nevos melanocíticos congénitos pueden manifestarse al nacimiento de múltiples formas, desde pequeñas máculas solitarias a grandes áreas de pigmento con o sin lesiones satélites asociadas, o lesiones de gran tamaño tipo spilus. Aunque la mayoría de los nevos melanocíticos congénitos no varían significativamente, algunos sí muestran cambios en el color o desarrollan nódulos en un corto periodo de tiempo, haciendo que los hallazgos clínicos e histológicos sean difíciles de interpretar. Por otro lado, está bien descrita la posibilidad de malignización de los nevos melanocíticos congénitos de gran tamaño, no solo a nivel cutáneo sino también a nivel del sistema nervioso central. Es pronóstico suele ser malo en estos pacientes, aunque en los últimos años se han hecho avances interesantes desde el punto de vista molecular, y empiezan a utilizarse fármacos empleados en la población adulta como los inhibidores de BRAF y MEK e inmunoterapia.

Figura 3

Figura 4

Bibliografía: 1. Carrera C, Scope A, Dusza SW, et al. Clinical and dermoscopic characterization of pediatric and adolescent melanomas: Multicenter study of 52 cases. J Am Acad Dermatol 2018; 78(2): 278-88. 2. Stefanaki C, Chardalias L, Soura E, et al. Paediatric melanoma. J Eur Acad Dermatol Venereol 2017; 31: 1604-15.

Figura 5 42


Figura 6 43


22Âş Curso de DermatopatologĂ­a

44


Caso clínico-patológico 6 Drs. V. Alegría, C C.. Santonja y L. Requena. Hospital Fundación Jiménez Díaz (Madrid)

Figura 1

Historia Clínica Mujer de 42 años que consultó en el servicio de dermatología refiriendo crecimiento de una lesión en cuero cabelludo presente aparentemente desde la infancia. En la exploración física se observó una placa azulada de 7 x 10 cm aproximadamente localizada en la región tempero-occipital. La lesión mostraba una superficie con un llamativo aspecto cerebriforme compuesta por pliegues y circunvoluciones.

do de la muestra tumoral se investigó el estado mutacional de regiones específicas de 161 genes encontrándose NTRK2 c.1010 C> G (p.Thr337Ser) y NRAS c.182 A>G ( p.Gln61Arg). Diagnóstico MELANOCITOSIS DÉRMICA FOCAL PRESENTANDOSE COMO CUTIS VERTICIS GYRATA. Comentarios Las melanocitosis dérmicas son lesiones melanocíticas peculiares caracterizadas por una coloración azulada de la piel, siendo las diferentes entidades designadas según las áreas del cuerpo a las que afectan. Histológicamente este tipo de dermatosis se caracteriza por la presencia de melanocitos dérmicos inmersos en fibras de colágeno que no muestras alteraciones.

Descripción Microscópica El estudio histopatológico reveló la presencia, bajo una epidermis normal, de melanocitos dendríticos alargados y pigmentados dispersos como unidades individuales en todo el espesor de la dermis entre fibras de colageno sin alteraciones y en ausencia de melanófagos acompañantes. Mediante secuenciación masiva del material genético obteni45


22º Curso de Dermatopatología

Figura 2

Figura 3

Figura 5

Figura 4

Clásicamente se ha considerado a este grupo de lesiones como variantes del nevus azul estando aparentemente vinculadas ambas entidades nosológicas por mutaciones iniciadoras comunes que involucran a los genes GNAC y GNA11 aunque en realidad son muy escasas las publicaciones que recogen el estado mutacional de las melanocitosis dérmicas. Por otro lado, el cutis verticis gyrata no es una entidad en sí misma, sino un término descriptivo que hace referencia a la presencia de pliegues y surcos que se asemejan a las circunvoluciones de la corteza cerebral. Discutimos la excepcional ocurrencia de una melanocitosis dérmica focal que se presenta clínicamente como un cutis verticis gyrata y que alberga una mutación en un gen cuyas fusiones se han relacionado con el melanoma primario y metastásico; revisamos la relación existente entre las melanocitosis dérmicas, el nevus azul y el nevus melanocítico congénito y proponemos una clasificación práctica para los cutis verticis gyrata.

3. Gerami P, Pouryazdanparast P, Vemula S, Bastian BC. Molecular analysis of a case of nevus of Ota showing progressive evolution to melanoma with intermediate stages resembling cellular blue nevus. Am J Dermatopathol. 2010;32:301305. 4. Roh MR, Eliades P, Gupta S, Tsao H. Genetics of melanocytic nevi. Pigment Cell Melanoma Res. 2015 Nov;28:661-672. 5. Lezcano C, Shoushtari AN, Ariyan C, Hollmann TJ, Busam KJ. Primary and Metastatic Melanoma With NTRK Fusions. Am J Surg Pathol. 2018;42:1052-1058.

BIBLIOGRAFÍA 2. Hori Y, Takayama O. Circumscribed dermal melanoses: classification and histologic features. Dermatol Clinic. 1988;6:315-326. 46


Caso clínico-patológico 7 Drs. M. Rodríguez Pinilla y L. Requena Hospital Fundación Jiménez Díaz (Madrid) Historia Clínica Mujer de 62 diagnosticada en 2016 (hace dos años) de Síndrome de Sezary. Debutó con eritrodermia generalizada asociada a linfocitosis absoluta y aparición de adenopatías periféricas. En la primera consulta acude con placas eritematodescamativas que abarcan > 80% BSA, placas redondeadas en las piernas. Plantas de los pies descamativas, agrietadas. Uñas hiperquereatósicas amarillentas en los pies. En las manos, sobre todo, afectación distal. Cuero cabelludo descamativo en casco. Tras diferentes líneas de tratamiento incluyendo fotoférexis extracorpórea en el R y C la enfermedad cursa en brotes de empeoramiento y mejoría clínica, con intenso prurito, eritrodermia y dolor en la piel. El 16 de febrero de 2018 se realiza transplante alogénico haploidéntico de intensidad reducida con protocolo de desensibilización previa por presencia de anticuerpos antiHLA. No tiene datos de EICH. En Mayo se suspende la inmunosupresión para favorecer enfermedad injerto contra linfoma y recibirá infusión de linfocitos de donante. Aparición de nuevas lesiones, ahora, eritematopapulosas en cara y espalda que no representan más del 10% de la SC, sobre lesiones de SS preexistentes. Se realiza profilaxis con tacrólimus ante sospecha de EICH grado I, que es posteriormente confirmado con biopsia cutánea. Reconstitucion autologa con perdida quimerismo y recaida del Sd Sezary (dia +86). Las lesiones cutáneas empeoran. Aparecen placas de alopecia, con lesiones liquenoides en cuero cabelludo en relación con EICH. Se decide inclusión del paciente en el ensayo

Figuras 1 y 2

47


22º Curso de Dermatopatología

Figura 3

Diagnóstico: PIEL CON LESIONES DE SEZARY, EICH Y TOXICODERMIA.

clinico BET116183 Fase I/II con GSK525762 (inhibidor de BET). Tras varias semanas con el inhibidor de BET se observa la aparición de nuevas lesiones ulceradas y erosionadas que se biopsian. En la biopsia se observan cambios histológicos focales sugestivos de reacción psoriasiforme, datos de EICH y presencia de células de S. Se retira el fármaco.

Comentarios Em este caso se han identificado en la evolución del paciente lesiones tanto de Sezary, como de EICH y de toxicodermia. Estas três lesiones presentan imágenses histológicas similares que em la mayoría de los casos son difíciles de diferenciar. Se presentan cada uma de estas lesiones y se comentan diferentes claves que permitan distinguirlas entre sí.

Descripción Microscópica Piel que presenta un infiltrado linfocitario que borra la interfase dermoepidérermica y se extiende a la demis papilar, especialmente alrededor de los vasos capilares superficiales. En la epidermis suprayacente se reconocen queratinocitos nectóticos de distribución focal en distintos estratos superficales y profundos epidérmicos.

Figura 4 48


Caso clínico-patológico 8 Drs. C ostig o y J.L. Rodríguez P eralto C.. P Postig ostigo Peralto eralto.. Hospital 12 de Octubre (Madrid).

Figura 1

Historia Clínica Varón de 49 años con antecedentes de dermatitis atópica en la infancia. Acude a dermatología por brotes de lesiones maculopapulosas eritematosas que se hacen confluentes en la cara anterior del tronco con ligerísima descamación y lesiones simétricas en palmas y plantas de 2 años de evolución. Inicalmente fueron interpretadas como exantema viral por Micoplasma. Descripción Microscópica Piel que presenta un denso infiltrado linfocitario que rellena la dermis papilar y reticular alta con epidermotropismo y disposición en fila india en la capa basalñ de la epidermis de forma focal. Las células son de mediano o gran tamaño, irregulares e hindentadas. El perfil inmunohistoquímico es T con expresión de CD3, CD43, CD4 y negatividad para CD8. Además, expresan de forma difusa CD30 con distribución membranosa y paranuclear.

Figura 2

Diagnóstico PAPULOSIS LINFOMATOIDE TIPO B Comentarios El presente caso es un ejemplo de la dificultad

Figura 3 49


22Âş Curso de DermatopatologĂ­a

Figuras 4 y 5

50


que a veces se plantea a la hora de diferenciar entre papulosis linfomatoide tipo B y micosis fungoide papulosa. Esto es un problema de gran trascendencia, puesto que la papulosis linfomatoide es un linfoma T de muy buen pronóstico en el que las pápulas aparecen y desaparecen solas, mientras que la micosis fungoide, aunque sea papular, y este caso es demostrativo, puede ser interpretada como un linfoma que se está trasformando a células grandes y por tanto de mal pronóstico. No obstante, algunos autores sostienen que la Papulosis linfomatoide B puede progresar a un linfoma B agresivo, por lo que es necesario un seguimiento estrecho. La variante papular de la Micosis fungoide fue definida por Kotama y cols por presentar 3 criterios: 1.- Pápulas con datos histológicos de micosis fungoide; 2.- Ausencia de regresión espontanea de las lesiones y 3.- Ausencia de otras evidencias de micosis fungoide o de reacciones medicamentosas linfomatoides. Sin embargo, en la descripción original de Kotama se describen dos casos que posteriormente desarrollaron placas típicas de micosis fungoide. Por otro lado, no es fácil interpretar la regresión espontanea de las lesiones de papulosis linfomatoide, ya que algunas de ellas tardan bastante tiempo en regresar porque lo hacen lentamente y en muchas ocasiones, debido a que los enfermos están sometidos a tratamientos corticosteroideos u otros, no es fácil definir si la regresión es espontanea o inducida por el tratamiento. De cualquier forma, la única forma de hacer un diagnóstico adecuado de Papulosis linfomatoide tipo B es realizar una adecuada correlación clínico-patológica en la que se va a demostrar que las lesiones son típicas microscópicamente de miocosis fungoide, pero sin embargo clínicamente presentan pápula múltiples, que desaparecen de forma espontánea.

3. Vonderheid EC, Kadin ME, Telang GH. Commentary about papular mycosis fungoides, lymphomatoid papulosis and lymphomatoid pityriasis lichenoides: more similarities than differences. J Cutan Pathol. 2016 Apr;43(4):303-12 4. Kim SJ, Schaffer A, Lee HY, Seo JY, Park HJ. A Case of Papular Mycosis Fungoides: A New Clinical Variant of Early Mycosis Fungoides. Ann Dermatol. 2016 Jun;28(3):383-4 5. Noe MH, Drake A, Link BK, Liu V. Papular mycosis fungoides: report of two patients, literature review, and conceptual re-appraisal. J Cutan Pathol. 2013 Aug;40(8):714-9 6. Martorell-Calatayud A, Botella-Estrada R, Sanmartín-Jimenez O, Requena C, Guillén-Barona C, Sangüeza OP. Papular mycosis fungoides: two new cases of a recently described clinicopathological variant of early mycosis fungoides. J Cutan Pathol 2010; 37: 330

BIBLIOGRAFÍA 1. Bouhamidi A, Hassam B. Type B lymphomatoid papulosis. Pan Afr Med J. 2018 Jun 18;30:138. 2. Wieser I, Oh CW, Talpur R, Duvic M. Lymphomatoid papulosis: Treatment response and associated lymphomas in a study of 180 patients. J Am Acad Dermatol. 2016 Jan;74(1):59-67 51


22Âş Curso de DermatopatologĂ­a

52


Caso clínico-patológico 9 Drs. P tiz, R. Ara gón y J.L. Rodríguez P eralto P.. Or Ortiz, Aragón Peralto Hospital 12 de Octubre (Madrid).

Figura 1

Historia Clínica Mujer de 80 años con carcinoma mama tratado con radioterapia, miocardiopatía dilatada tóxica (alcohólica, radioterápica) y enfermedad de Menière. Consulta por astenia, hiporexia, bradipsiquia y úlcera lingual asintomática de 2 meses de evolución

Evolución A la vista de que la lesión persistía y no había un diagnóstico específico, se decidieron explorar otras posibilidades y entre ellas, se planteó la posibilidad de que fuera una úlcera isquémica. Se realizaron pruebas angiográficas y radiológicas, donde se evidenció una posible lesión oclusiva vascular en arteria temporal, que se biopsió. La biopsia de la arteria temporal demostró una disminución de la luz y un engrosamiento de la pared de la arteria que estaba sustituida por fibrosis e infiltrado inflamatorio con abundantes células gigantes multinucleadas fagocitando las fibras elásticas de la íntima.

Descripción Microscópica En la biopsia de la úlcera lingual, sólo se observó una solución de continuidad de la mucosa que estaba sustituida por necrosis, fibrina y detritus celulares de neutrófilos y linfocitos. No se identificó una neoformación epitelial, mesenquimal o linfoide. Las técnicas de PAS, Grocott y Ziehl resultaron negativas.

Diagnóstico: ULCERA LINGUAL SECUNDARIA A ARTERITIS DE CÉLULAS GIGANTES DE LA ARTERIA TEMPORAL 53


22º Curso de Dermatopatología

Figuras 2 y 3

minada también pueden producir isquemia y úlceras linguales. Además de en procesos isquémicos sistémicos como infarto de miocardio, se ha descrito necrosis lingual en lupus eritematoso, fenómeno de Raynaud, síndrome antifosfolípido y en calcifilaxias asociadas a insuficiencia renal. El diagnóstico definitivo es identificar la causa. En nuestro caso se llevó a cabo por una biopsia de la arteria temporal, donde se identificó un proceso inflamatorio crónico con células gigantes multinucleadas fagocitando las fibras de las elásticas internas de la arteria. Tras el tratamiento con Corticoides intravenosos: Metilprednisolona 125 mg durante 3 días consecutivos y Corticoides orales: Prednisona 20 mg/día, desapareció la úlcera completamente, quedando una cicatriz lingual que no produjo ningún déficit posterior.

Comentarios La úlcera lingual con necrosis y pérdida de sustancia es un evento muy raro de etiología diversa: a) traumática, b) Infecciosa: fúngica (histoplasmosis, blastomicosis), bacteriana (sífilis terciaria) o viral (herpes tipo virus de Epstein Barr y citomegalovirus), asociadas HIV o inmunodeficiencias. c).- Tumoral por neoplasias primarias malignas como el carcinoma escamoso o metastásicas responsables del 1% de los tumores malignos de la cavidad oral, de los cuales un cuarto infiltran la lengua; d) Procesos isquémicos relacionados con vasculitis, enfermedades cardiovasculares o cerebrovasculares y otras enfermedades sistémicas. Entre estas causas de isquemia destaca la Arteritis de células gigantes, también conocida como arteritis de la temporal y craneal. Nuestro caso, precisamente se trata de este proceso, arteritis de la temporal, una vasculitis crónica de arteria de mediano y gran tamaño con componente inflamatorio crónico localizado y sistémico. Esta enfermedad tiene una incidencia de 10 a 70 por 100,000 individuos anualmente. Ocurre exclusivamente en pacientes mayores de 50 años, (media 70 a.) con predominio por las mujeres (3 veces más frecuentes que los hombres). Los pacientes sin tratamiento adecuado de la arteritis de células gigantes pueden padecer complicaciones isquémicas importantes, como ceguera repentina, neuropatía óptica, ictus, aneurisma de aorta, infarto intestinal, insuficiencia renal o infarto de miocardio. Se han descrito otras causas de isquemia que pueden producir úlceras necróticas linguales como la ligadura de la carótida externa tras disección radical de cuello con radioterapia o como complicación de embolización arterial por quimioterapia o radioterapia secundaria a tratamiento por cáncer. Enfermedades hematológicas como la trombocitosis esencial o la coagulación intravascular dise-

BIBLIOGRAFÍA 1. Córdoba S, Martínez D, Martín L, Borbujo JM. Tongue ulcer as a manifestation of temporal arteritis. Actas Dermosifiliogr. 2009 Apr; 100(3):237-8. 2. Stone JH, Papaliodis GN, Dunbar MR, Stone JR. Case records of the Massachusetts General Hospital. Case 4-2010. A 53-year-old man with arthralgias, oral ulcers, vision loss, and vocal-cord paralysis. N Engl J Med. 2010 Feb 11;362(6):53746 3. Zadik Y, Findler M, Maly A, Rushinek H, Czerninski R. A 78-year-old woman with bilateral tongue necrosis. Oral Surg Oral Med Oral Pathol Oral Radiol Endod. 2011 Jan;111(1):15-9 4. Grant SW, Underhill HC, Atkin P. Giant cell arteritis affecting the tongue: a case report and review of the literature. Dent Update. 2013 Oct;40(8):669-70, 673-4, 677 54


Figura 4

5. Fongaufier C1, Guffroy A2, Lutz JC. Tongue and Scalp Necrosis: Simultaneous Initial Complications Revealing Giant Cell Arteritis. J Rheumatol. 2018; 45(6):873-874. 6. Truffaut L, Lefebvre P. Tongue Necrosis in Giant-Cell Arteritis. N Engl J Med. 2018 Jun 28; 378(26):2517.

55


22Âş Curso de DermatopatologĂ­a

56


Casos del Seminario

57


22Âş Curso de DermatopatologĂ­a

58


Caso del Seminario 1 Dra. L. Nájera, D ez Massa, G. Roustán, L. Tur rión. D.. Suár Suárez urrión. Hospital Universitario Puerta de Hierro . Majadahonda (Madrid). Historia Clínica Niña de 7 años que consultó por una tumoración en el dorso del quinto dedo de la mano izquierda. Se realizó exéresis. Descripción Microscópica El estudio histopatológico demostró una proliferación melanocítica dérmica, multinodular, sin maduración en profundidad y que afectaba prácticamente a todo el espesor de la dermis. Las células eran de gran tamaño, epitelioides y en la dermis profunda se agrupaban y mostraban mayor tamaño y pleomorfismo. Se contabilizaron hasta 7 mitosis/10 CGA. Los estudios citogenéticos demostraron fusión génica de NTRK1. Diagnóstico: TUMOR DE SPITZ ATÍPICO Evolución Se realizó ampliación de los márgenes quirúrgicos de resección y biopsia de ganglio centinela axilar derecho, sin encontrarse presencia de tumor residual, ni metástasis en el ganglio linfático. Después de 15 meses de la cirugía, la paciente está asintomática. Comentarios Las neoplasias spitzoides constituyen un grupo de tumores melanocíticos con características histopatológicas distintivas que incluyen tumores benignos (nevo de Spitz), tumores malignos (melanomas spitzoides) y tumores con características histopatológicas intermedias y comportamiento biológico incierto (tumores de Spitz atípicos). Las neoplasias spitzoides albergan fusiones de ROS1, NTRK1, ALK, BRAF y RET. Estas fusiones de quinasas son mecanismos implicados en la activación de oncogenes en estos tumores. La ocurrencia frecuente de fusiones de quinasas en todo el espectro biológico de las neoplasias spitzoides sugiere que ocurren de manera precoz en el desarrollo de estas neoplasias. Estas fusiones son necesarias, pero no suficientes para la transformación maligna. Los tumores de Spitz atípicos representan lesio-

Figura 1

Figura 2 59


22º Curso de Dermatopatología

Figura 3

nes en las que una distinción histopatológica definitiva entre benigno y maligno no siempre es posible. Es de esperar que en un futuro próximo los avances en la identificación de las características citogenéticas de estos tumores ayuden a establecer con mayor seguridad su diagnóstico histopatológico, así como a predecir con mayor certeza su comportamiento biológico. BIBLIOGRAFÍA 1. Pediatric melanomas and the atypical Spitzoid melanocytic neoplasm. Sarah J Hill et al. Am J Surg Pathol 2012, 203: 761-767. 2. Pediatric «STUMP» lesions: Evaluation and management of difficult atypical Spitzoid lesions in children. Wynnis L Tom. J Am Acad Dermatol 2011; 64: 559-72. 3. The Genomic Landscape of Childhood and Adolescent Melanoma. Charles Lu. Investigative Journal of Dermatology. 2015, 135: 816-823. 60


Caso del Seminario 2 Drs. D ez Massa, L. Nájera, I. Salguer o y G. Roustán D.. Suár Suárez Salguero Hospital Universitario Puerta de Hierro (Majadahonda-Madrid)

Figura 1

Historia Clínica Paciente mujer de 32 años con diagnóstico de Lupus eritematoso sistémico desde los 14 con afectación articular de difícil manejo terapeútico, serosa con pericarditis y hematológica con leucopenia y anemia hemolítica. En febrero de 2018 se diagnostica de Nefritis lúpica clase IV-G (A) y pocos días después desarrolla lesiones cutáneas eritemato-edematosas anulares de comienzo en el dorso de las manos que posteriormente se extendieron al cuello y la parte superior del tronco incluyendo la raíz de los miembros superiores. Se hizo una biopsia cutánea.

Descripción Microscópica Se remitieron dos biopsias una de ellas en fresco para estudio de inmunofluorescencia directa IFD). En la biopsia fijada en formol e incluida en parafina se observa un infiltrado neutrofílico a todo lo largo de la dermis papilar con formación de microabscesos en las papilas, fragmentación nuclear y despegamiento a este nivel. En la biopsia en fresco para IFD se observa una ampolla subepidérmica. En el techo de la ampolla se observa la epidermis intacta, contiene infiltrado inflamatorio neutrofílico. En la dermis sólo se aprecia escaso infiltrado inflamatorio mixto. No hay lesión de interfase ni 61


22º Curso de Dermatopatología

vasculitis. El estudio de IFD demostró depósitos lineales en la membrana basal que fueron intensos con IgG e IgA y más tenues con IgM y C3 Diagnóstico: LUPUS ERITEMATOSO SISTÉMICO BULLOSO Comentarios El Lupus eritematoso bulloso es un trastorno autoinmune raro, debido a anticuerpos anti colágeno tipo VII, que se da en pacientes con diagnóstico previo de lupus eritematoso sistémico. Los pacientes con lupus eritematoso sistémico pueden sufrir otros trastornos autoinmunes con formación de ampollas de los que esta entidad debe ser distinguida. Menos del 5% de los pacientes desarrollan ampollas en el curso de su enfermedad. El Lupus eritematoso bulloso tiende a presentarse en mujeres en la segunda y tercera décadas de la vida. En los niños es excepcional. Suele darse en pacientes con manifestaciones graves de Lupus sistémico, especialmente nefritis lúpica, como reflejo de la actividad de la enfermedad. Los pacientes con Lupus eritematoso bulloso tienen una mayor incidencia de HLA DR2 que se asocia con mayor riesgo de desarrollar enfermedades autoinmunes. Clínicamente se manifiesta como una erupción vesículo-bullosa pruriginosa, generalizada, que afecta en mayor medida a zonas expuestas aunque no exclusivamente. Las regiones más afectadas son la cara, el cuello, región supraclavicular, parte superior del tronco y las manos. Puede afectar también a la mucosa oral. Las vesículas pueden aparecer sobre un fondo eritematoso o urticariforme. Lesiones tipo urticaria se asocian hasta en el 31% de los casos. Las mucosas pueden estar afectadas. El cuadro histológico se caracteriza por ampollas subepidérmicas en las que el techo de la ampolla incluye toda la epidermis habitualmente intacta. Contienen fibrina, neutrófilos y restos nucleares fragmentados. En la piel adyacente hay microabscesos en las papilas dérmicas cuya confluencia da lugar a las ampollas. En la dermis superficial subyacente suele haber inflamación mixta perivascular. En ocasiones se asocia una vasculitis leucocitoclástica.

Figuras 2 y 3

En el estudio de IFD se demuestran depósitos de inmunoglobulinas (Igs) y complemento en la membrana basal de la piel lesional y perilesional. Es frecuente que haya varias Igs (IgG en el 100% de los casos, IgA en el 67% e IgM en el 50%). El patrón puede ser granular (40%) o lineal (60%). Los anticuerpos reconocen dos antígenos de 290kD y 125kD que corresponde a dos dominios del colágeno tipo VII, que son los mismos que en la Epidermolisis bullosa adquirida (EBA). El nivel en suero de autoanticuerpos anti-colágeno tipo VII puede estar relacionado con la actividad de la enfermedad. En los estudio de IFD con piel separada los depósitos de anticuerpo se fijan habitualmente en el suelo de la ampolla. Desde el punto de vista clínico el diagnóstico diferencial principal es con la Epidermolisis bullosa adquirida aunque en los pacientes con Lupus eritematos sistémico bulloso no se producen ampollas por medios mecánicos y las lesiones no dejan cicatriz. El cuadro histológico en la Epidermolisis bullosa adquirida se parece más al de un Penfigoide bulloso con ampollas subepidérmicas con infiltrado eosinofílico, rara vez las lesiones histológicas consisten en microabscesos neutrofílicos en las papilas. El subtipo de los anticuerpos IgG pue62


de ser también útil: en los pacientes con Epidermolisis bullosa adquirida suelen ser IgG1 e IgG4 y en los pacientes con Lupus eritematoso bulloso suelen ser IgG2 e IgG3. Desde el punto de vista histológico el diagnóstico diferencial es con otras entidades que se manifiestan con ampollas subepidérmicas y abscesos de neutrófilos en las papilas, especialmente con la Dermatitis herpetiforme y la Dermatosis lineal IgA. En la Dermatitis herpetiforme los depósitos inmunológicos son de IgA granulares en las papilas dérmicas. En la Dermatosis IgA lineal la IFD se demuestra depósito de IgA con distribución lineal en la membrana basal. En un 25% de los pacientes hay también otras inmunoglobulinas implicadas. Por otra parte no hay que olvidar que los pacientes con Lupus eritematoso sistémico pueden desarrollar otras enfermedades ampollosas incluyendo las anteriormente mencionadas. En conclusión, el Lupus eritematoso sistémico bulloso es una enfermedad ampollosa autoinmune que se asocia con enfermedad activa y grave. Para el diagnóstico diferencial es imprescindible la correlación clínico patológica, el estudio de inmunofluorescencia directa y, cada vez más, el reco-

nocimento de los diferentes anticuerpos implicados en cada entidad. Evolución de la paciente Las lesiones cutáneas evolucionaron bien. No se hizo un tratamiento específico con Dapsona pues la paciente estaba con tratamiento inmunosupresor para otras manifestaciones de su enfermedad con Belimumab (que hubo de suspenderse por la aparición de infecciones oportunistas), Prednisona, Micofenolato mofetil y Dolquine.

BIBLIOGRAFÍA 1. De Risi-Pugliese T, et al. Clinical, histological, immunological presentations and outcomes of bullous systemic lupus erythematosus: 10 New cases and a literature review of 118 cases. Semin Arthritis Rheum. 2018 Aug;48(1):83-89. 2. Grabell DA, et al. Detection of type VII collagen autoantibodies before the onset of bullous systemic lupus erythematosus. JAMA Dermatol, 2015; 151:539–43. 3. Yell J.A., et al.: Bullous systemic lupus erythematosus: revised criteria for diagnosis. Br J Dermatol. 1995;132:921–928.

63


22Âş Curso de DermatopatologĂ­a

64


Caso del Seminario 3 Dr Dr.. F F.. Pinedo Hospital del Alcor cón (Madrid). Alcorcón

Figura 1

Historia Clínica Mujer de 80 años, residente en Alcorcón, aunque originaria del Congo, al que viaja con frecuencia, fototipo VI, remitida por su médico de Atención Primaria por pigmentación de ambas mejillas y párpados inferiores de larga evolución. La comunicación con la paciente resulta muy difícil por razones idiomáticas. Además, por motivos culturales se niega a ser fotografiada. Es hipertensa y sigue tratamiento con nifedipino, atenolol y torasemida. Exploración Física: Hiperpigmentación con lesiones papulosas, algunas de aspecto comedónico, en regiones malares, párpados inferiores y mentó. Se realiza biopsia de la mejilla.

mis papilar con inclusión del pigmento en las fibras colágenas, con discreto ensanchamiento y fragmentación de éstas. Se aprecian muy ocasionales melanófagos. No se observa eliminación transepidérmica ni depósito alrededor de los vasos y los apéndices cutáneos. Se identifica un pequeño quiste de inclusión epidérmica (comedón) (fotos 1, 2, 3 y 4). Evolución Se reinterroga a la paciente, confirmándose el uso desde hace años de productos tópicos adquiridos en sus viajes a su país de origen. No se detecta ácido homogentísico en orina de 24 horas.

Descripción Microscópica Biopsia cutánea con elastosis solar, y sin alteraciones relevantes en la epidermis, en la que se identifican múltiples depósitos de un material acelular y de coloración ocre predominantemente en la der-

Diagnóstico OCRONOSIS EXÓGENA Comentarios El término de ocronosis cutánea fue utilizado 65


22º Curso de Dermatopatología

Figura 2

por primera vez por Virchow en 1866 para describir una pigmentación cutánea azul negruzca en la piel asociada a un depósito microscópico de un pigmento color ocre. Este término se encontraba en el contexto de un paciente con alcaptonuria, enfermedad hereditaria producida por mecanismos patogénicos endógenos. No fue hasta 1975 cuando Findlay describe una forma de ocronosis producida por el uso continuado de cremas despigmentantes con hidroquinona, que se convierte a partir de los años 80 en la primera causa de ocro-

nosis exógena del mundo. Ambas formas de ocronosis presentan diferentes formas de presentación clínica y alteraciones de laboratorio, pero una imagen histológica casi idéntica. La forma endógena (alcaptonuria), es una rara enfermedad hereditaria del catabolismo de la tirosina, autosómico recesiva, caracterizada por un déficit de la enzima homogentísico 1,2 oxigenasa. Esta deficiencia produce la eliminación continuada de ácido homogentísico en orina (alcaptonuria) y la

Figuras 3 y 4

66


acumulación de pigmento del metabolito en el tejido conectivo (ocronosis). Los rasgos clínicos más característicos son la emisión de una orina que se torna de color negro-parduzca al contacto con el aire; la pigmentación oscura en escleras y piel; la calcificación de los discos intervertebrales, una artropatía y la afectación cardiovascular, sobre todo estenosis de la válvula aórtica. El diagnóstico se confirma mediante la detección de ácido homogentísico en orina. La ocronosis exógena cursa con depósito y polimerización de ácido homogentísico en la dermis superficial, por la inhibición local de la enzima, a causa del uso prolongado de tratamientos tópicos como la hidroquinona y otros productos (mercuriales tópicos, antipalúdicos vía oral o parenteral, aplicación de fenol, resorcina, ácido pícrico, así como la administración de levodopa.). Se manifiesta como una dermatosis de distribución bilateral y simétrica, que se localiza generalmente en áreas fotoexpuestas y prominencias óseas, afectando más comúnmente a la región malar, mandibular, temporal y mentoniana. Se caracteriza por una hiperpigmentación papulosa y parcheada que recuerda el aspecto del caviar y que tapona los orificios foliculares, aunque las primeras etapas puede ser solo evidente un mínimo eritema con inflamación leve. En el diagnóstico de sospecha de la enfermedad tiene importancia el origen del paciente, ya que el uso de productos despigmentantes constituye una práctica extendida entre la población de los países del África subsahariana y en algunos países asiáticos, que realizan esta práctica con la finalidad de alcanzar un tono claro de la piel que parece relacionarse con la belleza y un mayor estatus social. Estos productos, que pueden adquirirse a bajo coste en la mayoría de los mercados populares, están compuestos principalmente por corticoides de alta potencia, hidroquinona, mercuriales y agentes cáusticos, la mayoría de las veces combinados entre sí. Entre los diagnósticos diferenciales clínicos se encuentran el melasma, mucho más frecuente, sin formaciones papulosas ni tapones foliculares; y la hiperpigmentación por acumulo de metales (pseudocronosis) donde la hiperpigmentación es normalmente generalizada, afectando a esclera y mucosas, y otros como nevus de Ota bilateral, hiperpigmentación por fármacos, pigmentación postinflamatoria y dermatosis papulosa nigra. El diagnóstico definitivo es histológico, al observar depósitos de ácido homogentísico en la der-

mis superficial en forma de estructuras amarillodoradas alargadas y curvilíneas, entre los haces de colágeno, sin provocar alteraciones en la epidermis ni anejos. Estas estructuras adoptan diferentes formas, siendo la más representativa la forma de «plátano» o «banana». El pigmento es autofluorescente, negro con azul de metileno, pero no se tiñe con van Gieson, tinción de Perls o Masson-Fontana. Puede estar presente en el epitelio y la membrana basal de las glándulas ecrinas, en las células endoteliales, y dentro de macrófagos dérmicos. En la ocronosis debido a hidroquinonas, la piel puede mostrar además melanófagos en la dermis superior asociada a despigmentación de los melanocitos basales epidérmicos. En las lesiones tempranas, existe edema y degeneración de los haces de colágeno, infiltrado inflamatorio histiocitario y de células plasmáticas. Con la cronicidad, pueden desarrollarse grandes gránulos eosinófilos amorfos, que se asemejan al milio coloide. La elastosis solar y los granulomas de cuerpo extraño (a veces indistinguibles de la sarcoidosis) son características menos frecuentes. Ocasionalmente se ha documentado la eliminación transepidérmica y transfolicular de las fibras ocronóticas. En el caso del uso de antipalúdicos existe también depósito de melanina y hemosiderina. Respecto al manejo terapéutico, la primera medida en el caso de la ocronosis exógena es la interrupción del uso del producto responsable. Aunque se han descrito múltiples tratamientos el uso de determinados tipos de láser parece ofrecer los mejores resultados al fragmentar los cuerpos ocres y facilitar su fagocitosis.

BIBLIOGRAFÍA 1. Barja JM, Berdeal M, Vares M, Carballo E. Xeesal: la despigmentación cosmética en África subsaariana. Actas Dermosifiliogr 2010; 101:2701. 2. Córdova, ME, Pérez-Rojas DO, López-Marqueta AD. Ocronosis exógena en melasma facial. Actas Dermosifiliogr 2017; 108:381-3. 3. Kindem S, Serra-Guillén C, Guillén C. Hiperpigmentación facial parcheada. Actas Dermosifiliogr 2015; 106:317-8. 67


22Âş Curso de DermatopatologĂ­a

68


Caso del Seminario 4 Dr Dr.. F F.. Pinedo Hospital del Alcor cón (Madrid). Alcorcón Historia Clínica Varón de 82 años, con: AP: Ex-fumador. HTA. Hipercolesterolemia. Diabetes mellitus tipo 2. Cardiopatía isquémica crónica con infarto inferior (2006) con implante de stent convencional en coronaria derecha. Enfermedad renal crónica estadio 3. Colecistopancreatitis secundaria a coledocolitiasis (2014). Amaurosis derecha post-exposición al cemento. Podagra izquierda. Vitíligo. Diagnosticado de linfoma no Hodking B folicular grado 1 estadio IIB en marzo de 2018 con afectación bulky abdominal que condiciona uropatía obstructiva secundaria que ha precisado colocación de catéter doble J. Inicia tratamiento con esquema RBD (rituximab + bendamustina + dexametasona) en abril de 2018 sin incidencias. En tratamiento habitual con Adiro® 100 mg (ácido acetil salicílico), amlodipino 5 mg, doxazosina 8 mg, atorvastatina 20 mg, furosemida 40 mg, bisoprolol 2.5 mg, losartán 50 mg, omeprazol 20 mg, resincalcio, repaglinida 0.5, cotrimoxazol y alopurinol (profilácticos). Motivo de consulta (mayo 2018): Acude a cita de control de Hematología y para administración de segundo ciclo con esquema RBD, objetivándose lesiones cutáneas generalizadas ligeramente pruriginosas, con exudación muy intensa en zonas distales de miembros inferiores y testículos, además de mucositis grado 1, con lesiones sugestivas de candidiasis oral. Se considera el cuadro compatible con una toxicodermia medicamentosa en probable relación con cotrimoxazol, por lo que se realiza biopsia cutánea (abdomen), se suspende el fármaco y se cursa ingreso a cargo de Hematología. Se realizan curas con sulfato de zinc, además de fluconazol y corticoterapia (prednisona 30 mg/día). Descripción Microscópica Biopsia cutánea revestida por una epi-

Figuras 1 y 2 69


22º Curso de Dermatopatología

Figura 3

dermis de morfología aserrada, desprendida en gran parte de la dermis subyacente, con hipergranulosis e hiperqueratosis ortoqueratósica. Se acompaña de dermatitis de interfase, con numerosos queratinocitos necróticos tanto en la capa basal como en estratos superiores. Se aprecian ocasionales eosinófilos (fotos 1, 2 y 3). Diagnóstico anatomopatológico: Dermatitis de interfase, de patrón liquenoide, compatible con dermatitis medicamentosa.

sico, motivo por el que se administra nueva dosis de Rituximab. Diagnóstico PÉNFIGO PARANEOPLÁSICO. Ante el empeoramiento del cuadro, se decide ingreso en la UCI. Durante su estancia en la misma, ha presentado neumonía nosocomial, ITU, conjuntivitis y bacteriemia persistente, todos ellos por pseudomona aeruginosa, así como perforación y evisceración ocular derecha. Finalmente desarrolla shock séptico con fracaso multiorgánico refractario, siendo exitus en julio de 2018. La inmunofluorescencia indirecta con esófago de mono y vejiga de rata fue positiva (anticuerpo anti sustancia intercelular). Mediante ELISA se detectaron anticuerpos anti desmogleina-1 y anti desmogleina-3.

Evolución El paciente presenta muy escasa mejoría inicial, aumentándose los corticoides a 60 mg/día e iniciándose antihistamínicos. Aunque parece objetivarse lenta mejoría clínica (excepto el prurito) en los días sucesivos las lesiones son cada vez más extensas (fotos 4, 5, 6 y 7) y a pesar del tratamiento pautado persisten muy eritematosas, por lo que en junio de 2018 se repite la biopsia cutánea (brazo derecho). Biopsia (B): Biopsia cutánea con marcada elastosis solar, similar a la previa, revestida por una epidermis con acantosis, hipergranulosis e hiperqueratosis ortoqueratósica con algún parche de paraqueratosis. Como en la biopsia anterior, se acompaña de dermatitis de interfase con numerosos queratinocitos necróticos (fotos 8, 9 y 10). El estudio de inmunofluorescencia directa muestra depósito intercelular intraepidérmico focal de C3, de muy débil intensidad. Comentada la biopsia en sesión clínicopatológica se considera compatible con pénfigo paraneoplá-

Comentarios El pénfigo paraneoplásico (PPN) es una enfermedad ampollosa autoinmune poco frecuente que se presenta en el contexto de una neoplasia oculta o previamente diagnosticada. Fue descrito por primera vez con este nombre por Anhalt et al en 1990, aunque dado que pueden estar comprometidos varios órganos además de la piel y que los mecanismos fisiopatogénicos que intervienen en las lesiones no se limitan a la presencia de autoanticuerpos específicos contra las moléculas de adhesión, Nguyen et al propusieron en el año 2001 el término 70


Figuras 4 y 5

Figura 6

Figura 7

tomatitis es constante y muchas veces es un signo temprano de la enfermedad, tanto que su ausencia debe hacer dudar del diagnóstico de PPN. Las lesiones cutáneas suelen asentar en el tronco, las extremidades, la cabeza y el cuello y pueden ser muy polimorfas y remedar tanto a un pénfigo, un liquen plano, un eritema multiforme o una enfermedad de injerto contra huésped. El árbol traqueobronquial puede estar directamente comprometido, produciendo un fallo respiratorio por la presencia de bronquiolitis obliterante debido a los autoanticuerpos contra las plaquinas del epitelio bronquial, que ocasiona una insuficiencia respiratoria progresiva de pronóstico fatal. Es una complicación frecuente que afecta a un 30% de los casos en adultos, y a un 70% de los casos pediátricos. Anatomía Patológica: Los hallazgos histológicos son tan variados como el polimorfismo lesional que presentan estos pacientes. Se puede observar acantólisis suprabasal (menos marcada que en el pénfigo vulgar) y/o dermatitis liquenoide con diferentes grados de necrosis de los queratinocitos. Esto se debe a la coexistencia de una respuesta humoral y celular producto de un fenómeno de autoinmunidad contra el epitelio.

síndrome multiorgánico autoinmune paraneoplásico (SMAP). Sin embargo, para autores como Mascaró, debe mantenerse el nombre original en reconocimiento al Dr. Anhalt y para evitar confusión con otros procesos que podrían encajar también en el término propuesto. El PPN se asocia con mayor frecuencia en los adultos con linfomas no Hodgkin (B, en general), leucemia linfática crónica, macroglobulinemia de Waldeström y enfermedad de Castleman y menos habitualmente con timomas, sarcomas y carcinomas. En el momento del diagnóstico, la neoplasia subyacente se conoce en dos tercios de los casos. En niños y adolescentes la neoplasia más frecuente es la enfermedad de Castleman, y debutan con desconocimiento del proceso asociado en un 7080% de los casos. Se ha observado, además, concomitancia con otras enfermedades autoinmunes. Clínica: El PPN se caracteriza por la presencia de mucositis en forma de erosiones y ulceraciones dolorosas en la mucosa oral que se extienden hasta el bermellón de los labios, de muy difícil manejo y refractarias al tratamiento. Otras mucosas, como la conjuntival, traqueobronquial, esofágica y anogenital también pueden estar comprometidas. La es71


22º Curso de Dermatopatología

Figura 8

Inmunofluorescencia: Los hallazgos de la inmunofluorescencia directa muestran inmunoreactantes (IgG y C3) intercelulares, aunque es frecuente también hallar depósito granular de IgG y/o complemento en la membrana basal, siendo este dato útil en el diagnóstico diferencial con el pénfigo vulgar. Es importante remarcar que existe un número considerable de falsos negativos, más elevada que en otras formas de pénfigo; este hecho puede deberse a la presencia de tejido necrótico en las biopsias o al predominio de lesiones liquenoides, lo que demuestra, en estos casos, el predominio de la inmunidad celular sobre la humoral. En cuanto a la inmunofluorescencia indirecta, el suero de los pacientes contiene anticuerpos (sobre todo IgG), que reaccionan con los espacios intercelulares del epitelio de esófago de mono u otros epitelios planos estratificados, así como con el epitelio transicional de la vejiga de rata. Autoantígenos: Hay anticuerpos antidesmogleína (desmogleínas 1 y 3), anticuerpos antiplaquina (envoplaquina, periplaquina, desmoplaquina 1 y 2, plectina) y anticuerpos dirigidos contra una proteína de 170 kD no caracterizada. Sin embargo, existe un subgrupo de pacientes en los que los anticuerpos antidesmoplaquinas pueden estar ausentes, por lo que los anticuerpos contra la envoplaquina y la periplaquina se consideran los más específicos. Pronóstico: El diagnóstico precoz facilitará el hallazgo de la neoplasia oculta, en los casos que no se haya detectado, así como también la implementación temprana de un tratamiento adecuado. No obstante, presenta una tasa de mortalidad del 90%. Las causas de muerte suelen ser las complicaciones de la enfermedad y/o del tratamiento: sepsis, fallo multiorgánico, hemorragias gastrointestinales o fallo respiratorio, producto de la bronquiolitis obliterante.

Figuras 9 y 10

BIBLIOGRAFÍA - Cervini AB, Tosi V, Kim SH, Bocian M, Chantada G, Nousari C, Carballo OG, Pierini AM. Pénfigo paraneoplásico/síndrome multiorgánico autoinmune paraneoplásico. Presentación de dos casos en la edad infantil. Revisión de la literatura. Actas Dermosifiliogr 2010; 101:879-86.

- Hidalgo I, Martinez F, Grau C, Gil I, Azón A. Síndrome multiorgánico autoinmune paraneoplásico asociado a linfoma folicular. Actas Dermosifiliogr 2012; 103:244-6. - Mascaró Galy JM, Iranzo Fernández P, Herrero Mateu C. Pénfigo paraneoplásico. Piel. 2007; 22:63-71. 72


Caso del Seminario 5 Drs. V. Alegría, C C.. Santonja y L. Requena Hospital Fundación Jiménez Díaz (Madrid).

Figuras 1 y 2

Historia Clínica Mujer de 43 años diagnosticada de enfermedad de Crohn desde los 22 años habiendo presentado multiples exacerbaciones de la enfermedad que condujeron a la instauración de varias líneas de tratamientos, tanto médicos (5- aminosalicilatos, azatioprina, mercaptopurina, fármacos anti-TNF) como quirúrgicos (dilataciones, extirpaciones parciales y estricturoplastias). En los últimos años la sintomatología se había centrado en la región perineal donde podían apreciarse grandes conglomerados con aspecto en empedrado y trayectos fistulosos, que producían un importante perjuicio en la calidad de vida de la paciente. Coincidiendo con un nuevo brote de la patología, aparecieron de manera súbita en cara anterior de piernas y de manera simétrica, nódulos eritematosos, mal delimitados, con aumento de la temperatura local, dolorosos a la palpación, que se interpretaron clínicamente como eritema nudoso. El proceso inflamatorio remitió con reposo, medidas postulares y yoduro potásico.

conjuntivo que aparecían ocupados por granulomas no caseificantes de tipo tuberculoide. No se observaron hallazgos de vasculitis acompañante. Diagnóstico ERITEMA NUDOSO GRANULOMATOSO SECUNDARIO A ENFERMEDAD DE CROHN Comentarios La enfermedad inflamatorio intestinal (EII) es un proceso crónico inespecífico cuya etiología y patogénesis no se encuentran aún completamente dilucidadas. El proceso incluye la colitis ulcerosa (CU) y la enfermedad de Crohn (EC). Se acepta que factores ambientales, genéticos, inmunes así como la microbiota intestinal actuarían conjuntamente para dar lugar a una respuesta inmune altera-da frente a determinados antígenos intestinales como metabólicos alimentarios o ciertos microorganismos. Aproximadamente un 40% de pacientes con EII pueden desarrollar manifestaciones extra-intestinales y el espectro clínico de las mismas puede ser muy extenso, a veces con manifestaciones incluso más severas que la propia EII. Entre todas las manifestaciones extra-intestina-

Descripción Microscópica El estudio histopatológico de una de las lesiones reveló la presencia de un panículo adiposo llamativamente ensanchado a expensas de septos de tejido 73


22º Curso de Dermatopatología

Figuras 3 y 4

les, las más comunes las cutáneo-mucosas, seguidas por las de sistema musculoesquelético, ocular, hepatobiliar o endocrinológico. Entre el 22-75% de los pacientes con EC y el 5-11% de los pacientes con CU tienen manifestaciones cutáneas, y alrededor del 10% de estas manifestaciones están presentes en el momento del diagnóstico de la EII. En base a la asociación fisiopatológica con la

Figura 5 74


Figura 6

enfermedad intestinal subyacente, las manifestaciones cutáneas pueden clasificarse en 4 categorías: 1. Manifestaciones cutáneas específicas: los hallazgos histopatológicos son los mismos que la enfermedad intestinal de base por lo que se considera que se trata del mismo trastorno que se presenta fuera del aparato digestivo (EC oral, EC perianal y EC metastásica); 2. Manifestaciones cutáneas reactivas: no comparten necesariamente los mismos hallazgos histopatológicos y serían la consecuencia de una respuesta inmune a flora patógena compartida entre el intestino y la piel (eritema nudoso, pioderma gangrenoso, síndrome de Sweet, piodermatitis-pioestomatitis vegetans, síndrome SAPHO, síndrome PAPA); 3. Manifestaciones cutáneas secundarias a desnutrición o malabsorción: debidas al déficit de vitaminas y oligoelementos (acro-dermatitis enteropática, escorbuto, alteraciones de uñas y pelo, queilitis angular, frinoderma); 4. Manifestaciones cutáneas secundarias al tratamiento: reacciones en el sitio de inyección, infecciones, reacciones psoriasiformes, cáncer de piel, síndromes lupus eritematoso-like, dermatomiositis, reacciones granulomatosas, alopecia areata, eritema multiforme, síndrome de Stevens-Johnson, ne-crolisis epidérmica tóxica, entre otras. El eritema nudoso es la manifestación cutánea

más frecuente en los pacientes con EII, sobre todo en la EC (4-15% ). En el caso que presentamos aquí la controversia se plantea por el hallazgo de granulomas no caseificantes en lesiones que por otro lado corresponden a eritema nudoso clásico tanto en presentación clínica como en la afectación septal del panículo adiposo y que podrían plantear la posibilidad de que estuviéramos frente a una enfermedad de Crohn metastásica subcutánea. Se discute el diagnóstico diferencial histopatológico entre el eritema nudoso en fase granulomatosa y la afectación cutánea específica de la enfermedad de Crohn. BIBLIOGRAFÍA 1. Apgar JT. Newer aspects of inflammatory bowel disease and its cutaneous manifestations: a se-lective review. Semin Dermatol. 1991;10:138147. 2. Bender-Heine A, Grantham JT, Zaslau S, Jansen R. Metastatic Crohn disease: a review of derma-tologic manifestations and treatment. Cutis.2017; 99:E33-E40. 3. Greuter T, Navarini A, Vavricka SR. Skin manifestations of inflammatory bowel disease. Clin Rev Allergy Immunol. 2017;53:413-427. 75


22Âş Curso de DermatopatologĂ­a

76


Caso del Seminario 6 Drs C. Saus Hospital Son Espases (Palma de Mallorca).

Figura 1

Historia Clínica Mujer de 51 años presenta lesiones cutáneas eritematosas y pruriginosas de 2 días de evolución que aparecieron mientras trabajaba en su jardín. No había tomado medicamentos y nunca antes había tenido una reacción similar. No tenía fiebre y no había nada relevante en su hitoria clínica. En el examen físico, se observó una erupción papular eritematosa localizada en el antebrazo derecho del paciente. La dermatoscopia mostró múltiples líneas finas de color marrón y puntos rojos. Se tomó una biopsia de una lesión.

servó una estructura bien definida, refringente, tubular marrón en la dermis superficial, rodeada de eosinófilos. Diagnóstico ERUCISMO POR PROCESIONARIA Comentarios La oruga procesionaria pertenece a la familia Notodontidae, que es dañina para los humanos. La mayoría de los estudios sobre esta familia se han realizado en especies de Taumetopoea, particularmente en Thaumetopoea pityocampa, de Europa y el área del Mediterráneo. De enero a mayo, grupos de orugas de polilla procesionaria se mueven en una sola fila y descienden de los árboles al suelo para iniciar la fase de crisálida antes de convertirse

Descripción Microscópica El examen histológico reveló un infiltrado inflamatorio superficial perivascular e intersticial con abundantes eosinófilos. Tras cortes seriados, se ob77


22º Curso de Dermatopatología

Figuras 2 y 3

en polilla nocturna. En la fase larvaria, la oruga produce sedas urticantes (pelos urticantes) de 150 a 200 µm. en su parte posterior (Fig. 1) que se pueden propagar fácilmente por contacto directo o por el viento. Clínicamente, las sedas pueden producir reacciones en la piel (erucismo), afectación ocular y síntomas respiratorios, bien por una reacción mecánica irritativa directa de las sedas , bien por alergia mediada por IgE, que explicaría reacciones más graves en pacientes previamente expuestos. El estudio histopatológico no es necesario para el diagnóstico, pero puede ser útil en casos dudosos. Cuando se realiza, se suele observar un patrón no específico, caracterizado por un fenómeno necrótico localizado, formación de vesículas, Figura 4 infiltrado linfocítico y eosinófilos. No hemos encontrado descripciones previas en 2. Vega J, Vega JM, Moneo I. Skin reactions on la literatura en que las sedas se hayan visto en el exposure to the pine processionary caterpillar dermis en relación a la inflamación. El hallazgo en (Thaumetopoea pityocampa). Actas Dermosifiliodermis de una estructura tubular marrón , ligeragr 2011; 102: 658–67. mente refringente, no birregringente ,en cortes de 3. Vega J, Vega JM, García-Ortiz JC, Sánchezhematoxilina eosina y es característico y por lo tanVelicia L. Diagnostic utility of dermoscopy in cuto una clave diagnóstica. taneous reactions to Thaumetopoea pityocampa. J La dermatoscopia también puede ser útil, ya que Eur Acad Dermatol Venereol 2016 Oct; 30: e76– muestra las sedas en la piel como líneas finas de 7 color marrón, incluso varias semanas más tarde, lo 4. Corral-Magaña O, Del-Pozo-Hernando LJ, que permite el diagnóstico en una etapa tardía desQuintero AM, Saus C Erythematous papular rash pués del contacto. after gardening. Clin Exp Dermatol. 2018 Oct;43(7):832-834.

BIBLIOGRAFIA 1. Battisti A, Larsson S, Roques A. Processionary moths and associated urtication risk: global change-driven effects. Annu Rev Entomol 2017; 62: 323–42. 78


Caso del Seminario 7 Dr Dr.. C C.. Saus Hospital Son Espases (Palma de Mallorca).

Figura 1

Historia Clínica Mujer de 34 años con una lesión en la espalda de tiempo de evolución indeterminado, sugestiva clínicamente de dermatofibroma o quiste epidérmico inflamado.

CD117, Enolasa, E-Cadherina, WT1 (focal) y HMB-45 (focal),Melan-A (focal). El Ki-67 es de menos del 5% con una distribución homogénea. Resultan negativos la desmina, factor VIII, actina, EMA, CD10, CD99, Panel de Citoqueratinas, ALK, CD56, CEA, fascina y neurofilamentos. Se descartó sarcoma de células claras por FISH , que fue negativo para reordenamiento del gen EWS. Se realizó búsqueda de tumor melanocítico primario de otra localización, incluyendo mucosas, que fue negativa. Con estos datos se propuso el diagnóstico de tumor dérmico melanocítico de bajo grado o de malignidad incierta, de tipo tumor melanocítico dérmico «paraganglioma-like» (1). (año 2011). La paciente he sido seguida durante 7 años y no ha presentado recidiva ni metástasis. Recientemente, el grupo de Arnaud de la Fouchardière (2) describe un grupo de 5 casos de

Descripción Microscópica Microscópicamente se observa una tumoración nodular dérmica revestida por epidermis ligeramente hiperplásica. La tumoración ocupa todo el espesor del dermis afectando profun didad y está constituida por fascículos cortos y grupos de células fusiformes o epitelioides, con núcleos redondeados y nucléolos con frecuencia prominentes. Cromatina fina. Abundante citoplasma anfófilo en el que no se identifica pigmento melánico. Ocasionales figuras de mitosis a distintos niveles (1mitosis / mm2) . La inmunohistoquímica resulta positiva para Vimentina, S100, BCL2, Ciclina D1, FLI1 y 79


22º Curso de Dermatopatología

Figuras 2, 3, 4 y5

tumores nodulares no pigmentados primarios, con un aspecto histológico e inmunofenotipo melanocítico similar a nuestro caso, asociado a una novedosa fusión CRTC1-TRIM11. Se remitió el bloque a dicho centro que confirmándose el diagnóstico por FISH.

das ni eventos metastásicos durante el seguimiento (3 a 72 meses). El principal diagnóstico diferencial fué el sarcoma de células claras cutáneo. Las características morfológicas se superponen a las del melanoma dérmico primario y las de los tumores melanocíticos «paragangliomas-like». La fusión CRTC1-TRIM11 parece asociarse a un fenotipo no tumoral no pigmentado y un comportamiento de bajo grado. Sin embargo se requieren estudios adicionales con mayor número de casos para confirmar el comportamiento potencial de bajo grado de estos tumores, independientemente de su atipia morfológica.

Diagnóstico: MELANOCITOMA CUTÁNEO CON FUSIÓN CRTC1-TRIM11 Comentarios El «melanocitoma cutáneo con fusión CRTC1TRIM11» se describe como un tumor dérmico nodular de células que se disponen en nidos confluentes y fascículos, a veces con áreas centrales fibronecróticas. Las células son medianas o grandes, a veces multinucleadas, y de citología fusiforme epitelioide, con nucléolos prominentes no pigmentadas. Las mitosis oscilan entre 1 y 5 / mm². La inmunohistoquímica muestra constantemente una positividad difusa con S100, MiTF y Sox10, y una positividad heterogénea con MelanA o HMB45. Citoqueratinas de alto y bajo peso, EMA, SMA y neurofilamentos son negativas. En todos los casos, se demostró fusión CRTC1 -TRIM11. Ninguno de los 5 casos descritos presentó recaí-

BIBLIOGRAFÍA 1. Deyrup AT, Althof P, Zhou M, et al. Paraganglioma-like dermalmelanocytic tumor: a unique entity distinct from cellular blue nevus,clear cell sarcoma, and cutaneous melanoma. Am J Surg Pathol.2004;28:1579–1586. 2. Cellier L, Perron E, Pissaloux D, Karanian M, Haddad V, Alberti L, de la Fouchardière A. Cutaneous Melanocytoma With CRTC1-TRIM11 Fusion: Report of 5 Cases Resembling Clear Cell Sarcoma. Am J Surg Pathol. 2018 Mar;42(3):382391. 80


Caso del Seminario 8 Drs. C. Santonja y Carmen Ceballos Hospital Fundación Jiménez Díaz (Madrid).

Figura 1

Historia Clínica Varón de 47 años con infección VIH y VHB, en tratamiento antirretroviral (intermitente), que a los 14 días de introducción de Genvoya (Elvitegravir, Cobicistat, Emtricitabina y Alafenamidad de Tenofovir) presenta fiebre intermitente no termometrada y pápulas rojizas en cuello, parte superior del tronco, antebrazos y manos, no pruríticas, algo más sensibles que la piel circundante. También clínica de escozor al defecar, según el paciente debido a herida perianal; se observa una ulceración superficial a las 4h de disposición radial y fondo fibrinoso, dolorosa al roce. En la analítica, únicamente elevación de PCR y VSG. Con la sospecha clínica de síndrome de Sweet

(SS), eritema exudativo multiforme y prurigo estrófulo se realizan dos biopsias (cuello y espalda) separadas por dos meses. Descripción Microscópica En ambas muestras, bajo epidermis sin lesiones significativas, afectación en banda de la dermis superficial y media (sin edema apreciable) por infiltrado inflamatorio mononuclear intersticial o perivascular, con proporciones variables de linfocitos (CD3), histiocitos (CD163) y células de núcleo ovalado o arriñonado, positivas en las tinciones para mieloperoxidasa (MPO) y antígeno nuclear de diferenciación mieloide (MNDA). 81


22º Curso de Dermatopatología

Figuras 2 y 3

Diagnóstico SÍNDROME DE SWEET HISTIOCITOIDE EN PACIENTE HIV, EN RELACIÓN CON CAMBIO DE MEDICACIÓN ANTIRETROVIRAL

infección del tracto respiratorio superior, la coexistencia en nuestro caso de linfogranuloma venéreo es presumiblemente casual.

BIBLIOGRAFÍA 1. Berger TG et al Neutrophilic dermatoses in HIV infection. JAAD 1994;31:1045 2. Bevilacqua S et al Sweet syndrome in an HIVinfected patient. AIDS 1999;13:728 3. Requena L et al Histiocytoid Sweet syndrome: a dermal infiltration of immature neutrophilic granulocytes. Arch Dermatol 2005;141:834 4. Alegría-Landa et al Clinicopathologic, immunohistochemical, and molecular features of histiocytoid Sweet syndrome. JAMA Dermatology 2017;153:651

Evolución y comentarios Tras el diagnóstico patológico inicial se pautó prednisona oral (30 mg/día), con recaída de las lesiones al intentar reducir las dosis (se realizó entonces la segunda biopsia). Se detectó endoscópicamente proctitis intensa y en el exudado purulento estudiado se identificó mediante PCR Chlamydia tracomatis, que se trató con Doxiciclina. La variedad histiocitoide del SS fue descrita por Requena et al en 2005. En ella el infiltrado dérmico -a diferencia del de la forma convencional- es de aspecto monocitario-histiocítico pero muestra datos inmunohistoquímicos(MPO, MNDA) concordantes con población inmadura de granulocitos neutrófilos, acompañada de macrófagos CD163+ de distribución a menudo periférica. Al igual que en el SS habitual, los cuadros clínicos asociados abarcan un amplio espectro (desde administración de fármacos a enfermedades autoinmunes, pasando por enfermedad inflamatoria intestinal, artritis reumatoide, policondritis recurrente, parotiditis, uveítis y neoplasias). La frecuencia de asociación con neoplasias convencionales o hematológicas no es distinta de la observada en el SS convencional. Nuestro caso es peculiar por tratarse de un paciente con infección HIV; en 1994 Berger et al describieron dos pacientes con SS convencional, a uno de los cuales se le había administrado factor estimulante de colonias de granulocitos. Bevilacqua et al describieron en 1999 otro ejemplo, en el que las lesiones dermatológicas siguieron a la administración de antirretrovirales (a semejanza de nuestro caso, en el que hubo un cambio de medicación). Aunque a menudo el exantema va precedido de una 82


Caso del Seminario 9 Dras. M.J. Beato y A. Sánchez Or ta Orta Hospital Universitario La Paz (Madrid).

Figuras 1 y 2

Historia Clínica Una paciente de 26 años, diagnosticada de seudoxantoma elástico desde la infancia, consultó porque, en los últimos diez años, sobre las lesiones cervicales del seudoxantoma, presentaba otras lesiones que constantemente se inflamaban. A la exploración física, el cuello mostraba grandes placas amarillentas y, sobre ellas, había papulopústulas y comedones salpicados.

Comentarios El seudoxantoma elástico es una enfermedad hereditaria autosómica recesiva que produce la calcificación y fragmentación de fibras elásticas en determinadas zonas de la piel, ojos y sistema cardiovascular. Tiene una incidencia de 1/50.000100.000 nacimientos, es más prevalente en mujeres (2:1) y se debe a distintas mutaciones en el gen ABCC6. Las manifestaciones de la piel suelen ser las primeras, y comienzan en la segunda o tercera décadas de la vida. Se producen fundamentalmente en las flexuras, sobre todo en el cuello y axilas, donde aparece un reticulado de pápulas amarillentas que confluyen en placas, y producen una piel laxa. Al microscopio, la dermis reticular muestra fibras elásticas cortas, curvas y calcificadas (basófilas), que con las técnicas de von Kossa y Verhoeff se ven negras. En los ojos puede haber estrías angioides o una superficie «en piel de naranja» de la retina, entre otras alteraciones. Con respecto al sistema cardiovascular, la calcificación de arterias de mediano calibre puede traducirse en hipertensión arterial o infartos miocárdicos. En ocasiones, el seudoxantoma elástico puede ser perforante. En estos casos, sobre las placas amarillentas cutáneas se desarrollan lesiones acneifor-

Descripción Microscópica La biopsia de una de las pápulas, que se extirpó con un huso cutáneo amplio, mostró dos tipos de alteraciones. Por una parte, la dermis mostraba numerosas fibras elásticas cortas y basófilas (Fig. 2). No obstante, la lesión más llamativa eran túneles rodeados por un epitelio grueso, alrededor del cual había zonas con detritus celulares y las fibras elásticas mencionadas, que también podían verse en la luz, a lo largo del trayecto hacia la superficie (Figs. 3-6). Diagnóstico PSEUDOXANTOMA ELÁSTICO PERFORANTE 83


22º Curso de Dermatopatología

Figura 3

Figuras 4, 5 y 6

mes. La biopsia muestra túneles rodeados por una hiperplasia seudoepiteliomatosa o acantosis, en cuyas luces pueden verse las fibras elásticas basófilas. También es frecuente observar células gigantes multinucleadas y un cierto infiltrado inflamatorio, así como elastofagocitosis.

BIBLIOGRAFÍA 1. Patterson JW. Pseudoxanthoma elasticum. En: Patterson JW, ed. Chapter 12: Disorders of elastic tissue. Weedon’s Skin Pathology, 4ª ed. Churchill Linvingstone Elsevier, 2016: 388-9. 2. Andrés-Ramos I, Alegría-Landa V, Gimeno I, Pérez-Plaza A, Rütten A, Kutzner H, Requena L. Cutaneous elastic tissue anomalies. Am J Dermatopathol 2019; 41(2): 85-117. 84


Caso del Seminario 10 Dra. M.J. Beato, C. Chiloeches y E. Sendagorta Hospital Universitario La Paz (Madrid). Historia Clínica Una mujer de 42 años, violonchelista, estaba siendo tratada en el Servicio de Cirugía plástica por una úlcera. La paciente la refería como secundaria a un traumatismo previo, de aproximadamente un mes de evolución, y la describía como no dolorosa. A la exploración, se observaba una gran úlcera, situada en la cara ventral del antebrazo derecho, de 10 x 7 cm y de bordes necrosados. La úlcera era profunda, y dejaba al descubierto los tendones flexores (Fig. 1). La paciente no tenía otras lesiones. Se hizo una biopsia del borde de la úlcera. Descripción Microscópica El lecho de la úlcera mostraba una gran necrosis dérmica, y el borde, que tenía una dermis conservada, mostraba en cambio una extensa necrosis epidérmica (Figs. 2-4). No se observaron infiltrado inflamatorio, imágenes de vasculitis, ni otras alteraciones. Todos los estudios microbiológicos fueron normales. Evolución Tras la biopsia, la paciente sufrió una crisis de ansiedad. La mujer padecía una depresión mayor, y había tenido varios intentos previos de suicidio. Contaba una historia de mucha presión en el trabajo, haber sufrido acoso laboral, una relación problemática con su madre, y haber sido víctima de violaciones repetidas en su infancia. La lesión apareció unos días antes de una prueba muy importante en la vida

Figura 1

laboral de la paciente. Al principio, la paciente aseguraba desconocer la causa de su úlcera; no obstante, cuando se le insistió, reconoció haber utilizado sosa cáustica como desatascador los días previos a la aparición de la lesión, y dijo que era posible que el producto la hubiera dañado. 85

Diagnóstico ÚLCERA AUTOINFLIGIDA SECUNDARIA A CAÚSTICO Comentarios Las dermatosis autoinducidas son lesiones provocadas por el mismo paciente, de forma cons-


22º Curso de Dermatopatología

Figura 2

ciente o inconsciente. Es más frecuente en niñas o mujeres jóvenes, normalmente con algún problema psicológico, que suelen negar la culpa. Los mecanismos son muy diversos: mecánicos (por presión, corte, succión,…), tóxicos (quemaduras térmicas o químicas), por inoculación (infecciones) o inyección (medicamentos, objetos,…). Dependiendo del método, las lesiones pueden variar desde simples erosiones a grandes úlceras, ampollas, cicatrices o nódulos subcutáneos. No obstante, suelen tener una distribución «no concordante con las enfermedades orgánicas» o una forma peculiar, por ejemplo lineal o geométrica. Suelen ser lesiones múltiples y situadas en lugares accesibles para el paciente. Los hallazgos histopatológicos son muy diversos. Puede haber excoriaciones, úlceras, quemaduras, dermatitis irritativas o alérgicas, alopecia, abscesos, celulitis, paniculitis supurativas o hemorragias. En ocasiones se encuentra un cuerpo extraño. A pesar de que históricamente los hallazgos anatomopatológicos han sido inespecíficos, recientemente se han descrito algunos signos más específicos, como una necrosis epidérmica masiva con bordes abruptos, queratinocitos muy multinucleados en la epidermis y alargamiento de los núcleos de los queratinocitos (en el caso de herida eléctrica). En la mayoría de los casos, sean cuales sean los hallazgos histopatológicos, suele ser necesario un soporte clínico, debido a la mencionada falta de signos específicos, a las implicaciones legales y a la posibilidad de que se pase por alto una enfermedad orgánica.

Figuras 3 y 4

agents. Weedon’s Skin Pathology, 4ª ed. Churchill Linvingstone Elsevier, 2016: 612. 2. Persad L, Sadia S, Motaparthi K. Factitious dermatitis due to thermal burn with histologic features simulating fixed drug eruption. Am J Dermatopathol 2017; 39 (8): 622-4. 3. Gutierrez D, Schowalter MK, Piliang MP, et al. Epidermal multinucleated keratinocytes: a histopathologic clue to dermatitis artefacta. J Cutan Pathol 2016; 43: 880–3. 4. Winters C, Machan M, Liu D, et al. Multinucleated giant cells in factitial dermatitis. J Cutan Pathol 2013; 40: 205–8.

BIBLIOGRAFÍA 1. Patterson JW. Dermatitis artefacta. En: Patterson JW, ed. Chapter 21: Reactions to physical 86


Caso del Seminario 11 Drs. J. Cue vas, I. Eraña, M. Alonso ersitario de Guadalajara, Cuevas, Alonso.. Hospital Gral. Univ Universitario Hospital Fundación Jiménez Díaz y Hospital Universitario 12 de Octubre (Madrid).

Figura 1

Historia Clínica Varón de 77 años con lesión de 4 meses de evolución, presenta en cuero cabelludo, formación nodular elevada de superficie eritematosa de 1,5 cms. de diámetro (Fig. 1) Descripción Microscópica Piel que muestra una neoplasia con disposición nodular, que eleva la epidermis y que solamente de manera muy focal, contacta con la epidermis no objetivándose ulceración. La neoplasia de modo difuso se manifiesta como crecimiento sólido con afectación de la dermis superficial y profunda así como la hipodermis superficial. La neoplasia adopta en zonas, patrón fusocelular y en la mayor parte de la lesión, aspecto marcadamente epitelioide con núcleos muy irregulares, nucleolo vesiculoso, frecuente multinucleación y presencia muy abundante de mitosis llegándose a alcanzar hasta siete mitosis por milímetro cuadrado. En el seno de la tumoración descrita, densa epitelioide y fusocelular con células grandes, se evidencia pequeña área nodular, de células también atípicas con mitosis pero que presentan, nú-

Figura 2

cleos algo más pequeños y también con nucleolo prominente. No se observa en la totalidad de la neoplasia, presencia histológica de pigmento melánico. (Fig 2) Con técnicas de inmunohistoquímica, la neoplasia no expresa queratinas y expresa de modo muy intenso CD10 y en menor medida CD68. No expresa Melan A ni HMB45. De modo curioso en la pequeña área con aspecto menos fusocelular y epitelioide, sus células ex87


22º Curso de Dermatopatología

presan S-100 y SOX 10. Estos marcadores son negativos en el resto del tumor. Por el contrario en la zona donde se identifica inmunoexpresión de S-100 y SOX 10, no se observa tinción con CD10. Ante la necesidad de descartar una neoplasia de colisión o una única neoplasia con diferenciación atípica y divergente se realiza estudio estructural tomando muestras para tal estudio de las dos áreas, identificándose en ambas zonas, melanosomas tipo 3 (Fig 3) El espesor de la neoplasia es de 7mms. Diagnóstico MELANOMA NODULAR NO ULCERADO POBREMENTE DIFERENCIADO Comentarios La existencia de patrones peculiares y poco habituales en el seno del melanoma, solo viene a reforzar la vieja idea de que el melanoma puede ser uno de los tumores que en ocasiones puede presentarse como uno de los grandes simuladores. Por ello en casos concretos, debemos asumir inmunoexpresiones poco frecuentes, llevándonos a la necesidad de paneles amplios de inmunohistoquímica para estos casos. El caso presentado, por su rápida evolución, manifestación en zona expuesta y en persona de edad, no recoge el perfil clínico frecuente de un melanoma. Por otro lado, la morfología apoya en más de 2/3 de la neoplasia, el diagnóstico de Fibroxantoma atípico con expresión intensa de CD10 y negatividad para marcadores melanocíticos. Solamente en una pequeña área del tumor sí había inmunoexpresión de marcadores melanocíticos (S100 y SOX10).

Figura 3

Todavía se podría plantear la idea de la colisión de dos tumores. La expresión de BRAF en las células neoplásicas de ambas áreas y el estudio ultraestructural de las dos zonas, con la existencia de melanosomas de tipo 3 en ambas, aclara definitivamente el cuadro.

Diagnóstico final MELANOMA NODULAR NO ULCERADO, POBREMENTE DIFERENCIADO (TIPO PSEUDO FIBROXANTOMA ATÍPICO).

BIBLIOGRAFÍA 1. Díaz-Cascajo C, Weyers W, Borghi S. Pigmented atypical fibroxanthoma. A tumour that may be easily mistaken for malignant melanoma. Am J Dermatopathol 2003;25:1-5. 2. Longacre TA, Smoller BR, Rouse RV. Atypicalfibroxanthoma. Multiple immunohistologic profiles. Am J Surg Pathol 1993;17:1199209. 3. Monteagudo C, Calduch L, Navarro S, et al. CD99 Immunoreactivity in atypical fibroxanthoma. Am J Clin Pathol 2002;117:126-131. 4. Weedon D, Williamson R, Mirza B. CD10, a useful marker for atypical fibroxanthomas. Am J Dermathol 2005;27:181. 5. Sangüeza, Martin MD*; Zelger, Bernhard MD, MSc. Melanoma Simulating Atypical Fibroxanthoma. Am J Dermatopathol 2007; 29,6, 551-554 6. Gao, Zhenqiang MD, PhD; Stanek, Albert MD; Chen, Sheng MD, PhD. A Metastatic Melanoma with an Unusual Immunophenotypic Profile. Am J Dermatopathol 2007; 29, 2: 169-171 7. Maija Kiuru, MD PhD,1 Gregory McDermott, BA,1 Michael Berger, PhD,1,2 Allan C Halpern, MD,3 and Klaus J Busam, MD1. Desmoplastic Melanoma with Sarcomatoid De-differentiation. Am J Surg Pathol. 2014 Jun; 38(6): 864–870. 88


Caso del Seminario 12 Dra. V. Velasco Hospital Univ ersitario de Cruces (Barakaldo - Vizca ya) Universitario Vizcay

Figura 1

Historia Clínica Niña de 4 años de origen boliviano que consultó por la presencia de unas lesiones hipopigmentadas, lenticulares, en la zona anterior del abdomen. Comenzaron a aparecer a los pocos meses de edad, aumentando progresivamente en número. Asintomáticas.

Comentarios La papulosis de células claras es una entidad cuya apariencia clínica suele ser leve, e incluye un diagnóstico diferencial muy variable; hipopigmentación post-inflamatoria, vitíligo, verrugas planas, pitiriasis liquenoide crónica, pitiarisis versicolor. Se inicia en la infancia y es más frecuente en niños de origen asiático. El pronóstico es benigno y aunque se han realizado hipótesis de la relación de estas células con las de Paget, no hay ningún caso descrito de enfermedad de Paget con antecedentes de lesiones en la infancia. La histología puede pasar desapercibida dada la escasez de estas células y su posible confusión con melanocitos; son células claras entre los queratinocitos basales y ocasionalmente suprabasales, de mayor tamaño que los queratinocitos, con citoplasma claro y vacuolado con núcleos pálidos. El diagnóstico diferencial histológico inicial puede incluir melanoma, células de Langerhans, micosis fungoi-

Descripción Microscópica Epidermis de espesor normal, con presencia de algunas células redondas, de citoplasma pálido en las capas basales. Escasa inflamación inespecífica superficial. Las células descritas, cuyo tamaño es mayor al de los queratinocitos, son positivas para PAS y Ck7, siendo negativo HMB45. Diagnóstico: PAPULOSIS DE CÉLULAS CLARAS 89


22º Curso de Dermatopatología

de, enf de Paget, ca. escamoso in situ pagetoide, ca. sebáceo, o disqueratosis pagetoide. Muestran positividad para EMA, CEA, citoqueratinas AE1/AE2, CK7, CAM 5.2 y PAS. BIBLIOGRAFÍA 1. Sim JH, Do JE, Kim YC. Clear cell papulosis of the skin: acquired hypomelanosis. Arch Dermatol. 2011 Jan;147(1):128-9. 2. Kumarasinghe SP1, Chin GY, Kumarasinghe MP. Clear cell papulosis of the skin: a case report from Singapore. Arch Pathol Lab Med. 2004 Nov;128 (11):e149-52. 3. Wysong A1, Sundram U, Benjamin L. Clear-cell papulosis: a rare entity that may be misconstrued pathologically as normal skin. Pediatr Dermatol. 2012 Mar-Apr;29(2):1958. 4. Tseng FW1, Kuo TT, Lu PH, Chan HL, Chan MJ, Hui RC. Long-term follow-up study of clear cell papulosis. J Am Acad Dermatol. 2010 Aug;63(2):266-73.

Figuras 2, 3 y 4 90


Caso del Seminario 13 Dra. V. Velasco Hospital Univ ersitario de Cruces (Barakaldo - Vizca ya) Universitario Vizcay

Figura 1

Historia Clínica Unos hermanos trillizos acuden al servicio de dermatología en diferentes edades (acuden posteriormente, en diferentes años y edades), con lesiones papulares múltiples, algo queratósicas, en tronco, cuello y extremidades desde el nacimiento. Los dos hermanos varones mostraron anomalías oculocutáneas y la hermana únicamente lesiones cutáneas.

Diagnóstico: NEVUS EPIDÉRMICO PAPULAR CON CÉLULAS BASALES EN HORIZONTE «SKYLINE» (PENS) Comentarios El Nevus epidérmico papular con células basales en horizonte tiene un patrón histológico distintivo que lo diferencia del resto de nevs epidérmicos; la disposición de las células basales formando una empalizada en crestas largas y fusionadas. Clínicamente suelen ser lesiones múltiples, que aparecen poco después de nacer o ser congénitas. El rango varía de pápulas a pacas rugosas, entre 0,3 y 3cm, de color marrón-amarillo o blanquecino. Afectan a tronco, cuello y extremidades y pue-

Descripción Microscópica Se realizó biopsia punch de una lesión en pie y una en glúteo, de diferentes pacientes. Se observa hiperqueratosis ortoqueratósica y acantosis sutil, con crestas elongadas y células basales con distribución en empalizada, asemejando un horizonte. 91


22º Curso de Dermatopatología

Figura 2

den tener un patrón lineal. Recientemente se ha descrito la asociación de estas lesiones (50% de los casos) a problemas neurológicos como epilepsia, alteraciones del lenguaje y retraso mental y psicomotor, entre otros. Esto ha llevado a la descripción de un síndrome neurocutáneo: el síndrome PENS, por lo que el diagnóstico histológico de estas lesiones obliga a una exploración exhaustiva en el campo neuromotor. Varios estudios proponen patrones de herencia, pero aún no se ha descrito el gen responsable (descartados genes de otros nevus epidérmicos: FGFR3 y PIK3CA).

Figura 3

3. Rodríguez-Díaz E, Gonzalvo P, Colmenero I, Requena L, Hernández-Martín A, Torrelo A. Papular epidermal nevus with «skyline» basal cell layer (PENS) with extracutaneous findings. Pediatr Dermatol. 2013 Jul-Aug;30(4):e54-6. 4. Luna PC, Panizzardi AA, Martin CI, Vigovich F, Casas JG, Larralde M. Papular Epidermal Nevus with Skyline Basal Cell Layer (PENS): Three New Cases and Review of the Literature. Pediatr Dermatol. 2016 May;33(3):296-300.

BIBLIOGRAFÍA 1. Torrelo A, Colmenero I, Kristal L, Navarro L, Hafner C, Hernández-Martín A, Requena L, Happle R. Papular epidermal nevus with «skyline» basal cell layer (PENS). J Am Acad Dermatol. 2011 May;64(5):888-92. 2. Tadini G, Restano L, Happle R, Itin P. PENS syndrome: a new neurocutaneous phenotype. Dermatology. 2012;224(1):24-30. 92


Caso del Seminario 14 Dr gudo Dr.. C C.. Montea Monteagudo Hospital Clínico Univ ersitario de Valencia. Universitario Historia Clínica Mujer de 39 años con antecedentes de osteopoiquilosis, que acude a la consulta de dermatología por unas lesiones cutáneas que presentan sus dos hijos: un niño de 7 años y una niña de 4 años. El primero tenía como antecedentes: criptorquidia congénita, tortícolis congénita asimetría facial, hipoacusia bilateral, estrabismo, y retraso del lenguaje. Presentaba desde los 2 años unas lesiones sutilmente nodulares de 1 cm de diámetro, ligeramente amarillentas y distribuidas en extremidades superiores e inferiores. En flanco izquierdo varias de estas lesiones confluían formando una placa. Se le realiza una biopsia cutánea del flanco, una radiografía de manos bilateral (que mostró pequeñas lesiones óseas radioopacas, y un estudio genético gen LEMD3 (que mostró una mutación en heterocigosis, una deleción que crea un codón de parada, dando lugar a una proteína truncada de 525 aminoácidos, frente a los 911 de la proteína nativa). La niña presentaba desde hacía un año una lesión cutánea en nalga derecha, además de varias lesiones nodulares menores de 1 cm, distribuidas en tronco y extremidades, parecidas a las que tenía su hermano. A la vista de los hallazgos de sus hijos, se explora a la madre y se aprecia una lesión localizada en muslo derecho, indurada al tacto y de aspecto atrófico, que según ella presentaba desde la adolescencia y que no había consultado nunca, y de la que se toma una biopsia (laminilla remitida para el curso).

Figura 1

con un engrosamiento y alargamiento de las fibras elásticas, que asimismo rodeaban grupos de fibras colágenas. Diagnóstico: PLACAS DE ELASTORREXIS ASOCIADAS AL SINDROME DE BUSCHKE-OLLENDORFF

Descripción Microscópica Histológicamente, la biopsia del niño mostró con la tinción de orceína un aumento en la cantidad de fibras elásticas en todo el espesor de la dermis reticular, apareciendo éstas engrosadas, elongadas y que rodeaban y compartimentalizaban haces de fibras colágenas. La biopsia de la niña mostró una fragmentación generalizada (elastorrexis) de las fibras elásticas en la dermis reticular. La biopsia de la madre mostró unas características similares a las de su hija, con una elastorrexis extensa, pero en un pequeño foco profundo se apreció un incremento de elásticas que, a mayor aumento se correspondía

Comentarios El sindrome de Buschke-Ollendorff (SBO) fue descrito en 1928 y tiene una incidencia estimada de 1/20.000. Esta causada por mutaciones del gen LEMD3 que codifica una proteína de la membrana nuclear interna (MAN1) que interacciona con TGF-beta y BMP (bone morphogenetic protein) antagonizando ambas vías de señalización. Las lesiones óseas se manifiestan después de la infancia como osteopoiquilosis y afectan a huesos largos, así como los de las manos y pies, y son asintomáticas y no tienen riesgo incrementado de fracturas. También puede aparecer melorreostosis, otra displasia 93


22º Curso de Dermatopatología

Figura 2

ósea que se caracteriza por lesiones escleróticas con aspecto de cera derritiéndose. Los miembros de una misma familia pueden presentar de forma variable solo lesiones cutáneas u óseas, o ambas. Se han descrito también otras asociaciones como retraso mental y motor, estenosis espinal, estatura baja, sordera, escoliosis, criptorquidia y estrabismo. Las lesiones cutáneas empiezan en la infancia como pápulas amarillentas o color piel y/o placas, asintomáticas. Aumentan de tamaño y número con la edad, aunque algunas se resuelven espontáneamente, Se trata de nevus de tejido conectivo, siendo los más frecuentes los elastomas, y en menor medida formas combinadas con colagenomas. Son hamartomas de tejido conectivo que constituyen la manifestación más precoz del síndrome, apareciendo en la infancia temprana, en forma de pápulas, nódulos, placas de color piel o amarillentos, de consistencia firme, en espalda, nalgas y piernas, y que son asintomáticas. Se caracterizan por un desequilibrio entre la cantidad y distribución de los componentes de la matriz extracelular (colágeno,

elastina and proteinglicanos). Sin embargo, la fragmentación prominente de las fibras elásticas no es un rasgo del SBO, mientras que sí es característica de otros procesos como la elastorrexis papular y el nevus anelástico, que no forman parte del SBO. BIBLIOGRAFÍA 1. Pope V, Dupuis L, Kannu P, et al. BuschkeOllendorff syndrome: a novel case series and systematic review. Br J Dermatol 2016;174:723-29. 2. Saussine A1, Marrou K, Delanoé P, et al. Connective tissue nevi: an entity revisited. J Am Acad Dermatol. 2012;67:233-9. 3. Uitto J, Santa Cruz DJ, Eisen AZ. Connective tissue nevi of the skin: clinical, genetic, and histopathologic classification of hamartomas of the collagen, elastin, and proteoglycan type. J Am Acad Dermatol 1980;3:441-61 4. Cañueto J, Román C, Santos-Briz A, et al. Papular elastorrhexis and Buschke-Ollendorff syndrome are different entities. J Am Acad Dermatol 2011;65:e7-9. 94


Caso del Seminario 15 Dr gudo Dr.. C C.. Montea Monteagudo Hospital Clínico Univ ersitario de Valencia. Universitario

Figura 1

Historia Clínica Varón de 60 años, con antecedentes familiares (padre) de melanoma, que es remitido desde asistencia primaria por una lesión en la punta nasal con la sospecha clínica de carcinoma basocelular. Se realizó exéresis de la lesión.

alto peso molecular (CK34BE12), y presencia de algún macrófago CD68+ adyacente. No se observó inmunotinción significativa para S-100 ni HMB-45. Diagnóstico NIDOS «PSEUDOMELANOCÍTICOS» EN EL CONTEXTO DE LESIÓN INFLAMATORIA LIQUENOIDE

Descripción Microscópica Se observan escasos y aislados nidos o agregados celulares en la unión dermoepidérmica, alguno de los cuales parece tener morfológicamente una composición celular heterogénea. En la dermis papilar subyacente a los nidos se observan numerosos melanófagos así como signos de remodelación estromal y variable fibrosis. Los nidos descritos son positivos para Melan-A, y se observa en su interior algún queratinocito positivo para citokeratinas de

Comentarios Estas lesiones se desarrollan en áreas de inflamación liquenoide de distintos tipos (queratosis liquenoide, liquen plano pigmentoso, lesiones fototóxicas, entre otras), y con mayor frecuencia en áreas fotoexpuestas. Y por ello, el principal problema consiste en su posible interpretación como mela95


22º Curso de Dermatopatología

Figura 2

noma, especialmente de tipo lentigo maligno. Los agregados celulares o nidos característicamente presentes en la unión dermoepidérmica de estas lesiones no son nidos melanocíticos puros ya que su composición celular es heterogénea, y aunque hay un predominio de células positivas para Melan-A, no son todas ellas verdaderos melanocitos. También es variable la inmunotinción para MiTF, pero no hay que olvidar que MiTF no es específico de melanocitos y puede ser positivo también en macrófagos y mastocitos. En la mayor parte de casos, estas células son negativas para S-100 y HMB-45. Se ha propuesto también la utilidad de la inmunotinción para la adenil-ciclasa soluble para diferenciarlas de las lesiones melanocíticas verdaderas benignas y malignas en función de la presencia y patrón de la inmunotinción, aunque esta propuesta requiere ser confirmada. Algunos autores han propuesto denominar estas lesiones «nidos pseudomelanocíticos», aunque lo cierto que en algunos casos hay un número significativo de melanocitos, y por tanto este término no es siempre apropiado. Por esta razón, también se ha propuesto el término «nidos pseudomelanocíticos ricos en melanocitos» (término aún más confuso y aparentemente contradictorio) o bien el de

«pseudonidos melanocíticos», que es todavía más inapropiado porque si algo es constante e incuestionable es que se trata verdaderamente de nidos. Se recomienda realizar el diagnóstico diferencial con las lesiones melanocíticas verdaderas y en particular con el melanoma en función de varios parámetros como son: las características clinicopatológicas, el contexto morfológico histológico, y los marcadores inmunohistoquímicos, pero no únicamente con estos últimos.

BIBLIOGRAFÍA 1. Beltraminelli H, Shabrawi-Caelen LE, Kerl H, et al. Melan-A-positive «pseudomelanocytic nests»: a pitfall in the histopathologic and immunohistochemical diagnosis of pigmented lesions on sun-damaged skin. Am J Dermatopathol. 2009;31:305-8. 2. Nicholson KM, Gerami P. An immunohistochemical analysis of pseudomelanocytic nests mimicking melanoma in situ: report of 2 cases. Am J Dermatopathol. 2010;32:633-7. 96


3. Silva CY, Goldberg LJ, Mahalingam M,et al. Nests with numerous SOX10 and MiTF-positive cells in lichenoid inflammation: pseudomelanocytic nests or authentic melanocytic proliferation? J Cutan Pathol. 2011;38:797-800. 4. Chung HJ, Simkin AD, Bhawan J, et al. Melanocytic Nests Arising in Lichenoid Inflammation»: Reappraisal of the Terminology «Melanocytic Pseudonests». Am J Dermatopathol. 2015;37:940-3. 5. Hall LD, Bodendorf MO, Najarian DJ, et al. Soluble adenylyl cyclase (sAC) immunostaining distinguishes pseudomelanocytic nests in lichenoid tissue reaction. J Cutan Pathol. 2015;42:232-5. 6. McClanahan D, Choudhary S, Zahniser J, Ho J. Diagnostic pitfalls: pseudomelanocytic nests in the setting of lichenoid inflammation. Actas Dermosifiliogr. 2018. pii: S0001-7310(18)301728.

97


22Âş Curso de DermatopatologĂ­a

98


Caso del Seminario 16 Dr gudo Dr.. C C.. Montea Monteagudo Hospital Clínico Univ ersitario de Valencia. Universitario Historia Clínica Varón de 70 años que acude por presentar lesiones en el pene desde hace 2-3 meses, que asocian ocasional prurito. Refiere molestias e irritación en el surco balanoprepucial y proximal a éste, respetando el glande. Tras las relaciones sexuales se produce despegamiento superficial epitelial del glande, dejando una erosión. Su médico de asistencia primaria le pautó Adventan (presentando empeoramiento tras 2 días de aplicación) y Clotrimazol (ausencia de respuesta). Se aplica por su cuenta Nivea por molestias, y emplea un gel urológico para su higiene. Las lesiones empezaron antes de la aplicación de Nivea. Refiere mejoría desde que ha dejado de emplear las toallas para su secado. Exploración física: Eritema sin exudación ni lesiones pustulosas, localizada en el surco balanoprepucial y proximal a este. En un área se observa un discreto brillo perlado de forma reticulada. En el surco balanoprepucial presenta un área de erosión, secundario a fricción (que apareció esa misma mañana tras una relación sexual). Se trató con aquaphor con mejoría y se añadió suniderma y Vea Olio. En el último control (19/11/ 18) estaba muy bien (en tratamiento con suniderma 2v/día) con resolución del prurito y las molestias. En la exploración física se observó área de aspecto atrófico, pero hidratada y sin leucoplasia.

Figura 1

lizó en dos ocasiones, dio un resultado no valorable debido a la calidad del DNA. Diagnóstico: ACANTOMA DISQUERATÓTICO SOLITARIO DE GLANDE

Descripción Microscópica Histológicamente la lesión se caracteriza por la presencia de una discreta acantosis y moderada hiperqueratosis, destacando la presencia en todos los estratos suprabasales de numerosos queratinocitos disqueratóticos con morfología redondeada, con un citoplasma densamente eosinófilo, siempre de localización perinuclear, mientras que el citoplasma periférico es pálido. No se observa acantólisis ni signos característicos de epidermólisis. En el estrato córneo destaca la presencia de restos de células disqueratóticas a modo de «cuerpos redondos», con presencia de imágenes de halo perinuclear en algunos casos. No se observó inmunotinción para p16 en la lesión, y el estudio por PCR para papilomavirus, que se rea-

Comentarios Se han descrito múltiples tipos de acantomas cutáneos con disqueratosis. La mayoría son acantolíticos (acantoma acantolítico solitario, acantólisis y disqueratosis focal, nevus epidérmico acantolítico y disqueratótico, disqueratosis acantolítica papular de la vulva) o epidermolíticos (acantoma epidermolítico solitario, múltiple o diseminado). El término «acantoma disqueratótico» fue acuñado en 1995 por Roten y Bhawan para denominar una lesión presente en una mujer de 65 años y del que no existe ningún caso adicional en la literatura. Las características histopatológicas de estas 99


22º Curso de Dermatopatología

BIBLIOGRAFÍA 1. Roten SV, Bhawan J. Isolated dyskeratotic acanthoma. A variant of isolated epidermolytic acanthoma. Am J Dermatopathol. 1995;17:63-6. 2. Ko CJ, Barr RJ, Subtil A, et al. Acantholytic dyskeratotic acanthoma: a variant of a benign keratosis. J Cutan Pathol. 2008;35:298-301. 3. Kazlouskaya V, Lambe J, Elston D. Solitary epidermolytic acanthoma. J Cutan Pathol. 2013;40:701-7. 4. Abbas O, Wieland CN, Goldberg LJ. Solitary epidermolytic acanthoma: a clinical and histopathological study. J Eur Acad Dermatol Venereol. 2011;25:175-80. 5. Baliu-Piqué C, Iranzo P. Papular Acantholytic Dyskeratosis of the Vulva in a Woman With Benign Familial Pemphigus.Actas Dermosifiliogr. 2017;108:78-79. 6. Sánchez-Carpintero I, España A, Idoate MA. Disseminated epidermolytic acanthoma probably related to trauma. Br J Dermatol. 1999;141:72830.

lesiones son muy similares a las de los acantomas epidermolíticos. De hecho, en su descripción original, los autores lo consideraron una variante de acantoma epidermolítico, aunque algunos autores posteriores no están de acuerdo con esta interpretación. Sin embargo, en las lesiones epidermolíticas, a diferencia del caso aquí presentado, el citoplasma denso es típicamente periférico, y el perinuclear es pálido, con aspecto vacuolizado, disgregado o ausente. Además, típicamente presenta inclusiones granulares eosinofílicas intracitoplasmáticas. Las formas localizadas de acantoma epidermolítico se localizan con frecuencia en el área genital y con mayor incidencia en varones. Además, aunque los acantomas epidermolíticos y el único caso descrito de acantoma disqueratótico solitario presentan signos histológicos sugestivos de participación de papilomavirus, éstos no se han detectado en los primeros y no se analizaron en el segundo. Se ha considerado que tanto el acantoma epidermolítico y el acantoma disqueratótico solitario pueden tener un origen traumático, algo que parece justificado en nuestro caso.

100


Caso del Seminario 17 Drs. J.J. Ríos-Martín, L. Padilla España. Hospital Virg en de la Macar ena (Se villa). Virgen Macarena (Sevilla).

Figura 1

Historia Clínica Varón de 21 años de edad que consulta por la aparición progresiva, desde hace varios meses, de placas eritemacostrosas pruriginosas e infiltradas a la palpación, sobre las zonas de tinta roja de un tatuaje en el brazo derecho. Ante la falta de respuesta a tratamiento tópico oclusivo con corticoide de alta potencia se realizó biopsia-exéresis con la sospecha clínica de reacción granulomatosa a la tinta roja.

estudio inmunohistoquímico demostró una población similar de linfocitos T CD4 y CD8, mientras que los linfocitos intraepidérmicos eran sobre todo CD8. No existía pérdida de expresión de marcadores de línea T (CD2, CD5 y CD7). Las tinciones especiales para demostrar microorganismos fueron negativas. Diagnóstico REACCIÓN A TATUAJE TIPO MICOSIS FUNGOIDE LIQUENOIDE (PSEUDOLINFOMA)

Descripción Microscópica El estudio histológico objetiva una dermatitis liquenoide con abundantes linfocitos y aisladas células plasmáticas. El infiltrado linfocitario ocupa y expande la dermis papilar y muestra marcado epidermotropismo de linfocitos pequeños e hipercromáticos. La epidermis supralesional está atrófica y muestra discreta hiperqueratosis. En el seno del infiltrado linfocitario era difícil descubrir macrófagos con fagocitosis de restos de tinta del tatuaje. El

Comentarios Se han descrito diferentes patrones inflamatorios como reacción a tatuajes cutáneos: espongiótico, liquenoide, psoriasiforme, granulomatoso y pseudolinfomatoso (1). De todas ellos el patrón liquenoide es el que se observa con mayor frecuencia. Las reacciones pseudolinfomatosas a tatuaje son infrecuentes y generalmente son de predominio B, 101


22º Curso de Dermatopatología

Figura 2

aunque casos con hiperplasia linfoide T también existen. Sin embargo, simulando una micosis fungoide (MF) solo hay una descripción en la literatura revisada (2). En este caso, los linfocitos predominantes eran CD8 lo que pone de manifiesto la participación de estas células efectoras en las reacciones de hipersensibilidad cutáneas. Además de esta peculiar reacción inflamatoria simulando una micosis fungoide, el diagnóstico diferencial clásico de la MF incluye el liquen escleroso y atrófico, las dermatitis de contacto alérgica y los pseudolinfomas inducidos por drogas (anticonvulsivantes, beta-bloqueantes, etc) (3). El inicio de las reacciones inflamatorias a la tinta de los tatuajes varía de días a años, siendo generalmente la tinta de color rojo la responsable. Este caso enfatiza la necesidad de la información clinica para llegar al diagnóstico histológico correcto. La ausencia de pérdida de expresión de marcadores T (CD2, CD5, CD7) apoya que se trata de un proceso reactivo, así como la policlonalidad del RCT (receptor de células T), aunque clonalidad puede detectarse en procesos inflamatorios.

Figura 3

BIBLIOGRAFÍA 1. Thum ChK, Biswas A. Inflammatory complications related to tattooing: a histopathological approach based on pattern analysis. Am J Dermatopathol 2015; 37:54-66. 2. King BJ, Lehman JS, Macon WR, Scialis GF. Red tattoo-related micosis fungoides-like CD8+ pseudolymphoma. J Cutan Pathol 2018; 45:226228 3. Arps DP, Chen S, Fullen DR, Hristow AC. Selected inflammatory imitators of micosis fungoides. Histologic features and utility of ancillar studies. Arch Pathol Lab Med 2014; 138:1319-1327. 102


Caso del Seminario 18 Drs. J.J. Ríos-Mar tín y A.J. Mar cos Domínguez. Ríos-Martín Marcos Hospital Virg en de la Macar ena (Se villa). Virgen Macarena (Sevilla).

Figura 1

Historia Clínica Mujer de 51 años de edad que consulta por la aparición cíclica, unos días antes de la menstruación, de lesiones anulares eritematosas y pruriginosas en el abdomen, desde hace varios meses. Tras la menstruación, desaparecen espontáneamente. Se realiza una biopsia con el juicio clínico de dermatitis autoinmune por progesterona.

merosas células de Langerhans en la dermis. Diagnóstico: DERMATITITS AUTOINMUNE POR PROGESTERONA Comentarios La dermatitis autoinmune por progesterona es una reacción de hipersensibilidad cutánea a las fluctuaciones de la progesterona (PG) endógena durante el ciclo menstrual (1, 2). Tiene un amplio espectro de presentación clínica (urticarias, eccema y erupciones vesiculoampollosas), probablemente en relación con los niveles de progesterona en cada paciente afecto. En una revisión publicada en 2017 (3) los autores describen los hallazgos histológicos en 39 casos

Descripción Microscópica El estudio histológico objetiva una dermatitis espongiótica con presencia de numerosos abscesos intraepidérmicos de células de Langerhans. El infiltrado inflamatorio dérmico superficial es predominantemente linfocitario, aunque con las inmunotinciones para S100 y CD1a se identifican nu103


22Âş Curso de DermatopatologĂ­a

Figuras 2 y 3

104


BIBLIOGRAFÍA 1. Nguyen T, Ahmed AR. Autoimmune progesterone dermatitis: update and insights. Autoimmun Rev 2016; 15:191-197. 2. Buchheit KM, Bernstein JA. Progesterone hypersensitivity: heterogeneous manifestations with a common trigger. J Allergy Clin Immunol Pract 2017;5:566-574. 3. James T, Ghaferi J, LaFond A. The histopathological features of autoinmune progesterona dermatitis. J Cutan Pathol 2017; 44:70-74. 4. Grace SA, Sutton AM, Armbrecht ES et al. P53 is a hepful marker in distinguishing Langerhans cell histiocytois from Langerhans cell hiperplasia. Am J Dermatopathol 2017;39:726-730.

de esta entidad. El cuadro histológico más frecuente es una dermatitis perivascular superficial inespecífica (72%) seguido por una dermatitis de interfase (36%) y una dermatitis intersticial con infiltrado inflamatorio inespecífico (31%). Se describen eosinófilos en el 41% de los casos, neutrófilos en el 21% y espongiosis en el 18%. Por tanto, el diagnóstico se basa sobre todo en la historia clínica de la paciente, ya que los hallazgos histológicos son bastante inespecíficos. En el caso presentado llama la atención los numerosos abscesos de células de Langerhans así como la hiperplasia de estas células en el componente inflamatorio dérmico superficial. Estos abscesos, frecuentes en dermatitis de base alérgica, pueden representar la clave morfológica de una reacción de hipersensibilidad retardada. Si existen dudas sobre si se trata de una hiperplasia o una histiocitosis de células de Langerhans, recientemente se ha descrito la utilidad de la inmunotinción para p53 y CD31, ambas negativas en las hiperplasias (4). Los criterios clínicos diagnósticos de la dermatitis autoinmune por progesterona son los siguientes: - Aparición cíclica de las lesiones en relación con el ciclo menstrual (fase lútea) - Pruebas de sensibilización a la PG mediante tests intradérmicos, provocación oral, intramuscular o intravaginal, o bien mediante la demostración de anticuerpos circulantes anti-PG o test de transformación linfoblástica positivo frente a progesterona. Las pruebas intradérmicas pueden ser falsamente negativas y los anticuerpos se describen en un 66% de los casos. - Interrupción de los brotes inhibiendo la secreción de PG

105


22Âş Curso de DermatopatologĂ­a

106


Caso del Seminario 19 Drs. J.J. Ríos-Martín y R. González Cámpora. Hospital Virg en de la Macar ena (Se villa). Virgen Macarena (Sevilla).

Historia Clínica Mujer de 66 años de edad que consulta por la aparición de un nódulo de lento crecimiento en la piel del antebrazo izquierdo. Se realiza la extirpación de la lesión con el juicio clínico de posible dermatofibroma.

Figura 1

de recidivas locales) descrita en 1998 con diferente terminología («tumor mixohialino inflamatorio de las extremidades distales con virocitos o células de Reed-Sternberg-like», «sarcoma fibroblástico mixoinflamatorio acral», «tumor mixoide inflamatorio de partes blandas con células gigantes bizarras»). La localización puede ser acral o no, y la media de tamaño es de 3 cm. El diagnóstico histológico se basa en la presencia de tres componentes: áreas celulares con células grandes (epitelioides y fusiformes) con pleomorfismo nuclear y nucléolos evidentes, infiltrado inflamatorio (linfocitario y en ocasiones con neutrófilos, eosinófilos, macrófagos y células plasmáticas) y matriz mixoide. La proporción de cada componente varía entre cada caso. Aunque habitualmente la neoplasia es multinodular y se localiza en el tejido celular subcutáneo, también puede afectar a la dermis, motivo por el que el dermatopatólogo debe de estar familiarizado con la misma. El diagnóstico diferencial se plantea con procesos inflamatorios (fascitis proliferativa, infecciones) y tumores mesenquimales mixoides (fibromixoma acral superficial, mixofibrosarcoma superficial). El

Descripción Microscópica El estudio histológico objetivó una neoplasia dérmica circunscrita, aunque no encapsulada, de aspecto mixoide. Estaba constituida por células fusiformes de hábito fibroblástico, con núcleos pleomorfos y nucléolos prominentes, entremezcladas con abundante infiltrado inflamatorio con numerosos eosinófilos. Las células fusiformes fueron positivas con vimentina y CD34. El resto de marcadores inmunohistoquímicos fueron negativos (citoqueratinas, S100, SOX10, melan A, actina de músculo liso, desmina). Diagnóstico SARCOMA FIBROBLÁSTICO MIXOINFLAMATORIO Comentarios El sarcoma fibroblástico mixoinflamatorio (SFM) es un neoplasia locamente agresiva (22% al 67% 107


22Âş Curso de DermatopatologĂ­a

Figuras 2 y 3 108


BIBLIOGRAFÍA 1. Lucas R. Myxoinflammatory fibroblastic sarcoma. Review and update. Arch Pathol Lab Med 2017; 141:1503-1507. 2. Boland JM, Folpe AL. Hemosiderotic fibrolipomatous tumor, pleomorphic hyalinizing angiectatic tumor, and myxoinflammatory fibroblastic sarcoma: related or not?. Adv Anat Pathol 2017; 3. The t(1;10)(p22;q24) TGFBR3/MGEA5 translocation in pleomorphic hyalinizing angiectatic tumor, myxoinflammatory fibroblastic sarcoma, and Hemosiderotic fibrolipomatous tumor. Arch Pathol Lab Med 2019; 143:212-221. 4. Lao W, Yu L, Wang J. Superficial CD34positive fibroblastic tumour: a clinicopathological and immunohistochemical study of an additional series. Histopathology 2017; 70:394-401.

mixofibrosarcoma tiene un patrón multilobular, con borde periférico infiltrativo, y patrón vascular característico (vasos curvilíneos de pared fina), además de actividad mitótica y pseudolipoblastos (no los típicos «virocitos-like»). El fibromixoma acral superficial es una neoplasia de fibroblastos fusiformes CD34 positivos, con vasos prominentes, abundantes mastocitos y ocasionalmente células multinucleadas focales. Una entidad rara, recientemente descrita, es el llamado «tumor fibroblástico superficial CD34 positivo» (4), una neoplasia dérmica, bien circunscrita, de células fusiformes pleomórficas con bajo número de mitosis y ausencia de matriz mixoide. Aunque hay casos con componente inflamatorio, no se ha descrito la matriz mixoide característica del SFM. Hay un debate en la literatura sobre la posible relación entre el SFM, el tumor hemosiderótico fibrolipomatoso y el tumor angiectático hialinizante pleomórfico. Este debate parte de la descripción en estas neoplasias de una translocación común (t(1;10)(p22;q24) TGFBR3/MGEA5); sin embargo, Boland y Folpe (2) opinan que los datos genéticos no son concluyentes y que los SFM con reordenamiento TGFBR3/MGEA5 pueden tratarse de tumores hemosideróticos fibrolipomatosos que han progresado a un sarcoma mixoide.

109


22Âş Curso de DermatopatologĂ­a

110


Caso del Seminario 20 Dra. M. Garrido. Hospital Universitario 12 de Octubre (Madrid).

Figuras 1 y 2

Historia Clínica Mujer de 44 años, natural de filipinas, que consulta por una lesión dolorosa en zona perineal de 1 año de evolución. A la exploración física se observa una placa ulcerada perianal de 5x5 cms. de bordes sobre elevados y fondo eritematoso parcialmente cubierta por pseudo-membranas blanquecinas. Además, presenta edema intenso de labios mayores sin adenopatías inguinales. La serología para VHB/VHC/VIH/Lúes fue negativa.

tentes. El PAS y Grocott resultó negativo. Un cultivo microbiológico practicado demostró mycobacterium tuberculoso. Diagnóstico TUBERCULOSIS CUTÁNEA PERIORIFICIAL (PERINEAL) Comentarios La tuberculosis cutánea es una forma infrecuente de tuberculosis que habitualmente se observa en países en vía de desarrollo o en pacientes inmunodeficientes tras infección por HIV o tras tratamientos inmunosupresores. Su expresividad clínica dependerá del estado inmunológico del paciente, así como de la vía de entrada del bacilo. De esta forma, se pueden clasificar en: a) Por inoculación o contigüidad, entre las que tenemos el Chancro tuberculoso, escrofuloder-

Descripción Microscópica La biopsia vulvar practicada demuestra una ocupación del corion por múltiples estructuras granulomatosas con extensa necrosis central, constituidas por hiostiocitos epitelioides y multinucleados gigantes alrededor de la necrosis caseosa, sin corona linfocitaria. Con la técnica de Ziehl Neelsen se identifican abundantes bacilos ácido alcohol resis111


22º Curso de Dermatopatología

Figura 3

103:929-930 2. Assane K, Oumou NS, Mohamed C, Ndiaye ST, Moussa D, Thierno DM, Bassirou N.. Cutaneous tuberculosis in Dakar: 151 cases report. Mali Med. 2010; 25:14-17 3. Nico MMS, Gavioli CFB, Dabronzo MLD, Romiti R, Takahashi MD, Lourenço SV. The many faces of tuberculosis of the oral mucosa. Three cases with distinct pathomechanisms. J Eur Acad Dermatol Venereol 2017 Dec. 4. Gupta B, Shree S, Rajaram S, Goel N. Genital tuberculosis: Unusual presentations. Int J Mycobacteriol. 2016;5(3):357-359 5. Goyal BK, Pradhan S, Jose T, Duggal BS, Sengupta P. Woody vulva: an unusual presentation of vulvar tuberculosis. J Obstet Gynaecol India. 2014; 64(Suppl 1):85-87 6. Kulchavenya E, Dubrovina S. Typical and unusual cases of female genital tuberculosis. IDCases. 2014; 31:92-4 7. Nanjappa V, Suchismitha R, Devaraj HS, Shah MB, Anan A, Rahim SN. Vulval tuberculosis - an unusual presentation of disseminated tuberculosis. J Assoc Physicians India. 2012;60:49-52 8. Mondal SK. Histopathologic analysis of female genital tuberculosis: a fifteen-year retrospective study of 110 cases in eastern India. Turk Patoloji Derg. 2013;29:41-5

ma, tuberculosis periorificial, tuberculosis miliar, goma tuberculoso, tuberculosis verrucosa y lupus vulgar); b) Por diseminación hematógena (Tuberculosis miliar, goma tuberculoso y lupus vulgar). La tuberculosis cutánea periorificial es una forma de tuberculosis por contigüidad y constituye el 2% de las tuberculosis cutáneas. Se manifiesta en forma de pápulas o nódulos dolorosos en piel, mucosas oral, nasal o anogenital, que progresan rápidamente a úlceras de bordes escavados y fondo granular. Las lesiones se localizan próximas a los orificios naturales por los que se eliminan bacilos de una infección tuberculosa primaria activa (pulmonar, urogenital o intestinal). Nuestra paciente de hecho, presentaba una tuberculosis renal detectándose en los urocultivos micobacterias. El diagnóstico debe hacerse mediante la correlación clínico-patológica, siendo básico el estudio histopatológico y el cultivo microbiológico. El diagnóstico diferencial se debería establecer con otros procesos ulcerativos genitales como sífilis, herpes, linfogranuloma venéreo, úlcera de Lipschütz, o pioderma gangrenoso. BIBLIOGRAFÍA 1. Jimenez-Gallo, Navas-García N, AlbarránPlanelles C, Guerrero-Sánchez F. Tuberculosis periorificial vulvar. Acta Dermosifiliogr. 2012; 112


Caso del Seminario 21 Dra. M. Garrido. Hospital Universitario 12 de Octubre (Madrid). Historia Clínica Mujer de 53 años sin antecedentes personales de interés, que acude al Servicio de Urgencias por placas púrpuricas intensamente dolorosas que se distribuyen en MMII, abdomen bajo, zona posterior de brazos y mama izquierda. No presenta lesiones en mucosas. Con la sospecha clínica de vasculopatía/vasculitis generalizada secundaria probablemente a tóxicos vs postinfeccioso, se realiza una biopsia cutánea. Las lesiones progresan afectando la práctica totalidad de MMII, cara posterior de MMSS, abdomen, y mama izquierda y aparecen ampollas en la zona posterior de MMII. Se acompaña de una plaquetopenia severa de hasta 50x1000/¼l. En el análisis de orina se detectan metabolitos de cocaina y de cannabis Descripción Microscópica La biopsia cutánea demuestra una vasculopatía oclusiva con trombos de fibrina intravascular tanto en la dermis papilar como reticular alta y media, sin signos de vasculitis y sin necrosis epidérmica. Diagnóstico VASCULOPATÍA TROMBÓTICA ASOCIADA A COCAINA ADULTERADA CON LEVAMISOL Comentarios Desde 2005, la cocaína es muy frecuentemente cortada con levamisol (70-90% en USA), que potencia su efecto y aumenta su volumen. Es un polvo blanco de bajo coste, que aumenta los efectos estimulantes de la cocaína al aumentar la dopamina ce113


22Âş Curso de DermatopatologĂ­a

Figuras 2 y 3 114


BIBLIOGRAFÍA 1. John S, Manda S, Hamrock D. Cocaine-induced thrombotic vasculopathy. Am J Med Sci. 2011;342:524-6.. 2. Dartevel A, Chaigne B, Moachon L, Grenier F, Dupin N, Guillevin L, Bouillet L, Mouthon L. Levamisole-induced vasculopathy: A systematic review. Semin Arthritis Rheum. 2018. pii: S00490172(18)30254-3. 3. Tran H, Tan D, Marnejon TP. Cutaneous Vasculopathy Associated with Levamisole-Adulterated Cocaine. Clin Med Res. 2013 ;11(1):26-30. 4. Abdul-Karim R, Ryan C, Rangel C, Emmett M. Levamisole-induced vasculitis. Proc (Bayl Univ Med Cent). 2013 Apr;26(2):163-5.

rebral. El levamisol es un antihelmíntico, clásicamente utilizado en veterinaria. Durante mucho tiempo (década de 1970), se usó como un inmunomodulador en el síndrome nefrótico y para el cancera de colon. Sin embargo, se retiro en el año 2000, porque producía la aparición de efectos adversos graves: agranulocitosis (3%-10%), neutropenia, nefropatía y vasculopatía. Aunque es extensamente conocido que la cocaina tiene manifestaciones cutáneas tipo vasculitis, aparte de sus conocidas complicaciones cardiovasculares (infarto agudo de miocardio y ACV), el levamisol es cada vez más reconocido como el responsable de las erupciones cutáneas inducidas por la cocaina. Dichas lesiones cutáneas pueden ser muy variables, desde inespecíficas, lesiones liquenoides, exantema fijo, vasculitis, pápulas purpúricas, ampollas hemorrágicas e incluso destrucción de la línea media. De forma característica existe el síndrome de vasculopatía asociada a cocaina-levamisol que clínicamente se presenta con lesiones purpúricas retiformes, con una especial predilección por las orejas. La histología puede mostrar varias imágenes: desde vasculitis leucocitoclástica y vasculitis trombótica a vasculopatía oclusica sin vasculitis verdadera, como en nuestro caso. El síndrome de vasculopatía asociada a cocaina-levamisol se suele asociar a anomalías inmunológicas únicas. La más importante es la positividad ANCA, que supone un reto diagnóstico importante para distinguirla de una vasculitis sistémica idiopática verdadera.

115


22Âş Curso de DermatopatologĂ­a

116


Caso del Seminario 22 Dr ernández Flor es. Dr.. A. F Fernández Flores. Hospital del Bierz o (P onf er rada - León). Bierzo (Ponf onfer errada Historia Clínica Mujer que empezó a perder pelo a los 30 años, especialmente en zonas frontal y vertex, acentuándose hasta el estado actual. Esta pérdida ha sido asintomática salvo ocasionales episodios de tricodinia. La pérdida llegó a alcanzar el 70 % de su cabello, según la estimación subjetiva de la paciente. Decide someterse a un trasplante capilar con resultado inicial satisfactorio y sin incidencias. Sin embargo, en el curso de un año, ha perdido la mayor parte del cabello implantado por lo que acude a consulta. Al examen físico, muestra una alopecia marcada con pérdida de cabello en zonas frontal y de vertex. Se realiza tricograma y tricoscopia, evidenciando un 27% de cabellos miniaturizados y un 21% de cabellos en telogen así como pérdida de orificios foliculares con eritema peripilar. Se piden pruebas de laboratorio para hierro, ferritina, TSH, vitaminas B12 y D, ANA y prolactina, siendo todos los valores normales. Se decide hacer biopsia.

Figuras 1 y 2

Descripción Microscópica La biopsia muestra una alopecia con miniaturización e incremento de la relación telo/ catagénica (fig. 1), acompañada de un infiltrado liquenoide en muchos de los folículos (fig. 2), preferente afectando el infundíbulo, istmo y tercio medio folicular, sin afectación de los bulbos. El infiltrado inflamatorio es linfohistiocitario con lesión de interfase (fig. 3). Se acompaña de fibrosis concéntrica perifolicular con pérdida subsecuente de fibras elásticas (fig. 4). Estos hallazgos son los de una alopecia liquenoi117


22Âş Curso de DermatopatologĂ­a

Figuras 3 y 4 118


de fibrosante afectando a folículos vellosos.

sutiles: pérdida del pelo de las cejas, pequeñas cicatrices salpicadas… La importancia de este diagnóstico en el contexto actual en el que el trasplante capilar se ha vuelto algo cotidiano, radica que un diagnóstico de AFPAGA es una contraindicación para trasplante ya que el patrón liquenoide con fibrosis cicatricial se reproducirá en los nuevos folículos trasplantados, conduciendo poco después del trasplante a un rechazo con fibrosis residual.

Diagnóstico ALOPECIA FIBROSANTE DE PATRÓN ANDROGENÉTICO. Comentarios La alopecia fibrosante de patrón androgenético (AFPAGA) es una forma de alopecia de reconocimiento relativamente reciente: descrita por primera vez en 2000 por Zinkernagel.1 Su importancia radica en su presentación clínica imitando una alopecia androgenética tradicional. Sin embargo, a diferencia de esta última, se trata de una alopecia fibrosante, es decir cicatricial, con destrucción del bulge y por lo tanto con incapacidad de regeneración del folículo piloso y pérdida definitiva del mismo.1 Desde el punto de vista conceptual, se discute si esta forma sería una alopecia frontal fibrosante de distribución en áreas androgenéticas o bien una forma fibrosante de una alopecia androgenética. En la biopsia, el infiltrado liquenoide prominente así como la fibrosis, distinguen a este tipo de alopecia de una forma androgenética común. De igual manera, se pensaba que la presencia de miniaturización junto con la afectación de folículos vellosos por el infiltrado inflamatorio, diferenciaba a esta alopecia de una alopecia frontal fibrosante de presentación tradicional frontotemporal. Sin embargo, sabemos ahora que las formas convencionales de alopecia frontal fibrosante con presentación clínica en la línea de implantación presentan miniaturización del folículo así como afectación de folículos vellosos por el infiltrado inflamatorio.2 De este modo, el solapamiento entre estas entidades es mayor de lo que habíamos interpretado en un principio. El concepto unificador actual es incluir bajo la nomenclatura de «foliculitis liquenoide» todas aquellas entidades que se presentan con pápulas foliculares y alopecia cicatricial,3 mostrando una histología de vacuolización de interfase como daño por el infiltrado linfocitario. Bajo este epígrafe se incluirían el liquen planopilaris, el Graham-Little-Piccardi-Lassueur, la alopecia frontal fibrosante, la queratosis pilaris atrófica/uleritema ofriogenes, la atrofoderma vermiculata, y la queratosis folicular espinulosa decalvante. Todas estas entidades presentan solapamiento clínico e histopatológico aunque el diagnóstico diferencial se establece mediante caracteres clínicos

BIBLIOGRAFÍA 1. Zinkernagel MS, Trüeb RM. Fibrosing alopecia in a pattern distribution: patterned lichen planopilaris or androgenetic alopecia with a lichenoid tissue reaction pattern? Arch Dermatol. 2000;136(2):205-211. 2. Miteva M, Sabiq S. A New Histologic Pattern in 6 Biopsies From Early Frontal Fibrosing Alopecia. Am J Dermatopathol. 2018. 3. Turegano MM, Sperling LC. Lichenoid folliculitis: A unifying concept. J Cutan Pathol. 2017.

119


22Âş Curso de DermatopatologĂ­a

120


Caso del Seminario 23 Dr ernández Flor es. Dr.. A. F Fernández Flores. Hospital del Bierz o (P onf er rada - León). Bierzo (Ponf onfer errada

Figuras 1 y 2

Historia Clínica Varón de 69 años con varios ingresos en el último año por un cuadro neurológico de inestabilidad, nauseas, visión doble y desviación de la comisura bucal con empeoramiento progresivo. Resonancia con lesión captante en tronco (puente) de aspecto desmielinizante inflamatoria versus gliomatosa tumoral. Muestra lesiones papulo-nodulares eritematosas crónicas en antebrazo, una de las cuales se biopsia.

Diagnóstico: DERMATITIS EN EMPALIZADA GRANULOMATOSA Y NEUTROFÍLICA ASOCIADA A NEUROBEHÇET CON MANIFESTACIONES NEURONALES DE TIPO TROMBÓTICO PREVIAS A LOS SIGNOS CLÁSICOS DE ENFERMEDAD DE BEHÇET Comentarios La dermatitis en empalizada granulomatosa y neutrofílica (DEGN) es un patrón de reacción tisular necrobiótica que se encuentra en el contexto de enfermedad sistémica.1 Lo más importante a recordar es que la enfermedad sistémica está casi siempre presente. La definición exacta del patrón no está exenta de dificultades por distintos motivos: - Primero por la amplia nomenclatura con la que se ha aludido a esta entidad en la literatura: dermatitis granulomatosa intersticial con artritis, pápulas reumatoides, necrobiosis reumatoide superficial ulcerativa, granuloma necrotizante extravascular cutáneo y granuloma de Churg-Strauss. - Segundo por la variedad morfológica del patrón, que puede simular el granuloma anular o la necrobiosis lipoídica, haciendo que incluso algunos consideren a esta entidad dentro del espectro

Descripción Microscópica La biopsia mostraba un área de necrobiosis degenerativa dérmica (Fig. 1) representada por colágeno degenerado de coloración basófila y aspecto delustrado (Fig. 2) con evidencia de polimorfonucleares neutrófilos entremezclándose con estas áreas (Fig. 3). No había vasculitis. En el estudio con azul alcián no existía depósito de mucina en estas zonas. En la periferia de esta zona central, una corona histiocitaria se disponía en empalizada (Fig. 4). Evolución del paciente El paciente evolucionó desfavorablemente a estado comatoso y exitus, demostrando en el estudio necrópsico una vasculitis de pequeño vaso en el tronco del encéfalo con un infarto hemorrágico del tronco. 121


22Âş Curso de DermatopatologĂ­a

Figuras 3 y 4 122


del granuloma anular. Histológicamente, esta variedad de patrones puede obedecer a distintos estadios evolutivos en las lesiones, comenzando inicialmente con un cuadro de vasculitis de vaso pequeño de tipo leucocitoclástico como resultado del cual se producirían las lesiones necrobióticas con ulterior ordenamiento de los histiocitos en un patrón en empalizada en torno a estos focos de degeneración conectiva. Los estadios más avanzados presentan áreas de fibrosis. Varios tipos de lesiones pueden coexistir en un mismo paciente.2 - Tercero por las muchas entidades sistémicas que han sido asociadas a este patrón tisular, entre las que se encuentran la artritis reumatoide, el lupus eritematoso, el síndrome de Sjögren, el díndrome de Churg-Strauss, la crioglobulinemia mixta, la esclerosis sistémica, la tiroiditis, el síndrome de Raynaud, la enfermedad inflamatoria intestinal, la enfermedad de Still del adulto, las hepatitis, distintos trastornos linfoproliferativos, diversas vasculitis, los síndromes mielodisplásicos, el síndrome urémico hemolítico, la púrpura trombótica trombocitopénica, varios tipos de carcinomas, la diabetes, reacciones a distintos fármacos (sobre todo sulfonamidas) y diversas infecciones bacterianas o virales). No encontrar una enfermedad sistémica es más la excepción que la norma. - Cuarto -como es de esperar- por la variedad de presentaciones clínicas bajo las que esta entidad se manifiesta, incluyendo pápulas o nódulos (menos frecuentemente placas) en un patrón más o menos lineal, a veces confluyendo en auténticas bandas lineales (el signo de la cuerda) pero también con presentaciones urticariformes o placas únicas anulares. De todos estos trastornos, la asociación con el Behçet es rara y sólo ocasionalmente se ha descrito en la literatura.3 Por el contrario, la enfermedad de Behçet suele asociarse a eritema nodoso, tromboflebitis o foliculitis. La enfermedad de Behçet cursa con afectación de varios órganos entre los que se encuentra el sistema nervioso central (SNC). A esta última condición se la conoce como neurobehçet y ocurre raramente (menos del 5% de los casos). Más raro todavía es que un neurobehçet se presente en el momento del diagnóstico del Behçet y mucho más raro es que un neurobehçet preceda a las manifestaciones clásicas de uveítis o de aftas orales. En la afectación del neurobehçet, hay alteraciones de dos grandes grupos: las afectaciones parenquimatosas del tipo rombencefalitis y mielitis, y

las secundarias a episodios trombóticos arteriales o venosos. Las últimas representan sólo el 20% de las manifestaciones descritas en caso de neurobehçet.4 Se piensa que las manifestaciones del primer grupo podrían estar causadas por vasculitis por inmunocomplejos mientras que las del segundo, podrían ser debidas a disfunción endotelial primaria. Para concluir, se debe sospechar un neurobehçet en aquellos casos que se presenten como meningoencefalitis linfocitaria aséptica o trombosis venosa cerebral sin ningún factor procoagulante subyecente.

BIBLIOGRAFÍA 1. Rosenbach M, English JC. Reactive Granulomatous Dermatitis: A Review of Palisaded Neutrophilic and Granulomatous Dermatitis, Interstitial Granulomatous Dermatitis, Interstitial Granulomatous Drug Reaction, and a Proposed Reclassification. Dermatol Clin. 2015;33(3):373387. 2. Kalen JE, Shokeen D, Ramos-Caro F, Motaparthi K. Palisaded neutrophilic granulomatous dermatitis: Spectrum of histologic findings in a single patient. JAAD Case Rep. 2017;3(5):425-428. 3. Kim SK, Park CK, Park YW, Jun JB, Yoo DH, Bae SC. Palisaded neutrophilic granulomatous dermatitis presenting as an unusual skin manifestation in a patient with Behçet’s disease. Scand J Rheumatol. 2005;34(4):324-327. 4. Akman-Demir G, Serdaroglu P, Tasçi B. Clinical patterns of neurological involvement in Behçet’s disease: evaluation of 200 patients. The Neuro-Behçet Study Group. Brain. 1999;122 ( Pt 11):2171-2182.

123


22Âş Curso de DermatopatologĂ­a

124


Caso del Seminario 24 Dra. I. Colmenero. Hospital del Niño Jesús (Madrid).

Figura 1

Historia Clínica Niña de 9 años que consulta por presentar una lesión adquirida de 4 años de evolución y lento crecimiento adyacente a comisura labial derecha. En la exploración física se apreciaba una pápula violácea de 0,5 cm de diámetro, de bordes bien definidos y consistencia semisólida, ligeramente depresible. Con la sospecha de tumor vascular se realiza extirpación de la lesión. La paciente permanece asintomática, sin signos de recidiva tras 8 meses de seguimiento.

Diagnóstico HEMANGIOMA PAPILAR (HP) Comentarios Aunque la mayoría de los hemangiomas cutáneos pediátricos presentan una clínica, evolución e histología características, existen tipos histológicos infrecuentes difíciles de diagnosticar clínicamente. El hemangioma papilar corresponde a una de estas lesiones de difícil diagnóstico. El HP se ha incluido recientemente en la nueva clasificación de la ISSVA y se considera un tumor vascular benigno. El HP es una variante muy infrecuente de hemangioma que se presenta típicamente en cara y cuello, con predilección por varones. La mayoría de las lesiones se presentan en la edad adulta, con una mediana de 57 años, pero existen casos descritos en la infancia. La histología es característica y es el único método de confirmación diagnóstica de certeza. La mayoría de las lesiones muestran un crecimiento predominantemente intravascular hacia el interior de vasos dérmicos dilatados y formación de papilas con ejes celulares constituidos por células estro-

Decsripción Microscópica La biopsia muestra una lesión dérmica profunda constituida por grandes lóbulos bien definidos compuestos por capilares ramificantes y formación de estructuras papilares con revestimiento endotelial en tachuela. Se observan ocasionales inclusiones intracitoplasmáticas globulares eosinofílicas endoteliales. Las células endoteliales son reactivas para marcadores panendoteliales y negativas para marcadores endoteliales linfáticos y GLUT1. 125


22º Curso de Dermatopatología

BIBLIOGRAFÍA 1. Suurmeijer AJ, Fletcher CD. Papillary haemangioma. A distinctive cutaneous haemangioma of the head and neck area containing eosinophilic hyaline globules. Histopathology. 2007 Nov;51(5):638:48. 2. Ide F, Mishima K, Saito I. Papillary hemangioma on the face. J Cutan Pathol. 2009 May;36(5):601-2

males y pericitos. La superficie de las papilas está revestida por células endoteliales tumefactas que pueden contener glóbulos hialinos intracitoplasmáticos. La ME ha demostrado que estos glóblulos están compuestos por lisosomas gigantes que contienen detritos de organelas y vacuolas de grasa (tanatosomas). El HP suele presentar un comportamiento benigno y la exéresis simple es el tratamiento de elección. No se han descrito metástasis ni recurrencias, salvo un caso recurrente de comportamiento no agresivo.

126


Caso del Seminario 25 Dra. I. Colmenero. Hospital del Niño Jesús (Madrid).

Figura 1

Historia Clínica Niño de 3 años con padres sanos no consanguíneos de ascendencia mexicana. Fue visto en su tercer día de vida por presentar múltiples lesiones pustulosas de diferentes tamaños sobre una base eritematosa, en cara y parte superior del tronco. Esta erupción se resolvió en una semana sin dejar cicatrices. Dos meses más tarde, desarrolló lesiones generalizadas que afectaban el tronco, las extremidades superiores e inferiores, escroto y área perianal. Estas lesiones comenzaron como grupos de pápulas y pseudovesículas redondeadas de color amarillo-rosa y evolucionaron en pocos días a placas escamosas, infiltradas y de color marrón rojizo de tamaño entre 0,5 y 30 mm. Se decide realizar biopsia de una de las lesiones. Además de la clínica cutánea, el paciente presentaba un llamativo eritema conjuntival no puru-

lento. A los 5 meses de vida, el paciente comenzó a presentar episodios recurrentes de diarrea y a los 10 meses de vida presentó crisis convulsivas en relación con una vasculitis cerebral. Las pruebas de laboratorio fueron normales o negativas, excepto por una leve neutropenia y niveles séricos disminuidos de inmunoglobulinas. La sospecha inicial fue el síndrome de Blau, pero las pruebas moleculares para el gen NOD2 no detectaron mutaciones. Se detectó, sin embargo una mutación patogénica heterocigota de PLCG2 en el paciente, pero no en los padres, compatible con el diagnóstico de síndrome APLAID. El paciente tiene actualmente 3 años de edad y está siendo tratado con levetiracetam (25 mg / kg / día), infliximab, Ig IV (450 mg / kg bimestral) y metotrexato (2,5 mg por semana). Se mantiene 127


22º Curso de Dermatopatología

Figura 2

estable, con un retraso en el desarrollo y un leve retraso psicomotor, y continúa sufriendo infecciones leves y lesiones inflamatorias ocasionales en la piel.

deficiency and immune dysregulation) es una enfermedad autoinflamatoria monogénica (AID) extremadamente rara y recientemente descrita, caracterizada por lesiones inflamatorias estériles de inicio temprano en la piel, articulaciones y ojos, así como infecciones bacterianas recurrentes debidas a deficiencia en la producción de anticuerpos (Zhou 2012). Hasta el momento, solo se han notificado 5 casos de APLAID (Zhou 2012, Torrelo 2007, Neves 2018). Esta enfermedad fue descrita por primera vez en 2007 por Torrelo et al con el nombre de «dermatitis granulomatosa y neutrófila perforante del recién nacido». En 2012, Zhou et al encontraron mutaciones patogénicas de PLCG2 en dos pacientes (un padre y su hija) con lesiones cutáneas de inicio temprano, neumonitis intersticial, artralgias, inflamación ocular, enterocolitis, celulitis e inmunodeficiencia; estos autores propusieron el acrónimo APLAID. Los pacientes con APLAID presentan una erupción vesiculopustulosa en el período neonatal que se resuelve espontáneamente en las primeras semanas de vida. Más tarde presentan episodios recurrentes de pápulas eritematosas discretas que se convierten en placas compuestas por vesículas y pústulas agrupadas. Con el tiempo desarrollan gran-

Descripción Microscópica La biopsia de una de las placas muestra un denso infiltrado histiocitario intersticial dérmico, que en algunas áreas forma granulomas rodeando áreas de necrobiosis. Aunque la mayoría de los histiocitos son mononucleados, también se observa una significativa cantidad de células gigantes multinucleadas. Además se aprecian agregados densos de neutrófilos en la dermis con fenómenos de cariorrexis. Llama la atención la eliminción transfolicular de material necrobiótico y restos celulares. Las tinciones para BAAR, bacterias, hongos y espiroquetas fueron todas negativas. Diagnóstico DERMATITIS GRANULOMATOSA Y NEUTROFÍLICA en el contexto del SINDROME APLAID Comentarios El síndrome APLAID (autoinflammation and phospholipase Cγ2 (PLC-γ2)-associated antibody 128


Figura 3

des ampollas hemorrágicas que se ulceran y dejan cicatrices cribriformes. Las lesiones aparecen en brotes y suelen empeorar con el calor y la exposición al sol (Zhou 2012, Torrelo 2007). La dermatopatología del síndrome APLAID es tan típica que puede permitir el diagnóstico. Las dos características principales incluyen un infiltrado neutrofílico denso y un inflitrado histiocitario de patrón intersticial con formación de granulomas necrobióticos. En ocasiones se aprecia un intenso edema que puede conducir a ampollas subepidérmicas. Pueden verse células gigantes multinucleadas, cariorrexis y vasculitis leucocitoclástica. El material necrobiótico a menudo sufre eliminación transepidérmica a través de los folículos pilosos (Zhou 2012, Torrelo 2007). Las manifestaciones sistémicas no cutáneas incluyen neumonitis intersticial inespecífica con bronquiolitis respiratoria, infecciones sinusales y pulmonares, artralgias, inflamación ocular y enterocolitis con dolor abdominal recurrente y diarrea sanguinolenta. Nuestro paciente presentó una vasculitis del SNC que no había sido descrita previamente en estos pacientes, pero sí en otros síndromes autoinflamatorios. Considerando que el número de pacientes reportados con síndrome APLAID es muy pequeño, parece razonable pensar

que el espectro general de las manifestaciones clínicas de esta enfermedad no se haya dilucidado por completo. BIBLIOGRAFÍA 1. Torrelo A, Vera A, Portugués M, de Prada I, Sanz A, Colmenero I, Zulaica A, de Lucas R, Fraga J, Pedraz J, Fontán S, Zambrano A. Perforating neutrophilic and granulomatous dermatitis of the newborn—a clue to immunodeficiency. Pediatr Dermatol. 2007 May-Jun;24(3):211-5. 2. Zhou Q, Lee GS, Brady J, Datta S, Katan M, Sheikh A, Martins MS, Bunney TD, Santich BH, Moir S, Kuhns DB, Long Priel DA, Ombrello A, Stone D, Ombrello MJ, Khan J, Milner JD, Kastner DL, Aksentijevich I. A hypermorphic missense mutation in PLCG2, encoding phospholipase C³2, causes a dominantly inherited autoinflammatory disease with immunodeficiency. Am J Hum Genet. 2012 Oct 5;91(4):713-20. 1. Neves JF, Doffinger R, Barcena-Morales G, Martins C, Papapietro O, Plagnol V, Curtis J, Martins M, Kumararatne D, Cordeiro AI, Neves C, Borrego LM, Katan M, Nejentsev S. Novel PLCG2 Mutation in a Patient With APLAID and Cutis Laxa. Front Immunol. 2018 Dec 14;9:2863. 129


22Âş Curso de DermatopatologĂ­a

130


Caso del Seminario 26 Dr Dr.. A. Santos Briz. Hospital Clínico Universitario de Salamanca.

Figura 1

Historia Clínica Mujer de 64 años que acude por lesión redonda, engastada, de color eritemato-violáceo localizada en cara lateral de brazo derecho de muchos años de evolución (Fig.1). Desde hacía unos meses notaba pinchazos en la zona con molestias. Se realizó biopsia que mostró los hallazgos típicos de un dermatofibroma celular. Seis meses derpués acudió al dermatólogo por presentar molestias en la zona. En ese momento se evidencia la cicatriz de la intervención y bajo ella un nódulo subcutáneo. Con la sospecha de recidiva se realizó una nueva exéresis.

celular elevada. No se observaba actividad mitótica. La lesión se extiendía infiltrando la dermis cercana disecando haces de colágeno, constituyendo nidos de colágeno «queloideo». Alcanzaba focalmente el margen lateral de la pieza (Fig.2). La segunda biopsia mostraba una tumoración mesenquimal que se extiendía desde la dermis superficial ocupando todo el espesor de la pieza, hacia la dermis profunda y el tejido celular subcutáneo (Fig.3). Presentaba un crecimiento infiltrativo, disecando haces de colágeno y adipocitos aislados (Fig.4). Las células presentaban fundamentalmente una morfología fusiforme y se disponían en haces entrelazados, ocasionalmente estoriformes. Tenían núcleos de tamaño algo aumentado, con cromatina grumosa, observando ocasionales figuras mitóticas. Los citoplasmas eran eosinófilos fusiformes. En algunas áreas las células adquiríann un aspecto más fibrohistiocítico o ligeramente epitelioide, con núcleos de características similares (Fig.5). La lesión alcanzaba ampliamente los márgenes de resección. El estudio inmunohistoquímico demostró ne-

Descripción Microscópica La primera biopsia mostró una lesión nodular de bordes imprecisos localizada en la dermis superficial media y profunda extendiéndose hacia el tejido celular subcutáneo. Estaba constituida por una mezcla de células fusiformes dispuestas en haces entrelazados, junto a otras de naturaleza histiocitoide con citoplasmas amplios espumosos. Ambos tipos celulares presentaban núcleos ovoideos sin atipia llamativa, identificando una densidad 131


22º Curso de Dermatopatología

Figura 2

gatividad de las células para CD34, con positividad frecuente aunque no difusa para desmina (no presente en la biopsia previa) y calponina, con una tinción irregular difícilmente valorable para actina y caldesmon. Las células eran negativas para Myo D1. Existía negatividad para SOX10 y proteína S100. En áreas profundas se identificaba una tinción de elementos celulares aislados para CAM5.2 y AE1-AE3. Tras el diagnóstico de transformación maligna de dermatofibroma celular, la paciente fu operada mediante cirugía de Mohs en otro centro, que requirió varios pases. En el preoperatorio que realiza, evidencian nódulos pulmonares filiados como metástasis, motivo por lo que se indica quimioterapia que completa. En el momento actual la paciente está viva con buen estado de salud.

Figura 3

tre otros el DF epitelioide, en empalizada, el DF atrófico, el osificante, con células gigantes de tipo osteoclástico, lipidizados ‘de tipo tobillo», queloideo, mixoide, plexiforme, de células granulares, con diferenciación miofibroblastica, o con glóbulos eosinofílicos intracitoplasmáticos. Es importante saber que, desde un punto de vista clínico, el dermatofibroma celular 2, el dermatofibroma aneurismático3 y el dermatofibroma4 atípico, así como los dermatofibromas que surgen en la cara5 y en los tejidos blandos subcutáneos y profundos1 tienen un mayor riesgo de recidiva local (hasta 20%) en contraste con la forma convencional. En los últimos años, se han descrito algunos casos aislados y un par de series de dermatofibromas aneurismáticos, celulares, así como de dermatofi-

Diagnóstico TRANSFORMACIÓN MALIGNA DE DERMATOFIBROMA CELULAR CON EXTENSIÓN METASTÁSICA. Comentarios El dermatofibroma (DF) o histiocitoma fibroso representa una neoplasia mesenquimal benigna muy frecuente en la piel, que raramente puede ocurrir en tejidos blandos subcutáneos y profundos.1 A lo largo de los años, se han descrito distintas variantes morfológicas, y su conocimiento es importante para el diagnóstico diferencial con otras neoplasias más agresivas. Estas variantes incluyen en132


Figuras 4 y 5

BIBLIOGRAFÍA 1. Gleason BC, Fletcher CDM. Deep ‘benign’ fibrous histiocytoma: clinicopathologic analysis of 69 cases of a rare tumor indicating occasional metastatic potential. Am J Surg Pathol 2008;32:354– 362. 2. Calonje E, Mentzel T, Fletcher CDM. Cellular benign fibrous histiocytoma. Clinicopathologic analysis of 74 cases of a distinctive variant of cutaneous fibrous histiocytoma with frequent recurrence. Am J Surg Pathol 1994;18:668–676. 3. Calonje E, Fletcher CDM. Aneurysmal benign fibrous histiocytoma: clinicopathological analysis of 40 cases of a tumour frequently misdiagnosed as a vascular neoplasm. Histopathology 1995;26:323–331. 4. Kaddu S, McMenamin ME, Fletcher CDM. Atypical fibrous histiocytoma of the skin. Clinicopathologic analysis of 59 cases with evidence of infrequent metastasis. Am J Surg Pathol 2002;26:35–46. 5. Mentzel T, Wiesner T, Cerroni L, Hantschke M, Kutzner H, Rütten A, Häberle M, Bisceglia M, Chibon F, Coindre JM. Malignant dermatofibroma: clinicopathological, immunohistochemical, and molecular analysis of seven cases. Mod Pathol. 2013;26:256-67. 6. Doyle LA, Fletcher CD. Metastasizing «benign» cutaneous fibrous histiocytoma: a clinicopathologic analysis of 16 cases. Am J Surg Pathol. 2013;37:484-95 7. Charli-Joseph Y, Saggini A, Doyle LA, Fletcher CD, Weier J, Mirza S, Vemula S, LeBoit PE. DNA copy number changes in tumors within the spectrum of cellular, atypical, and metastasizing fibrous histiocytoma. J Am Acad Dermatol. 2014;71:256-63.

bromas en la cara y en tejidos blandos profundos que metastatizaron a los ganglios linfáticos y los pulmones e incluso causaron la muerte de pacientes en algunos casos. 4-6 Morfológicamente, las neoplasias primarias y metastásicas mostraron características clásicas de las variantes mencionadas de dermatofibroma, y hasta ahora no se han identificado características mofológicas predictivas para separar el dermatofibroma benigno clásico del dermatofibroma benigno de metástasis. Nuestro caso mostró un comportamiento agresivo, con recidiva y metástasis pulmonar, pero a diferencia de los casos descritos con una transformación a un tumor sarcomatoso de alto grado con expresión inmunohistoquímica aberrante.

133


22Âş Curso de DermatopatologĂ­a

134


Caso del Seminario 27 Dr Dr.. A. Santos Briz. Hospital Clínico Universitario de Salamanca.

Figuras 1 y 2

Historia Clínica Varón de 52 años con antecedente de trasplante de pelo en tira en 2010. En 2015 comienza con picor en cuero cabelludo intenso y formación de escamas en la zona receptora del trasplante. Posteriormente, a pesar de tratamiento antiinflamatorio y sintomático continúa con picor intenso, observando disminución de la densidad del pelo. Con dermatoscopia se observa incremento de la vascularización, hiperqueratosis perifolicular e interfolicular. No se ha perdido la línea de implantación del pelo. No hay vellos. En la zona central del vértex presenta menor densidad y fondo eritematoso. No presenta pérdida de cejas ni alopecia en otras zonas del cuerpo. Tampoco receso en la línea de implantación. Hay miniaturización en los diámetros de los tallos y disminución del número de pelos por unidad folicular. Se realiza biopsia.

Descripción Microscópica El estudio histológico muestra una biopsia por cilindro que incluye 6 unidades foliculares, la mayoría de tallos terminales en anagen. Llama la atención la presencia de fibrosis concéntrica rodeando al infundíbulo y al istmo folicular en al menos 3 folículos. Se acompaña de un ligero infiltrado inflamatorio linfomonocitario perivascular en dermis media. La epidermis presenta modificaciones de carácter reactivo por rascado con acantosis y ligera hiperqueratosis. No se observan imágenes de lesión de la interfase, engrosamiento de la membrana basal, depósito intersticial de mucina ni infiltrados inflamatorios perianexiales en dermis media ni profunda. Diagnóstico ALOPECIA CICATRICIAL DE TIPO LIQUEN PLANO FOLICULAR EN ZONA RECEPTORA DE TRASPLANTE DE PELO 135


22º Curso de Dermatopatología

Figuras 3 y 4

Comentarios El liquen planopilar (LPP) es un trastornoción poco frecuente que produce una pérdida permanente del cabello a través del daño producido por la inflamación del folículo piloso.1,2 Se caracteriza clínicamente por la pérdida de ostia folicular, hiperqueratosis folicular y eritema perifolicular, frecuentemente con prurito. El vértex del cuero cabelludo es la zona más frecuentemente afectada.3-5 La alopecia frontal fibrosa (AFF), que se conosidera una variante frontal de LPP, se observa típicamente en mujeres posmenopáusicas que desarrollan una alopecia cicatricial progresiva en forma de una banda frontal.6 Tanto la AFF como el LPP comparten hallazgos histopatológicos similares con características clave que incluyen un infiltrado linfocítico de densidad variable alrededor del folículo piloso distal, la presencia de células apoptóticas en la vaina de la raíz externa del folículo piloso, fibrosis perifolicular progresiva y sustitución final del folículo por columnas de fibrosis cicatricial.7 Hasta la fecha se han descrito 27 casos de pacientes que han desarrollado lesiones de tipo LPP tras trasplante de pelo.8-10 La patogenia de esta asociación es desconocida. Se ha sugerido como posible causa el fenómeno de Koebner.11 En este caso, el desarrollo de las lesiones caracte-

rísticas de una enfermedad en particular ocurre en el sitio del trauma físico o algún otro estímulo nocivo (reacción isomórfica). Como alternativa, es posible que la causa original de la pérdida de cabello haya sido un mal diagnóstico incial y que la lesión se tratase de un LPP desde el principio con patrón que simula alopecia androgenética. La alopecia cicatricial inflamatoria, como la LPP, conlleva el riesgo de reactivación incluso en una etapa de agotamiento y después de varios años sin tratamiento. Puede ser que estos procedimientos de trasplante capilar o de estiramiento facial predispongan la reactivación de la LPP, como lo sugiere un informe de caso de foliculitis decalvans que se desarrolló después de 20 años de cirugía restauradora del cabello. Finalmente, el desarrollo de la LPP en estos pacientes puede representar una mera coincidencia, aunque la presencia de dos series de 10 y 17 pacientes publicados lo hacen poco posible.9,10

BIBLIOGRAFÍA 1. Harries MJ, Paus R. The pathogenesis of primary cicatricial alopecias. Am J Pathol 2010;177:2152–62. 136


2. Harries MJ, Paus Scarring alopecia and the PPAR-gamma connection. J Invest Dermatol 2009; 129:1066–70. 3. Harries MJ, Trueb RM, Tosti A et al. How not to get scar(r)ed: pointers to the correct diagnosis in patients with suspected primary cicatricial alopecia. Br J Dermatol 2009; 160:482–501. 4. Chieregato C, Zini A, Barba A et al. Lichen planopilaris: report of 30 cases and review of the literature. Int J Dermatol 2003; 42:342–5. 5. Cevasco NC, Bergfeld WF, Remzi BK et al. A case-series of 29 patients with lichen planopilaris: the Cleveland Clinic Foundation experience on evaluation, diagnosis, and treatment. J Am Acad Dermatol 2007; 57:47–53. 6. Kossard S, Lee MS, Wilkinson B. Postmenopausal frontal fibrosing alopecia: a frontal variant of lichen planopilaris. J Am Acad Dermatol 1997;

36:59–66. 7. Whiting DA. Cicatricial alopecia: clinicopathological findings and treatment. Clin Dermatol 2001; 19:211–25. 8. Kossard S, Shiell RC. Frontal fibrosing alopecia developing after hair transplantation for androgenetic alopecia. Int J Dermatol 2005; 44:321– 3. 9. Donovan J. Lichen planopilaris after hair transplantation: report of 17 cases. Dermatol Surg. 2012;38:1998-2004. 10. Chiang YZ, Tosti A, Chaudhry IH, Lyne L, Farjo B, Farjo N, Cadore de Farias D, Griffiths CE, Paus R, Harries MJ. Lichen planopilaris following hair transplantation and face-lift surgery. Br J Dermatol. 2012;166:666-370. 11. Koebner H. Zur Aetiologie der Psoriasis. Vierteljahrsschr Dermatol Syphilis 1876; 3:559.

137


22Âş Curso de DermatopatologĂ­a

138


Caso del Seminario 28 Dra. M.T ernández Figueras. M.T.. F Fernández Hospital Universitari General de Catalunya (Barcelona). Historia Clínica Varón de 66 años con severo daño actínico que ha recibido múltiples tratamientos locales. Tras el uso de imiquimod para tratar las queratosis actínicas de la zona preesternal aparece importante irritación local. La inflamación se extiende rápidamente a otras queratosis actínicas del tronco y, progresivamente, desarrolla lesiones eritematoescamosas diseminadas por tronco y extremidades que tienden a confluir (muestra A). En 1-2 semanas se encuentra prácticamente eritrodérmico (muestra B) con afectación facial y de dorso de manos que se acompaña de importante descamación palmar. Descripción Microscópica La primera biopsia del paciente (Figura 1) muestra tres focos afectación de características muy diferentes. En un extremo de la biopsia existe únicamente hiperqueratosis paraqueratósica. En el centro intensa acantosis afectando al ostium de un folículo piloso con hiperqueratosis y un pequeño tapón folicular. En el extremo contralateral se obser-

Figura 1

Figura 2 139


22º Curso de Dermatopatología

van fenómenos de acantólisis con disqueratosis, hiperqueratosis paraqueratósica y una capa basal muestra proyecciones lentiginosas cortas hacia epidermis de aspecto «tipo Darier» La segunda biopsia (Figura 2) muestra una acantosis de silueta psoriasiforme con leve espongiosis difusa e hiperqueratosis con paraqueratosis. No existe adelgazamiento significativo del espacio suprapapilar ni se observan microabscesos de neutrófilos. En dermis superficial se observa leve infiltrado linfoide con áreas de exocitosis a epidermis.

serían enfermedad de Grover o enfermedad de Darier (en base al acantólisis con disqueratosis) sobre piel con cambios psoriasiformes, pero parecen poco probables y pueden descartarse por la presentación clínica. El fármaco Imiquimod es un agonista del tolllike receptor- 7 que estimula la respuesta inmunológica facilitando la liberación de citoquinas proinflamatorias como interleukinas, IFN-±, TNF±. Se ha descrito también el desarrollo de pitiriasis rubra pilaris de forma secundaria al tratamiento fotodinámico para queratosis actínicas, probablemente por un mecanismo similar, ya que responde a inhibidores de TNF±.

Diagnóstico PITIRIASIS RUBRA PILARIS ASOCIADA A TRATAMIENTO CON IMIQUIMOD Comentarios En los textos de dermatopatología, la acantólisis aparece como un hallazgo histopatológico típico de la pitiriasis rubra pilaris. En la práctica, la acantólisis suele limitarse a cambios muy focales y sutiles. Sólo muy ocasionalmente son extensos y evidentes, planteando incluso el diagnóstico diferencial con pénfigo. No obstante, en la revisión de See et al, un 15,2% de las biopsias de mostraban pitiriasis rubra pilaris mostraban acantólisis (5 de 33 casos) y en 3 de ellos (9,1%) se identificaban escasas células disqueratóticas. Se trata por tanto de un hallazgo que puede servir como clave diagnóstica para distinguir esta enfermedad de la psoriasis y por otro lado, en los casos en que es muy manifiesta una posible fuente de confusión llevando al diagnóstico erróneo de Pénfigo o enfermedad de Grover. La aparición de pitiriasis rubra pilaris en relación con el tratamiento con imiquimod es bien un reacción adversa bien conocida y parece asociarse a fenómenos de acantólisis con mayor frecuencia de lo habitual en este proceso. Aun sin conocer la historia clínica, los hallazgos de la primera biopsia pueden sugerir el diagnóstico pitiriasis rubra pilaris, ya que muestran acantosis e hiperqueratosis centrada en el epitelio de un folículo junto con acantólisis y disqueratosis. La presencia de cambios psoriasiformes en la segunda biopsia realizada una semana después, cuando la lesión estaba más evolucionada, confirmaría diagnóstico, especialmente por la ausencia de adelgazamiento del espacio suprapapilar y la escasez de neutrófilos que irían en contra el diagnóstico de psoriasis. La correlación con la clínica es el dato que completa el diagnóstico. Los diagnósticos diferenciales alternativos

BIBLIOGRAFÍA 1. See SHC, Peternel S, Adams D, North JP. Distinguishing histopathologic features of acantholytic dermatoses and the pattern of acantholytic hypergranulosis. J Cutan Pathol. 2019 Jan;46(1):615. 2. Lilo MT, Yan S, Chapman MS, Linos K. Pityriasis Rubra Pilaris With Extensive Follicular Acantholysis Resembling Pemphigus Vulgaris: A Case Report. Am J Dermatopathol. 2019 Jan;41(1):37-39 3. Gómez-Moyano E, Crespo-Erchiga A, Vera Casaño A, Sanz Trelles A. Pityriasis rubra pilaris with focal acantholytic dyskeratosis during treatment with imiquimod 5% cream Actas Dermosifiliogr. 2010 Dec;101(10):898-900 4. López-Ferrer A, Dalmau J, Fernández-Figueras MT, Puig L. Pityriasis rubra pilaris triggered by photodynamic therapy with response to tumor necrosis factor ±-blocking agents and acitretin. Cutis. 2014 Mar;93(3):E6-7. 140


Caso del Seminario 29 Dra. M.T ernández Figueras. M.T.. F Fernández Hospital Universitari General de Catalunya (Barcelona).

Figura 1

Historia Clínica Varón de 62 años con buen estado general que acude por presentar una placa inflamatoria de 4 cm en pantorrilla izquierda que el paciente relaciona con una picadura de insecto (muestra A). En los 6 meses siguientes al diagnóstico, el paciente recibe 3 inyecciones locales de Glucantime obteniéndose una ligera reducción de tamaño (3,5 cm) pero con incremento de la induración y persistencia de las molestias locales. Se toma una segunda biopsia (muestra B).

mente engrosada que está separada del tejido subcutáneo con una delimitación lineal, paralela a la superficie epidérmica. A pequeño aumento se distinguen numerosos quistes de milium en superficie y áreas de denso infiltrado celular separadas por bandas anchas y oblicuas, paralelas o entrecruzadas, de aspecto necrobiótico, en cuya periferia se adivinan células gigantes multinucleadas. El examen del tejido a mayor aumento de la biopsia confirma la existencia de áreas necrobióticas con algunas empalizadas periféricas de histiocitos epitelioides y células gigantes multinucleadas que con frecuencia tienen núcleos dispuestos en herradura (células de Langhans) y presencia entre las áreas necrobióticas de muy abundante infiltra-

Descripción Microscópica La primera biopsia (Figura 1) muestra un punch de silueta rectangular con una dermis aparente141


22º Curso de Dermatopatología

Figuras 2, 3 y 4

Diagnóstico LEISHMANIASIS CUTÁNEA CON PATRÓN DE NECROBIOSIS LIUPOIDICA

do linfoplasmocitario. Tanto la silueta rectangular como el signo de la línea son típicos de lesiones esclerosantes como la morfea, pero la asociación a cambios de aspecto necrobiótico y quistes de millium junto con el infiltrado linfoplasmocitario intersticial sugieren más una necrobiosis lipoídica. No obstante, en la porción superficial se identifican pequeños granulomas de bordes mal definidos con histiocitos epitelioides de núcleos algo retorcidos que contienen estructuras sugestivas de corresponder a leishmania (Figura 2). El estudio inmunohistoquímico con el anticuerpo antileishmania demostró positividad (Figura 3). La segunda biopsia (Figura 4) presenta atrofia epidérmica con comedones y algunos quistes de Millium residuales. En la dermis superficial se observa infiltrado linfoide disperso que también aparece en torno a vasos todo el espesor dérmico está sustituido por colágeno denso con frecuentes hendiduras entre haces el que está esclerosis se dispone de manera concéntrica en torno a estructuras vasculares y se extiende a grasa subcutánea rodeando lobulillos adiposos. La tinción para detección de leishmania resulto negativa en esta segunda muestra.

Comentarios La leishmaniasis es una enfermedad de distribución mundial que puede mostrar una amplia variedad de manifestaciones clínicas e histopatológicas. En España la presentación más habitual es en forma del llamado botón de oriente. No obstante, es preciso considerar el diagnóstico leishmaniasis ante lesiones múltiples o diseminadas o procesos inflamatorios que no cuadren con otros diagnósticos. Estas formas inusuales de presentación en nuestro medio pueden tener lugar tanto en pacientes con leishmaniasis de importación (generalmente del norte de África o el sur de América) como en individuos en que, como el caso actual, la leishmaniasis se adquirió localmente. Al inicio, el aspecto microscópico de esta lesión planteaba el diagnóstico diferencial con necrobiosis lipoídica. No obstante, éste es un patrón que no puede considerarse específico y se ha descrito también en otros procesos granulomatosos. Tras el tratamiento, la lesión adquirió intensa esclerosis con 142


rasgos de morfea. De nuevo nos encontramos ante un patrón de respuesta que no es específico y se ha descrito en relación con otros procesos infecciosos. Por tanto, la leishmaniasis debería incluirse en el diagnóstico diferencial de los granulomas necrobióticos y las lesiones esclerosantes. En el seguimiento, el paciente no ha desarrollado otras lesiones tipo necrobiosis lipoídica o morfea, ni ha desarrollado diabetes.

2. Yang S, Draznin M, Fung MA. The «Line Sign» Is a Rapid and Efficient Diagnostic «Test» for Morphea: Clinicopathological Study of 73 Cases. Am J Dermatopathol. 2018 Dec;40(12):873878. 3. Fernandez-Flores A, Boada A, Fernández-Figueras MT. Generalized Leishmanides in an Immunodeprived Man. Am J Dermatopathol. 2017 Feb;39(2):e29-e33. 4. Fernandez-Flores A, Valerio L, Carrato C, Hernández-Gallego A, Fernández-Figueras MT. An Epidemic Outbreak of Cutaneous Leishmaniasis Presenting as Suppurative Folliculitis: A Study of 6 Cases. Am J Dermatopathol. 2017 May;39(5):363-366. 5. Tolkki L, Hokynar K, Meri S, Panelius J, Puolakkainen M, Ranki A. Granuloma Annulare and Morphea: Correlation with Borrelia burgdorferi Infections and Chlamydia-related Bacteria. Acta Derm Venereol. 2018 Mar 13;98(3):355-360

BIBLIOGRAFÍA 1. Torres-Guerrero E, Quintanilla-Cedillo MR, Ruiz-Esmenjaud J, Arenas R. Leishmaniasis: a review. F1000Res. 2017;6:750.

143


22Âş Curso de DermatopatologĂ­a

144


Turn static files into dynamic content formats.

Create a flipbook
Issuu converts static files into: digital portfolios, online yearbooks, online catalogs, digital photo albums and more. Sign up and create your flipbook.